You are on page 1of 154

Tp ch online ca cng ng nhng ngi yu Ton

Tp ch online ca cng ng nhng ngi yu Ton

Tp ch online ca cng ng nhng ngi yu Ton

Tp ch online ca cng ng nhng ngi yu Ton

EPSILON
Ch bin: TRN NAM DNG
Bin tp vin: V QUC B CN
Bin tp vin: L PHC L

S 1, ngy 13 thng 02 nm 2015

Tp ch online ca cng ng nhng ngi yu Ton

Tp ch online ca cng ng nhng ngi yu Ton

LI NG
Ban bin tp Epsilon

Epsilon, tc l rt nh, nhng khng bng 0. V nhiu epsilon


cng li c th tr thnh nhng ci ng k. C th l 1, l 2, c
th l v cng. iu quan trng l ta c bit cch kt hp cc
epsilon khc nhau li hay khng. Epsilon l t bo ca cng
ng, dnh cho cng ng. N l mt s khi u. Cn tip ni
nh th no s hon ton ph thuc vo s n nhn, ng h,
tr gip, tham gia ca cng ng. c c s xut hin u
n, ng hn, Epsilon s khng c bt c mt gii hn v s
trang ca mt k, s trang ca mt bi, v cng khng gii hn
ch , khng bt buc phi c mc ny, mc kia.
Ch ca Epsilon a dng nhng s ch yu l v ton v
cc vn lin quan, mc thng thc ph thng, truyn
b ton hc.
Epsilon lun mong mun nhn c s ng gp t pha cc
nh ton hc, cc nh khoa hc, cc thy c gio, cc bn sinh
vin, cc bn hc sinh v tt c nhng ngi yu ton v nhng
ngi yu nhng ngi yu ton. nng cao cht lng tp
ch, chng ti xin c php s trao i vi tng tc gi, cng
bin tp li cc bi bo ph hp.
S bo m cc bn ang c l s 1 ca tp ch. Trong s ny,
chng ti c tng cng 9 bi vit. Bn cnh cc bi lin quan
n k thi HSG cp quc gia (VMO) 2015 va qua, chng ti
cng gii thiu mt s bi vit thng thc, l thuyt Ton c
in v hin i.
Epsilon s c gng ra u n 2 thng 1 ln, vo cc ngy 13
ca cc thng chn. Chn ngy 13 th hin s quyt tm.
Vn s khi u nan. Chng ta hy c gng khi u. V c
gng i tip. i nhiu ngi, bn s i rt xa. . .
5

Tp ch online ca cng ng nhng ngi yu Ton

Tp ch online ca cng ng nhng ngi yu Ton

MC LC

Li ng . . . . . . . . . . . . . . . . . . . . . . . . . . . . . . . . . . . . . . . . . . . . . . . . . . 5

S phc v a thc
Trn Nam Dng . . . . . . . . . . . . . . . . . . . . . . . . . . . . . . . . . . . . . . . . . . 9

Thut ton phc hi s hu t


Nguyn Hng Sn . . . . . . . . . . . . . . . . . . . . . . . . . . . . . . . . . . . . . . 21

Ton hc gii tr v cc bi ton i nn


ng Nguyn c Tin . . . . . . . . . . . . . . . . . . . . . . . . . . . . . . . . 31

V bi hnh hc thi VMO 2015


Trn Quang Hng . . . . . . . . . . . . . . . . . . . . . . . . . . . . . . . . . . . . . . 47

V bi bt ng thc trong thi VMO 2015


V Quc B Cn . . . . . . . . . . . . . . . . . . . . . . . . . . . . . . . . . . . . . . . . 57

Phn tch v m rng trong cc bi ton t hp


L Phc L . . . . . . . . . . . . . . . . . . . . . . . . . . . . . . . . . . . . . . . . . . . . 101

Cc vn c in v hin i
Trn Nam Dng . . . . . . . . . . . . . . . . . . . . . . . . . . . . . . . . . . . . . . . 139

Bi ton chuyn xe Bus


L T ng Khoa . . . . . . . . . . . . . . . . . . . . . . . . . . . . . . . . . . . . . 147

10 Nhn xt v k thi VMO 2015 . . . . . . . . . . . . . . . . . . . . . 151


7

Tp ch online ca cng ng nhng ngi yu Ton

Tp ch online ca cng ng nhng ngi yu Ton

S PHC V A THC
Trn Nam Dng (HKHTN, HQG Tp HCM)

Tm tt
Trong k thi chn hc sinh gii Ton Quc gia nm hc
2014-2015 va qua, c 2 bi ton c th gii rt hiu qu
v ngn gn nu dng n s phc. Th nhng, s hc
sinh nm vng s phc s dng mt cch hiu qu li
khng nhiu, v cc bn rt vt v gii cc bi ton
cho bng cc phng php khc.
Trong bi vit nh ny, chng ti mun gii thiu trc ht
l cc ng dng ca s phc trong bi ton v a thc,
sau l ng dng ca s phc v a thc trong cc bi
ton t hp m.

1. S phc trong cc bi ton v a thc


Nghim ca a thc ng vai tr quan trng trong vic xc
nh mt a thc. C th nu a thc P(x) bc n c n nghim
x1 , x2 , . . . , xn th P(x) c dng P(x) = c(x x1 )(x x2 ) (x xn ).
Tuy nhin, nu ch xt cc nghim thc th trong nhiu trng
hp s khng c s nghim.
Hn na, trong cc bi ton phng trnh hm a thc, nu ch
xt cc nghim thc th li gii s l khng hon chnh. nh
l c bn ca i s v vy ng mt vai tr ht sc quan trng
trong dng ton ny. V ta s dng cch pht biu n gin
nht ca n: mt a thc vi h s phc (thc) lun c t nht
mt nghim phc. Di y ta xem xt mt s p dng.
Bi ton 1. Tm tt c cc a thc P(x) khc hng sao cho:
P(x) P(x + 1) = P(x2 + x + 1).
9

(1)

Tp ch online ca cng ng nhng ngi yu Ton

Li gii. Gi s l nghim ca P(x) = 0. Khi 2 + + 1 cng


l nghim. Thay x bng x 1 trong (1), ta thy rng
P(x 1) P(x) = P(x2 x + 1).
V P() = 0 nn 2 + 1 cng l nghim ca P(x) = 0.
Chn l nghim c m-un ln nht (nu tn ti vi nghim
vi m-un ln nht, ta chn mt trong s cc nghim ). Cch
chn nh vy suy ra |2 + + 1| 6 || v |2 + 1| 6 || v c
2 + + 1 v 2 + 1 u l nghim ca P(x) = 0.
Ta nhn xt rng 6= 0. Tip theo, ta c


2|| = (2 + + 1) (2 + 1) 6 |2 + + 1| + |2 + 1| 6 2||.

V u v v cui ca bt ng thc trn bng nhau nn du


bng phi xy ra, t y ta suy ra 2 + + 1 = (2 + 1) vi
mt hng s dng no . Hn na t tnh ln nht ca ||
ta cng c |2 + + 1| = |2 + 1| = ||. Nh vy = 1 v ta c
2 + + 1 = (2 + 1)
suy ra 2 + 1 = 0. T = i v x2 + 1 l tha s ca P(x). Nh
vy ta c th vit P(x) di dng:
P(x) = (x2 + 1)m Q(x)
trong Q(x) l a thc khng chia ht cho x2 + 1. Th ngc
tr li vo phng trnh (1), ta thy Q(x) cng tho mn:
Q(x) Q(x + 1) = Q(x2 + x + 1).
Nu Q(x) = 0 li c nghim th l lun trn y suy ra nghim
c m-un ln nht ca n phi l i. iu ny khng th xy
ra v x2 + 1 khng chia ht Q(x). Ta suy ra rng Q(x) l mt hng
s, gi s l c. Thay vo phng trnh ca Q, ta c c = 1. Nh
vy lp cc a thc tho mn phng trnh (1) l P(x) = (x2 +1)m
vi m l mt s nguyn dng no .
Bi ton 2. Tm tt c cc a thc P(x) khc hng sao cho:
P(x) P(x + 1) = P(x2 ).
10

Tp ch online ca cng ng nhng ngi yu Ton

Li gii. Gi s l nghim ca P(x) = 0. Khi t phng


trnh suy ra 2 , 4 , 8 , . . . cng l nghim ca P(x) = 0. T y
suy ra rng || = 0 hoc || = 1, v nu ngc li ta s thu
c dy v hn cc nghim ca P(x). Tng t, bng cch thay
x = 1, ta suy ra ( 1)2 cng
ca P(x).
Bng
l nghim


cc
2
2


l lun tng t, ta cng c ( 1) = 0 hoc ( 1) = 1.


Gi s rng || = 1 v ( 1)2 = 1. Vit = cos + i sin, ta c
1 = (1 cos) i sin

= 2 sin2 2i sin cos


2 
2
2

= 2 sin sin i cos


2
2
2

nn (1 )2 = 4 sin2 2 (cos + i sin), suy ra




(1 )2 = 4 sin2 = 2 2 cos.
2



Do (1 )2 = 1 nn 2 cos = 1. T y suy ra cos = 12 , ta tnh
. Gi s = 3 .
c = 3 hoc 5
3

2
2
Xt 2 cng l nghim ca
2 P(x)2 = 0. Nh vy2( 1) cng l
nghim ca P(x) = 0 v ( 1) = 2 2 cos 3 = 3. Mu thun
v mi nghim ca P(x) = 0 c m-un bng 0 hoc 1. Tng t
.
vi trng hp = 5
3

Nh vy, ta c th kt lun rng = 0, hoc 1 = 0. T y


P(x) c dng cxm (1 x)n , vi c l mt hng s khc 0 no v
m, n l cc s nguyn khng m khng ng thi bng 0.
Thay vo phng trnh cho, ta d dng kim tra c rng
c = 1 v m = n. Nh vy lp cc a thc tho mn iu kin
cho l P(x) = xm (1 x)m trong m l mt s t nhin.
Nghim phc ca a thc vi h s nguyn, trong nhiu trng
hp l cha kho chng minh tnh bt kh quy (trn Z v Q)
ca a thc . Chng ta tm hiu cc l lun mu trong vn
ny thng qua cc v d sau:
Bi ton 3 (IMO, 1993). Chng minh rng vi mi n > 1, a thc
xn + 5xn1 + 3 khng th phn tch thnh tch ca hai a thc c
bc khng nh hn 1 vi h s nguyn.
11

Tp ch online ca cng ng nhng ngi yu Ton

Li gii. Gi s x1 , x2 , . . . , xn l tt c cc nghim ca P(x). Khi


ta c P(x) = (xx1 )(xx2 ) (xxn ). Suy ra 3 = (1)n x1 x2 xn .
n1
Ta c vi mi i th xn
+ 3 = 0, suy ra
i + 5xi
3 = |xi |n1 |xi + 5|,

i = 1, 2, . . . , n.

(1)

Gi s ngc li rng a thc P(x) kh quy, tc l P(x) = Q(x)


S(x) vi Q(x), S(x) l cc a thc khng hng vi h s nguyn.
Th th r rng Q(x) s l tch ca mt s tha s x xi v S(x)
l tch ca cc tha s cn li.
Khng mt tnh tng qut, gi s:
Q(x) = (x x1 ) (x xk ),

S(x) = (x xk+1 ) (x xn ).

Suy ra |x1 x2 xk | v |xk+1 xn | l cc s nguyn c tch l 3.


Nh vy mt s bng 1 v mt s bng 3. Khng mt tnh tng
qut, gi s |x1 x2 xk | = 3 v |xk+1 xn | = 1.
Trong (1) cho i chy t 1 n k ri nhn v theo v, ta c




3k = |x1 xk |n1 (x1 + 5) (xk + 5) = 3n1 Q(5) .

Suy ra k > n 1. Nh vy S(x) l nh thc bc nht, suy ra P(x)


c nghim nguyn. Nhng nghim nguyn ca P(x) ch c th
l 1, 1, 3, 3. Kim tra li th chng u khng l nghim ca
P(x). Mu thun. iu ny chng t iu gi s l sai, tc l a
thc P(x) bt kh quy.
Bi ton 4 (Nht Bn, 1999). Chng minh rng a thc:
f(x) = (x2 + 12 )(x2 + 22 ) (x2 + n2 ) + 1
khng th phn tch thnh tch ca hai a thc h s nguyn bc
ln hn hay bng 1.
Li gii. Gi s ngc li f(x) = g(x) h(x) vi g(x), h(x) l cc
a thc vi h s nguyn c bc ln hn hay bng 1. Khi ,
rng f(ki) = 1 vi k = 1, 2, . . . , n, ta c
1 = g(ki) h(ki),

k = 1, 2, . . . , n.

Ch rng 1 ch c 4 cch phn tch thnh tch ca cc s


nguyn trong Z[i] l 1 1, (1) (1), i (i) v (i) i nn ta c
vi mi k {1, 2, . . . , n} th
 

g(ki), h(ki) (1, 1), (1, 1), (i, i), (i, i) .
12

Tp ch online ca cng ng nhng ngi yu Ton

Nh vy trong mi trng hp ta u c g(ki) = h(ki) = h(ki).


Nh th a thc g(x) h(x) c 2n nghim phn bit, trong khi
bc ca n nh hn 2n. Vy ta phi c g(x) h(x) l a thc
hng 0, tc l g(x) = h(x). T deg(g) = deg(h) = n.
V f(x) l a thc n khi nn ta c th gi s g(x), h(x) n
khi. Khi a thc g2 (x) h2 (x) c bc nh hn 2n. a thc
ny c t nht 2n nghim ki vi k {1, 2, . . . , n}. Suy ra
g2 (x) h2 (x) = 0. Ta khng th c g(x) = h(x) v g v h n
khi. Vy ta phi c g(x) = h(x). Nh th f(x) = g2 (x).
T y suy ra g2 (0) = f(0) = (n!)2 + 1. iu ny l khng th v
g(0) l s nguyn v n > 1.
Bi ton 5. Chng minh rng nu a thc P(x) = (x2 7x+6)2n +13
c th phn tch thnh tch ca hai a thc Q(x), S(x) vi h s
nguyn th Q(x) v S(x) u c bc 2n.
Li gii. Tht vy, gi s P(x) = Q(x) S(x). Gi x1 , x2 , . . . , x4n
l cc nghim phc ca P(x) th Q(x) v S(x) s l tch ca cc
tha s (x xi ). nh s li nu cn, ta gi s:
Q(x) = (x x1 )(x x2 ) (x xk ) vi 1 6 k < 4n.


2n
Ta c (xi 1)(xi 6)
= 13. T y suy ra


1
(xi 1)(xi 6) = 13 2n
.

()


(1x1 ) (1xk )
Mt khc, (1x1 ) (1xk ) = Q(1) nguyn nn

cng nguyn.
Tng t |(6 x1 ) (6 xk ) nguyn. T y suy

ra m = (x1 1)(x1 6)(x2 1)(x2 6) (xk 1)(xk 6) nguyn.
k
Nhng theo () th m = 13 2n . Suy ra k = 2n. Vy Q(x), S(x) u
phi c bc l 2n.
Ch t kt qu bi ton ny d dng suy ra kt qu bi 2 ca
thi hc sinh gii Ton Quc gia nm hc 2013-2014:
Bi ton 6 (VMO, 2014). Cho a thc P(x) = (x2 7x + 6)2n + 13
vi n l s nguyn dng. Chng minh rng a thc P(x) khng
th biu din c di dng tch ca n + 1 a thc khc hng s
vi h s nguyn.
13

Tp ch online ca cng ng nhng ngi yu Ton

Nu a thc P(x) chia ht cho a thc Q(x) th mi nghim ca


Q(x) u l nghim ca P(x). Tnh cht n gin ny l cha
kho gii nhiu bi ton v s chia ht ca a thc. Chng
ta xem xt mt s v d.
Bi ton 7. Vi gi tr no ca n th a thc x2n + xn + 1 chia ht
cho a thc x2 + x + 1?

Li gii. Ta c = 21 + i 23 = cos 2
+ i sin 2
l nghim ca a
3
3
2
2
thc Q(x) = x + x + 1. V a thc x + x + 1 l bt kh quy nn a
thc P(x) = x2n + xn + 1 chia ht cho Q(x) khi v ch khi P() = 0.
iu ny tng ng vi
cos
hay

4n
2n
2n
4n
+ i sin
+ cos
+ i sin
+ 1 = 0,
3
3
3
3



2n
2n

cos 3 2 cos 3 + 1 = 0



2n
2n

sin
2 cos
+1 =0
3
3

T h phng trnh trn, ta d dng suy ra 2 cos 2n


+ 1 = 0,
3
tc n phi l s khng chia ht cho 3. Vy vi n = 3k + 1 hoc
n = 3k + 2 th P(x) chia ht cho Q(x).
Trong v d di y, mt ln na, cn ca n v li ng vai
tr then cht trong vic i n li gii.
Bi ton 8 (USAMO, 1976). Cho P(x), Q(x), R(x), S(x) l cc a
thc sao cho
P(x5 ) + x Q(x5 ) + x2 R(x5 ) = (x4 + x3 + x2 + x + 1) S(x).

(1)

Chng minh rng P(x) chia ht cho x 1.


2i

Li gii. t = e 5 th 5 = 1. Thay x ln lt bng , 2 , 3 ,


4 , ta c cc phng trnh:
P(1) + Q(1) + 2 R(1) = 0,
P(1) + 2 Q(1) + 4 R(1) = 0,
P(1) + 3 Q(1) + 6 R(1) = 0,
P(1) + 4 Q(1) + 8 R(1) = 0.
14

Tp ch online ca cng ng nhng ngi yu Ton

Nhn cc phng trnh t 1 n 4 ln lt vi , 2 , 3 ,


4 , ta c cc phng trnh sau:
P(1) 2 Q(1) 3 R(1) = 0,
2 P(1) 4 Q(1) R(1) = 0,
3 P(1) Q(1) 4 R(1) = 0,
4 P(1) 3 Q(1) 2 R(1) = 0.
Cng tt c 8 phng trnh li theo v v s dng tnh cht ca
l 1++2 +3 +4 = 0, ta c 5P(1) = 0, tc x1 | P(x).
Bi ton 9 (VMO, 2015). Cho dy a thc fn (x) c xc nh
bi f0 (x) = 2, f1 (x) = 3x, fn (x) = 3x fn1 (x) + (1 x 2x2 )fn2 (x)
vi mi n > 2. Tm tt c cc gi tr n a thc fn (x) chia ht
cho a thc x3 x2 + x.
Li gii. Vi mi x c nh, ta xt dy s ai = fi (x), khi ta
c a0 = 2, a1 = 3x, an = 3xan1 + (1 x 2x2 )an2 vi mi n > 2.
Xt phng trnh c trng X2 3xX + 2x2 x 1 = 0 c hai
nghim l x + 1 v 2x 1. T ta c dng tng qut ca (an )
l an = c1 (x + 1)n + c2 (2x 1)n . T cc iu kin ban u ta suy
ra c1 = c2 = 1, tc l an = (x + 1)n + (2x 1)n . iu ny ng vi
mi gi tr ca x, do ta c fn (x) = (x + 1)n + (2x 1)n .
By gi, ta tm n sao cho fn (x) = (x + 1)n +(2x 1)n chia ht cho
a thc Q(x) = x3 x2 + x. V 0 v = 1+i2 3 l nghim ca Q(x)
nn iu ny xy ra khi v ch khi 0 v l nghim ca fn (x).
c iu ny ta phi c:
i) 1n + (1)n = 0, suy ra nl.


n

n
3+i n
+ in = 0. Chuyn cc s
ii) 3+i2 3 + i 3 = 0, tc
2
phc sang dng lng gic v dng cng thc ly tha, ta
c cos n
+ i sin n
+ cos n
+ i sin n
= 0, tc n phi l
6
6
2
2
s chia ht cho 3.
Kt hp hai iu kin i) v ii), ta suy ra iu kin cn v
fn (x) chia ht cho Q(x) l n = 6m+3 vi m nguyn khng m.

15

Tp ch online ca cng ng nhng ngi yu Ton

2. S phc v a thc trong bi ton m


S phc c nhng ng dng rt hiu qu trong cc bi ton
m. V vai tr trung tm trong k thut ng dng s phc vo
cc bi ton m tip tc l cn nguyn thu ca n v. Ch
l nu l cn nguyn thu bc n ca n v th ta c
i) 1 + + + n1 = 0.
ii) 1 + k + + k(n1) = 0 vi (k, n) = 1.
y chnh l tnh cht quan trng ca cn nguyn thu thng
c s dng trong gii ton.
Bi ton 10 (Chn i tuyn PTNK, 2009). Tm s tt c cc s
c n ch s lp t cc ch s 3, 4, 5, 6 v chia ht cho 3.
Li gii. Gi cn l s cc s c n ch s lp t cc ch s
3, 4, 5, 6 v chia ht cho 3. Gi l mt nghim ca phng
trnh 2 + + 1 = 0. Khi 3 = 1 v 2k + k + 1 nhn gi tr = 0
nu k khng chia ht cho 3 v = 3 nu k chia ht cho 3.
Xt a thc P(x) = (x3 + x4 + x5 + x6 )n . D thy cn chnh l bng
tng cc h s ca cc s m chiaPht cho 3 trong khai
P2n trin
6n
k
ca P(x). Ni cch khc, nu P(x) = k=0 ak x th cn = k=0 a3k .
Mt khc ta c
2

P(1) + P() + P( ) =

6n
X

ak (1 + + ) = 3

k=0

2n
X

a3k .

k=0

Cui cng, do P(1) = 4n , P() = P(2 ) = 1 nn ta c


cn =

2n
X

a3k =

k=0

4n + 2
.
3

Bi ton 11 (VMO, 2015). Cho K N . Tm s cc s t nhin n


khng vt qu 10K tha mn ng thi cc iu kin sau:
i) n chia ht cho 3.
ii) Tt c cc ch s trong biu din thp phn ca n u thuc
tp hp A = {2, 0, 1, 5}.
16

Tp ch online ca cng ng nhng ngi yu Ton

Li gii. V 10K khng chia ht cho 3 nn ta ch cn xt cc s


t 0 cho n 99 9 (K ch s 9). B sung cc ch s 0 vo trc
nu cn thit, ta a v xt cc s c dng a1 a2 aK vi ai
thuc {2, 0, 1, 5}. Ta cn m cc s nh vy v chia ht cho 3.
Ch l a1 aK chia ht cho 3 khi v ch khi a1 + +aK chia ht
cho 3, ta a bi ton v vic m s cc b (a1 , a2 , . . . , aK ) AK
sao cho a1 + a2 + + aK chia ht cho 3.
Tip theo, hon ton tng t nh bi trn, xt a thc:
P(x) = (x2 + 1 + x + x5 )K .
Ta c
X

P(x) = (x2 + x + 1 + x5 )k =

xa1 +a2 ++aK .

(a1 , a2 , ..., aK )Ak

Tng cc h s ca P(x) bng s cc b (a1 , . . . , aK ) AK v bng


4K . Hn na s cc b (a1 , . . . , aK ) AK sao cho a1 + a2 + + aK
bng tng cc h s ca cc s m chia ht cho 3 trong P(x).
t P(x) = a0 + a1 x + a2 x2 + a3 x3 + a4 x4 + . Ta cn tnh:
S = a0 + a3 + a6 + .
Gi l nghim ca phng trnh x2 + x + 1 = 0 th ta c 3 = 1.
T d dng suy ra 1 + k + 2k nhn gi tr bng 0 vi mi k
khng chia ht cho 3 v bng 3 vi k chia ht cho 3.
()
Ta c
P(1) = a0 + a1 + a2 + a3 + a4 + ,
P() = a0 + a1 + a2 2 + a3 3 + a4 4 + ,
P(2 ) = a0 + a1 2 + a2 4 + a3 6 + a4 8 + .
p dng tnh cht (), ta suy ra
P(1) + P() + P(2 ) = 3a0 + 3a3 + 3a6 +
2

2K

4K

)
Suy ra S = P(1)+P()+P(
= 4 + 3 + . Cui cng, li p dng
3
K
tnh cht (), ta suy ra S = 4 31 nu K khng chia ht cho 3 v
K
S = 4 3+2 nu K chia ht cho 3.

17

Tp ch online ca cng ng nhng ngi yu Ton

Khi ta lm vic vi cc tp con, tc l cc t hp khng lp th


m hnh nhng a thc trn y khng s dng c na. Vi
cc bi ton ny, ta cn n mt m hnh khc.
Bi ton 12. Cho X = {0, 1, . . . , 25}. Tm s cc tp con 7 phn t
c tng cc phn t chia ht cho 19.
Li gii. Vi mi tp con A X, gi S(A) l tng cc phn t
ca A. Ta cng quy c S() = 0. Vi mi i = 0, 1, . . . , 18, t:


P(i) = A X | |A| = 7 v S(A) i (mod 19) .
Ta cn tnh P(0). Gi a l cn nguyn thy bc 19 ca 1. Khi
1 + a + a2 + + a18 = 0 v x19 1 = (x 1)(x a) (x a18 ).
Xt a thc Q(x) = (x 1)(x a)(x a2 ) (x a25 ). Ta tnh h s
ca x19 trong Q(x) bng 2 cch. Mt mt, nu khai trin Q(x) ra
th c x19 , ta cn ly x t 19 du ngoc, cn 7 du ngoc
khc s ly cc s c dng ak vi k thuc {0, 1, . . . , 25}. Nh th,
ta s c tng cc s c dng aS(A) vi A chy qua tt c cc tp
con 7 phn t ca X. Ch rng aS(A) ch ph thuc vo s d
khi chia S(A) cho 19 ( l l do ti sao ta ly cn bc 19 ca
n v) nn t y d dng suy ra tng ni trn bng:
h
i

P(0) + P(1) a + + P(18) a18 .
Mt khc, P(x) = (x19 1)(x 1)(x a) (x a6 ). Suy ra h s
ca x19 bng 1 a a2 a6 = a2 . T y suy ra








P(0) + P(1) a + P(2) 1 a2 + + P(18) a18 = 0.

iu ny ng vi mi a l nghim ca phng trnh:

1 + x + x2 + + x18 = 0.








Suy ra a thc P(0) + P(1) x + P(2) 1 x2 + + P(18) x18 t
l vi a thc 1 + x + + x18 v v th:






P(0) = P(1) = P(2) 1 = = P(18) .



Nh vy, tt c cc P(i) , i 6= 2, bng nhau v bng
C7 1
|P(2)| ln hn ng 1 n v! Vy p s l 1919 .
18

C719 1
.
19

Ring

Tp ch online ca cng ng nhng ngi yu Ton

Cui cng, ta p dng hiu qu phng php trn y vo mt


bi ton thi v ch Quc t, mt bi ton p ca IMO 1995.
Bi ton 13 (IMO, 1995). Cho p l mt s nguyn t l. Tm s
cc tp con A ca tp hp {1, 2, . . . , 2p}, bit rng:
i) A cha ng p phn t;
ii) Tng cc phn t ca A chia ht cho p.
Li gii. Xt a thc P(x) = xp1 + xp2 + + x + 1. a thc ny
c p 1 nghim phc phn bit. Gi l mt nghim bt k ca
P(x). Ch rng , 2 , . . . , p1 l p 1 nghim phn bit ca
P(x) v p = 1. Do , theo nh l Vieta:
xp 1 = (x 1)(x )(x 2 ) (x p1 ).
Xt a thc Q(x) = (x )(x 2 ) (x 2p ) v gi:


H = A {1, 2, . . . , 2p} | |A| = p .
P
i
Gi s Q(x) = 2p
i=0 ai x . Khi :
X
X
ap =
S(A) , S(A) =
x.
AH

xA

V nu S(A) i (mod p) th S(A) = i nn


ap =

p1
X

ni i ,

i=0

trong ni l s cc A H sao cho S(A) i (mod p). Mt khc,


Q(x) = (xp 1)2 , suy ra ap = 2. Thnh th:
p1
X

ni i = 2.

i=0

()

P
i
Xt a thc R(x) = p1
i=1 ni x + n0 2. T ng thc () suy ra
l mt nghim ca R(x). V degP = degR v l mt nghim bt
k ca P(x) nn P(x) v R(x) ch sai khc nhau hng s nhn.
T np1 = np2 = = n1 = n0 2, suy ra
n0 2 =

Cp2p 2
np1 + np2 + + n1 + n0 2
=
.
p
p

Vy p s ca bi ton l n0 = 2 +
19

Cp
2p 2
.
p

Tp ch online ca cng ng nhng ngi yu Ton

3. Bi tp
Bi ton 1. Tm tt c cc a thc P(x) tha mn iu kin:
P(x) P(x + 1) = P(x2 + 1).
Bi ton 2. Cho n l s nguyn dng. Chng minh rng a
thc xn + 4 kh quy khi v ch khi n chia ht cho 4.
Bi ton 3. Chng minh rng a thc (x2 7x + 6)2n + 13 bt
kh quy vi mi n nguyn dng. (Ch rng bi ny mi ch
l gi thit, bn c c th ph nh bi ton nu kt qu sai.)
Bi ton 4. Vi gi tr no ca n th a thc (x + 1)n + xn + 1
chia ht cho a thc x2 + x + 1?
Bi ton 5. C bao nhiu tp con ca X = {1, 2, . . . , 2015} c
tng cc phn t chia ht cho 3?
Bi ton 6 (IMO 2014 Training Camp). C bao nhiu s t
nhin c 9 ch s khng cha ch s 0 v chia ht cho 11?
Bi ton 7. Cho ba s nguyn dng m, n, p, trong m > 1
v n + 2 0 (mod m). Tm s b (x1 , x2 , . . . , xp ) gm p s nguyn
dng sao cho tng (x1 + x2 + + xp ) chia ht cho m, trong
mi s x1 , x2 , . . . , xp u khng ln hn m.

20

Tp ch online ca cng ng nhng ngi yu Ton

THUT TON
PHC HI S HU T
Nguyn Hng Sn (University of Warsaw)

1. M u
Cch y khng lu ti c cc bn tr mt bi ton nh,
nhng mang tnh thc t, nh sau:
Mt v gio s ton-tin rt cn thn nhng ng tr. Cch y
vi hm ngn hng gi ng mt bc th thng bo mt khu
ca th tn dng. Mt khu l mt s c 6 ch s: abcdef. ng
khng mun gi li bc th v s n c th lt vo tay k gian.
V vy ng dng 1 chic my tnh xch tay n gin (gm 4
php tnh +., , v 10 ch s) tnh t s abc def. ng
nhn c kt qu gn ng l 0, 195323246 v ghi nh li
ln mt t giy.
Lm th no v gio s c th tm li c mt khu trong
thi gian ngn nht nu ng ch c trong tay chic my tnh
xch tay n gin v mt khu l g?
Thc ra bi ton ny lin quan n mt s ng dng ca thut
ton Euclid v l thuyt v phn s chui trong s hc. Sau y
chng ta s ln lt tm hiu cc l thuyt lin quan, li gii
ca bi ton trn, v th lm cc bi tp tng t.

2. Thut ton Euclid


y l mt trong cc phng php tm c s chung ln nht
SCLN(a, b) ca hai s t nhin. Khong 300 nm trc Cng
Nguyn, Euclid nh ton hc c ngi Hy lp m t thut
ton ny trong cun c s (Elements).

21

Tp ch online ca cng ng nhng ngi yu Ton

tng chnh ca thut ton ny l:


Nu k, r l hai s nguyn sao cho a = kb + r th:
SCLN(a, b) = SCLN(r, b).
Trong thut ton Euclid, ta s chn k l phn nguyn ca php
chia a cho b (k = ba/bc), cn r l phn d khi chia a cho b
(r = a ba/bc b). Thut ton ny c m t dng biu
Hnh 3.1. V d nu mun tm SCLN ca 2 s 324 v 918 th
cc bc ca thut ton s nh sau:
STT
1.
2.
3.
4.
5.

a
b ba/bc r = a mod b
d
324 918
0
576
918 324
2
270
324 270
1
54
270 54
5
0
54
0
54

Nhp 2 s t nhin a, b

Kim tra
b 6= 0?

b=0

b 6= 0
r := a mod b
a := b
b := r

d := a
Xut d

STOP;

Hnh 3.1: Thut ton Euclid tm c s chung ln nht ca


hai s t nhin a, b.

22

Tp ch online ca cng ng nhng ngi yu Ton

3. Lin phn s
Lin phn s hu hn l mt biu thc c dng:
1

a0 +

a1 +
a2 +

1
+

1
an

trong a0 Z, a1 , . . . , an l cc s nguyn dng v an > 1.


Lin phn s trn c k hiu l [a0 : a1 , a2 , . . . , an ], trong
n chnh l di ca lin phn s.
Nh ta bit mi s hu t u c th c vit di dng ab ,
trong a Z l s nguyn cn b N {0} l s nguyn dng.
Mt phn s c th chuyn thnh lin phn s theo phng
php lp i lp li 2 bc (1) v (2) sau y: (1) tch ra phn
nguyn, (2) nghich o phn phn s.
V d phn s

1517
c th chuyn thnh lin phn s nh sau:
1073

1517
444
=1+
=1+
1073
1073

1
1
=1+
185
1
2+
2+
74
444
2 + 185

=1+
2+
Nh vy ta chuyn

1
2+

1517
1073

=1+

1
37
2+ 74

2+

1
2 + 2+1 1
2

thnh lin phn s [1 : 2, 2, 2, 2, 2].

23

Tp ch online ca cng ng nhng ngi yu Ton

Bn c tinh c th thy nhiu im tng t gia phng


php tm lin phn s v thut ton Euclid. Thc vy, nu ta
p dng thut ton Euclid cho hai s 1517 v 1073 th qu trnh
tnh ton s nh sau:
STT
a
b ba/bc r = a mod b
d
1. 1517 1073
1
444
2. 1073 444
2
185
3. 444 185
2
74
4. 185
74
2
37
5.
74
37
2
0
6.
37
0
37
D dng nhn ra s trng hp gia lin phn s [1 : 2, 2, 2, 2, 2]
v ct ba/bc ca thut ton Euclid. Nh vy nu p dng thut
ton Euclid cho a v b, nhng trong mi bc ta vit ra gi tr
ca ba/bc th ta s c khai trin ca phn s ab thnh dng
lin phn s.
Nhp 2 s t nhin a, b

Nhp

a0 := bxc;
r := x a0 ;
n := 0;

n := 0;

b 6= 0?

xR

b=0
Xut an ;
n := n + 1;

b 6= 0
k := ba/bc
r := a kb
a := b
b := r

r 6= 0?

r=0

r 6= 0
Xut an = k ;
n := n + 1;

STOP

an := b1/rc;
r := 1/r an ;

STOP

Hnh 3.2: Thut ton Euclid tm lin phn s cho phn s


(tri) v cho s thc x R (phi).
24

a
b

Tp ch online ca cng ng nhng ngi yu Ton

Thut ton trn c m t Hnh 3.2.a (tri). Da vo thut


ton ta c th chng minh nh l sau:
Mi s hu t u c th khai trin di dng mt lin
phn s hu hn [a0 : a1 , a2 , . . . , an ], y a0 l phn
nguyn ca s hu t cho.
Lin phn s c cc nh ton hc nh Rafael Bombelli
(1572), Pietro Catldi (1613), Daniel Schwenter (1625), Wallis (1695)
hoc nh thin vn hc Christian Huygens (1698) bit n t
th k XVI v XVII.
Tuy nhin phi n th k XVIII nh ton hc Leonhard Euler
(1707-1783) mi bt u nghin cu mt cch h thng lin
phn s. Euler khng ch a ra thut ton m cn tm ra rt
nhiu lin phn s.
Thc ra thut ton ca Euler l trng hp tng qut ca thut
ton chuyn s hu t thnh lin phn s. N c th p dng
cho mt s thc xbt k (xem Hnh 3.2.b (phi)). p dng thut
ton ny cho x = 2 ta s c:

1
1
1

=1+
=1+
2=1+ 21=1+
1
2 + 2+1
2+1
2+ 21
=1+

1
2+

1
2+ 21

= = 1 +

1
2+

1
1
2+ 2+

Nh vy s 2 c th biu din di dng lin phn s v hn


tun hon = [1 : 2, 2, 2, . . .] = [1 : 2].
Euler pht hin ra rng nu mt s c th biu din di
dng lin phn s v
hn tun hon (t mt v tr no ) th s
phi c dng a + b c, a,
b, c Q (hay cn gi l s i s bc

51
hai). V d t l vng = 2 c th biu din dng lin phn
s gm ton s 1 (xem bi tp 2).
Hoc nu x = [2 : 2, 2, 2, . . .] = [2 : 2], th ta c x = 2 + x1 t

suy ra x = 1 + 2. Nhng phi 20 nm sau ch l o mi c


chng minh bi Lagrange (1768):
Mi s i s bc 2 u c th khai trin thnh lin
phn s tun hon (bt u t mt v tr no ).
25

Tp ch online ca cng ng nhng ngi yu Ton

Chng ta va nhn ra rng khai trin lin phn s ca cc s


hu t v c cc s v t trng c v nh tin li hn khai trin
thp phn. Mt cu hi c tnh trit l v lch s c t ra l:
ti sao trong trng ph thng chng ta s dng s thp phn
nhng li khng dng lin phn s? Cu tr li c l l do php
cng v nhn cc lin phn s khng h d dng g. M cng
c th thiu cc phng php (thiu cc thut ton hu hiu)
l do cc nh ton hc (tin hc) cha tm k. Ta ch c th kt
lun rng hnh nh ca ton hc ngy nay khng phi l duy
nht, v n hon ton c th chuyn sang hng khc.

4. Phc hi s hu t
4.1. V d minh ha
Chng ta hy quay li bi ton ban u. V gio s c th kim
tra tt c cc phn s dng abcdef cho n khi tm c phn
s c gi tr nh yu cu. Tuy nhin phng php ny khng
hiu qu v mt qu nhiu thi gian.
Trc ht chng ta c th ch rng nu qp v sr l hai phn
s c t s v mu s u l cc s c ba ch s v hai phn
s
p r ging6nhau t nht l n ch s th 6 sau du phy th
< 10 . T suy ra
q
s
|ps qr| 6 qs 106 < 103 103 106 = 1.

V p, s, q, r l cc s nguyn v 0 6 |ps qr| < 1, t suy ra


ps qr = 0. iu ny tng ng vi qp = sr .
Phng php hiu qu hn chnh l khai trin s x = 0, 195323246
thnh dng lin phn s x = [a0 : a1 , a2 , . . .].
Nu t xn = [a0 : a1 , . . . , xn ] l lin phn s dng n s hng u
tin ca lin phn s x = [a0 : a1 , a2 , . . .], ta s thy xn l xp x
ca x v n cng ln th xn cng c gi tr gn x. V vy ta s s
dng thut ton trn cho n khi c mt lin phn s gn
ging vi phn s cn tm.
26

Tp ch online ca cng ng nhng ngi yu Ton

S dng thut ton Hnh 3.2.b (phi) ta ln lt c:


a0 = 0 r = 0, 195323246 x0 = 0
1
a1 = 5 r = 0, 119718315 x1 =
5

8
1
1 =
41
5+ 8
1
17
a3 = 2 r = 0, 833338458 x3 =
1 =
87
5 + 8+ 1
2
1
25
a4 = 1 r = 0, 199992620 x4 =
=
1
128
5 + 8+ 1
a2 = 8 r = 0, 352940538 x2 =

2+ 1
1

a5 = 5 r = 0, 000184506 x5 =

Kim tra li ta thy rng


s cn tm l 142727 .

142
727

1
5+

1
8+

2+

1
1
1+ 1
5

142
727

= 0.195323246 . . . Vy m s v gio

4.2. Trng hp tng qut


tng qut ha bi ton trn chng ta xt vn sau y:
VN PHC HI S HU T: Cho hai s nguyn dng K, M
hy tm hai s nguyn dng u, v sao cho:
DK1 :
DK2 :

0 6 u, v < N



u
K 6 1
v M M

(N l s nguyn dng cho trc)


u K
(2 phn s ,
gn bng nhau)
v M

(1)
(2)

Paul Wang nghin


cu vn phc hi s hu t trong
q
M
trng hp N =
. Vi la chn ny ta c th chng minh
2
rng nu tn ti li gii cho vn phc hi s hu t th li
gii ny l duy nht.
Thc vy, gi s tn ti 2 li gii (u1 , v1 ) v (u2 , v2 ) tha mn
cc iu kin (1) v (2). Ta c:




u1 u2 u1
u1

K
K
6 + 6 2 ,
v1
v2 v1
M v1
M M
27

Tp ch online ca cng ng nhng ngi yu Ton

t suy ra
2v1 v2
2N2
|u1 v2 u2 v1 | 6
<
.
M
M
q
Trong trng hp N = M
, ta c |u1 v2 u2 v1 | < 1. Ngoi ra, do
2

|u1 v2 u2 v1 | l s nguyn nn ta suy ra rng u1 v2 u2 v1 = 0 hay


u1
= uv22 . Hn na, t iu kin (2) suy ra
v1
|Mu Kv| 6 v < N N < r = Mu Kv < N.
K
T suy ra nu hai phn s uv , M
gn bng nhau th tn ti
mt s nguyn r sao cho |r| < N v r Kv mod M. Lc K
c gi l ng d vi phn s vr modulo M v c k hiu l
r
K mod M. Nh vy vn phc hi s hu t c th pht
v
biu mt cch tng ng nh sau:

VN PHC HI S HU T (bin th): Cho trc hai s


nguyn dng K, M, hy tm cp s nguyn (r, v) tha mn ng
thi hai iu kin sau y:
p
p
DK3 : 0 6 |r| < M/2 v 0 < v < M/2
(3)
DK4 : r Kv (mod M)
(4)

Nhp 2 s t nhin K, M

r1 := M ;
r2 := K;

O(r1, r2);
O(v1, v2);

v2

v1 := 0;
v2 := 1;

p
M/2?

NG

Xut (0, 0);


(tc l Khng tn ti )
STOP;

SAI
Q := br1 /r2 c;
r1 := r1 Qr2 ;
v1 := v1 Qv2 ;

SAI

p
r2 < M/2?

NG

Xut

!
v2
r2 , |v2 | ;
|v2 |

STOP;

Hnh 3.3: Thut ton phc hi s hu t RATCONVERT.


28

Tp ch online ca cng ng nhng ngi yu Ton

D thy rng nu tn ti cp s (r, v) tha mn hai iu kin


cng s tha mn
(3) v (4) th cp s (u, v), trong u = Kvr
M
c hai iu kin (1) v (2).
P. Wang (1981) cn xut mt thut ton gii quyt vn
phc hi s hu t trong trng hp li gii tn ti. Thut ton
ny da vo tng ca thut ton Euclid v c mang tn
RATCONVERT (xem Hnh 3.3).
Bng 1.1 c trnh by trang sau minh ha thut ton RATCONVERT qua v d chng 1.

5. Mt s bi tp tham kho
Bi ton 1. Chng minh rng mi s hu t u c th biu
din bng ng hai cch khc nhau di dng lin phn s
hu hn.
Bi ton 2. Chng minh rng [1 : 1] =

51
.
2

Bi ton 3. Via l s nguyn dng hy tm khai trin lin


phn s ca s a2 + 1.
Bi ton 4. Hy kim chng rng phng php phc hi s
hu t (c hai phng php trnh by chng 4.1 v 4.2) vn
hiu qu khi ta ch dng 6 ch s sau du phy (tc l 0, 195323)
nhng nu ch dng nm ch s (0, 19532) th s khng th phc
hi c m s ban u.
Bi ton 5. p dng thut ton phc hi s hu
t tm xp



p
x hu t q ca s 2 ' 1, 414213562373 sao cho qp 2 < 0, 001.

29

30

Lt
Q
r1
r2
v1
v2
r = 158 v = 727 u =

Kv r
= 142
M

1
2
3
4
5
6
5
8
2
1
5
195323246 23383770 8253086 6877598 1375488
Xut
23383770 8253086 6877598 1375488
158 (158, 727);
1
5
41
87
128
5
41
87
128
727
STOP;

p
M/2 = 22360, 7

Bng 1.1: V d minh ha cho thut ton RATCONVERT.

1000000000
195323246
0
1

Bt u

M = 1000000000 K = 195323246

Tp ch online ca cng ng nhng ngi yu Ton

Tp ch online ca cng ng nhng ngi yu Ton

TON HC GII TR V
CC BI TON I NN
ng Nguyn c Tin (Trento, Italy)

1. Ton hc gii tr
Ton hc gii tr (Recreational Mathematics) l mt thut ng
chung cho nhng vn ton hc m mc ch ch yu dng
gii tr. i khi nhng bi ton gii tr ny c xut hin
di dng giai thoi hay nhng ch c lin quan n ngh
thut v ton, nhng ph bin nht l di dng nhng cu
m li gii a phn ch cn nhng kin thc ton hc s cp.
Tuy khng phi l mt ngnh nghin cu nghim tc, nhng
xuyn sut chiu di pht trin ca ton hc, ta lun thy s
song hnh ca nhng bi ton gii tr i cng vi nhng pht
minh ca ton hc, i khi mt bi ton cng c th l
bi m u cho c mt lnh vc nghin cu.
Nhng v d tiu biu c th k n bi ton on tui ca i-phng (Diophantus, nh ton hc Hi Lp, th k th 3 sau
cng nguyn) v s ra i ca phng trnh nghim nguyn;
hay bi ton m th ca Leonardo Bonacci cng mi lin h
vi dy s mang tn ng: Fibonacci; ri mt trng hp khc l
bi ton by cy cu Konigsberg (hay cn c gi l by cy
cu Euler) vi l thuyt th; v rt nhiu v d khc.
Ngun gc tuy t xa xa nh th, nhng ton hc gii tr ch
tht s c h thng v ph bin vo khong cui th k 19
nh cng ca nhng ngi tin phong nh Charles Lutwidge
Dodgson (1832-1898), nh vn, nh ton hc, nh thn hc,
nhip nh gia ngi Anh c rt nhiu ngi bit n vi bt
danh Lewis Carroll, tc gi ca Alice lc vo x thn tin; ri
tip theo l Yakov Perelman (1882-1942), nh ton hc X-Vit,
tc gi ca cc b sch Ton hc vui hay Vt l vui rt quen
31

Tp ch online ca cng ng nhng ngi yu Ton

thuc vi c gi Vit Nam; hay bi Samuel Loyd (1841-1911),


nh ton hc, k th c vua ngi M, c cng tp hp
v sng to hn 5000 bi ton gii tr; v cui cng th khng
th khng nhc n Martin Gardner (1914-2010), nh ton hc
ngi M c cng ng gp c th ni l quan trng nht trong
lch s pht trin ca ton hc gii tr.

Hnh 4.1: Thp H Ni, mt trong nhng bi ton kinh in


ca ton hc gii tr, ln u tin c ng bi nh ton
hc ngi Php Francois-douard-Anatole Lucas (1842-1894),
trong Ton hc gii tr (Rcratin Mathmatiques).

2. Cc bi ton i nn
Ton hc gii tr xut hin rng khp cc nhnh ca ton hc,
v c trong cc ngnh khoa hc khc. Trong chuyn mc m
u ny, chng ti gii thiu vi c gi mt nhm bi ton
kinh in, thng c gi l nhm Bi ton i nn.
Dng thc chung ca cc bi ton i nn nh sau: mt s
ngi s c i mt hoc mt s nn. Cc nn ny c mu
trong mt tp hp cc mu cho trc. C nhn mi ngi khng
bit mu nn ca mnh, nhng c th thy c nn ca cc
ngi khc. Nhim v ca h l phi on c mu nn ca
mnh, v khng c trao i thng tin sau khi nn c i.
Bi ton i khi xut hin di dng tr chi trn truyn hnh
32

Tp ch online ca cng ng nhng ngi yu Ton

vi ngi dn tr v ngi tham gia tr chi; c khi xut hin


di dng bi ton v nhng nh logic; c khi l cai ngc v
t nhn. . . nhng ct li bi ton l tm chin lc cho nhng
ngi ny trc khi c i nn, sao cho khi nhn thy mu
nn ca nhng ngi trong nhm th h s c chin thut
on ng cng nhiu mu nn cng tt.
C rt nhiu c s cho thy rng bi ton c lu truyn
trong dn gian t rt lu, nhng k t nm 1961 nhm bi ton
i nn mi c chnh thc ghi nhn bi Martin Gardner. Bi
ton sau c pht trin vi rt nhiu bin th, vi cc kt
qu v phng php gii rt khc nhau. Mt trong nhng phin
bn kinh in ca bi ton c xut bi Konstantin Knop
trong k thi Olympic ton ton quc Nga ln th 23, nm
1997. Sau , bi ton c kho st chi tit trong lun n tin
s ca Todd Ebert vo nm 1998.
n nm 2001, bi ton c ng li bi cy bt Sara Robinson trong chuyn mc Khoa hc ca thi bo New York s ngy
10 thng 4. n nm 2009, mt ln na mt m rng ca bi
ton c ng li cng thi bo New York, s ngy 23 thng
3. Cho n nm 2011, Lionel Levine lm mi bi ton vi trng
hp v hn nn, thu ht c nhiu phng php gii mi l.
Gn y nht, vo nm 2013, trong mt k thi ton ti Nga, mt
ln na bi ton c lm mi vi mt phin bn tuyt p ca
Konstantin Knop. y cng l phin bn m rng cui cng m
chng ti ghi nhn c tnh n thi im vit bi ny.
Vi nhng pht biu vn kh kh khan, nh mt tr chi trn
truyn hnh, hay thm ch phi l nh th thch gia cai ngc
v t nhn, v sao cc bi ton gii tr v i nn li thu ht s
quan tm ca cc nh ton hc n nh vy? V s hp dn,
tnh sng to ca bn thn bi ton cng nh li gii? Hay v
kt qu ca bi ton dn n nhng ng dng quan trng, c
bit l trong l thuyt m ha? Chng ti xin mi c gi hy
cng tm ra cu tr li bng mt cuc du ngon qua nhng bi
ton i nn ny.

2.1. Bi ton i nn s 1: Hai chng r


y c l l phin bn c nht trong s cc bi ton i nn.
y, chng ti gii thiu mt d bn nh sau: Ngy xa c nh
33

Tp ch online ca cng ng nhng ngi yu Ton

i gia kn r cho hai c con gi. Kn chn mi c hai chng


tun t vn hay ch tt. Ngi cha mt ln na mun th ti
tr ca hai chng r tng lai bn by ra thch .
ng cho mi ngi mt chic nn. Mi ngi khng c nhn
thy nn ca mnh m ch nhn thy nn ca ngi cn li. Sau
cng lc c hai phi vit ra mu nn ca mnh cho ngi
cha xem. Nu t nht c mt ngi on ng, ng s chn c
hai chng r, nu c hai u on sai th phi ra v khng.
Bit l nn c hai mu, trng hoc en.
Hai chng trai tr vn l bn ca nhau. Trc khi thch bt
u, h ngm ngm trao i chin thut v cui cng ci c
hai nng tiu th xinh p.
Theo c gi, hai chng trai ni g vi nhau?

2.2. Bi ton i nn s 2: Bch nin thng th


Bi ton s 2 l mt dng m rng ca bi ton s 1, c mt
d bn nh sau:Vo nm mng thng th trm tui ca nh
vua, ngi mi n mt trm ngi khch, pht cho mi ngi
mt chic nn c mu trng hoc en v by mt tr chi vi
nn. Mi ngi ch thy mu ca 99 ngi khc nhng khng
thy c mu nn ca mnh. Cng lc h phi on mu nn
ca mnh v khng c c bt k trao i g vi nhau. Ai on
trng s c ban bng lc.
Bng lc ca nh vua rt ln nn c mt ngi khch l ma
mnh nhanh chng nm bt thi c. Hn r tai nhng ngi
khch khc rng ch cn lm theo cch ca hn th s ngi
34

Tp ch online ca cng ng nhng ngi yu Ton

ginh c phn thng s l cao nht. V mi k nhn thng


phi chia mt phn tin cho K L .
Cch ca hn l g c c mn li to nht? Bn c hy
cng on th xem.

2.3. Bi ton i nn s 3: Mt n ci ca qu
Trong rng su c mt con qu d nm no cng bt v mi
linh hn sng. N gom ht m linh hn li vi nhau, eo cho
mi linh hn mt chic mt n ci mu sc rc r. Lun c
mi mu, nhng s lng cc mu th thay i theo tng nm.
C nm mi ci mt n c mi mu, c nm li xen k, c
nm li hon ton ging nhau. l nhng chic mt n dnh
cho tr chi ca qu.
Tr chi quy nh k eo mt n chng th nhn thy mu mt
n ca mnh m ch c th nhn thy mu mt n ca nhng
linh hn xung quanh. V th l tt c cng bt u tr phng
on. Cng mt lc cc linh hn phi ni ln mu mt n ca
mnh. Ch cn duy nht mt k on ng th tt c c tha,
bng khng tt c nhng linh hn mi mi phi tr thnh
n l cho qu d.
Mt nm n, c mt nhm linh hn thng minh, trc khi tr
chi bt u, h ni vi nhau. . .
V ri nhng linh hn thng.
Chin thut ca h l g? Bn on c khng?

2.4. Bi ton i nn s 4: Th thch 3 chic nn


y l mt phin bn rt ni ting ca nhm bi ton ny.
y chng ti gii thiu vi c gi phin bn trn thi bo New
York ngy 10 thng 4 nm 2001: C 3 ngi tham gia mt tr
chi, trong mi ngi c i ngu nhin mt nn c mu
hoc xanh dng. H nhn thy mu nn ca 2 bn mnh
nhng khng bit mu ca mnh. Mi ngi cn phi on ra
mu nn ca mnh, hoc chn b qua nu khng on c.
Nu t nht mt ngi on ng mu nn v nhng ngi cn
li khng on sai, h thng tr chi. H s thua nu c ngi
35

Tp ch online ca cng ng nhng ngi yu Ton

on sai hoc c 3 cng chn b qua. H c trao i chin


thut vi nhau trc khi chi nhng trong khi tham gia th
khng c trao i bt c thng tin g. Tm chin thut c xc
sut thng cao nht.

2.5. Bi ton i nn s 5: Bi ca ca 15 g say


Bi ton s 5 l mt trng hp tng qut ca bi ton s 4 vi
mt d bn nh sau: Quanh mt chic hm cp bin, c mi
lm g say ru i nn ngi ci. Mt g va ci va ht v
mu nn ca nhng k k bn. Bi ht nh sau:
Trng v en hay im lng
Ny nhng g say
Hoc ni hoc cm lng
Ngoi bin khi kho bu qu ang ch.
Truyn thuyt ni l mt bi ht v cng l mt cu . Mi
ngi ch thy nn ca 14 ngi khc v khng thy nn ca
mnh v c ba la chn ni ln mu nn ca mnh, trng
hoc en, hoc b qua. Ch cn tt c cc g ni trng mu
nn ca mnh th chic hm cp bin s m ra.
Hy tm chin thut tt nht cho mi lm tn say.

36

Tp ch online ca cng ng nhng ngi yu Ton

2.6. Bi ton i nn s 6: Ngi dn tr chi


Xo quyt
bi ton ny, chng ti mi c gi cng quay li bi ton
i nn s 4. Vi bi ton ny, nu mt ngi chn ngu nhin
mu nn bt k v 2 ngi cn li chn b qua, h s thng vi
xc sut 12 . Tuy nhin, lun n ca Ebert trnh by mt li
gii tt hn, trong nu mt ngi thy 2 bn mnh i nn
khc mu nhau, s chn b qua v nu 2 bn i cng mu
nn, ngi ny s chn mu cn li. Vi chin thut ny, xc
sut thng tr l 34 .
Li gii trn khi ngun cho bi ton i nn s 5: sau nhiu
ln chi i chi li, ngi dn tr lu c hn v thy rng ngi
chi s thua nu c 3 i nn cng mu, do vy ngi dn tr
chn cch i nn khng tht s ngu nhin na. Liu rng
c chin thut no ngi chi vn gi c kh nng chin
thng l 34 trong tnh hung ny?
C 6 bi ton i nn c gii thiu trn u c cng iu
kin l mi ngi chi u thy c mu nn ca tt c nhng
ngi chi khc, do vy, nhm bi ton ny cn hay c pht
biu di dng ngi chi xp thnh vng trn. Bi ton hin
vn cn c pht trin v cc li gii p hin ch xut hin
cc trng hp c bit, v d trng hp bi ton i nn s 5.
Cc trng hp tng qut vi n ngi chi v m mu nn hin
ch dng mc xc nh chn trn ca kh nng chin thng.
Tip theo y l ba bi ton i nn m , ngi chi ch
c thy nn ca mt s ngi khc, thng thng l nhng
ngi ng trc mnh khi xp thnh hng.

2.7. Bi ton i nn s 7: Bi ton 100 ngi


Bi ton ny vi trng hp 2 mu v 3 mu ln u tin c
a ra bi Konstantin Knop k thi Olympic ton ton quc
Nga ln th 23, nm 1997. Pht biu ca bi ton nh sau:
C 100 ngi c xp thnh mt hng, mi ngi c i
mt nn c mu trng hoc en. Mi ngi ch nhn thy mu
nn ca nhng ngi ng trc mnh m khng thy nn ca
mnh v nhng ngi ng sau. Ln lt mi ngi s phi
37

Tp ch online ca cng ng nhng ngi yu Ton

on mu nn ca mnh v h to cho nhng ngi khc nghe.


Ngi ng cui cng (l ngi thy mu nn ca ton b 99
ngi trc) l ngi bt u phi on.
Ngi chi khng c trao i bt k thng tin g vi nhau
ngoi tr lng nghe mu nn t ngi on trc. ng sai
cng ch c bit khi ngi cui cng on xong. Hy tm
chin chut sao cho s ngi on sai l t nht.

Hy gii bi ton vi trng hp 100 ngi chi, 3 mu nn. Liu


c th tng qut ln vi N ngi chi v M < N mu nn?

2.8. Bi ton i nn s 8: Vo cng thin ng


mi thin thn u phi ci hoa!
Tng t vi bi ton i nn s 6, nhng bi ton ny c mt
tch chuyn kh th v vi mi thin thn xp hng vo cng
thin ng. Mi thin thn u ci trn tc mt o hoa trng
hoc v ch nhng thin thn ng sau mi nhn thy mu
hoa trn tc nhng thin thn ng trc. th thch lng
nhn nhn v tnh on kt ca cc thin thn, nh Tri ra lnh
cho h ln lt on mu hoa trn tc ca mnh theo th t
t sau ra trc. Tuy nhin cc thin thn vn c quyn khng
on m chn b qua. Tt c s c vo cng thin n nu
khng c ai on sai v t nht mt thin thn on ng. Trong
qu trnh on mu cc thin thn khng c trao i bt c
thng tin g vi nhau. Nhng thin thn ny qua cng nh
Tri bng phng thc no?

38

Tp ch online ca cng ng nhng ngi yu Ton

2.9. Bi ton i nn s 9: Bi ton 101 mu


16 nm sau khi bi ton i nn vi 100 ngi v 2 mu nn
(bi ton i nn s 7) ra i, Konstantin Knop li lm mi bi
ton vi 101 mu, pht biu nh sau:
C 100 ngi xp thnh hng, mi ngi c i mt nn trong
s 101 nn khc mu nhau. Mi ngi ch thy nn ca nhng
ngi ng trc mnh v khng thy nn ca mnh cng nh
nhng ngi ng sau. Ln lt mi ngi t sau ra trc phi
on mu nn ca mnh v h to cho mi ngi cng nghe.
Mu no h ri s khng c h li na. Ngi chi khng
c trao i thng tin vi nhau. Tm chin thut sao cho kh
nng tt c u on ng l cao nht.

Trong 9 bi ton i nn trc, ngi chi u tham gia vi vai


tr h tr cho nhau. Tip theo, di y chng ti xin gii thiu
mt dng khc ca bi ton, m ngi chi s phi cnh
tranh vi nhau.

2.10. Bi ton i nn s 10: Kho bu nh vua


Bi ton c ghi nhn t rt sm bi Martin Gardner vo nm
1961. Mt d bn ca bi ton c thut li nh sau: Ba ngi
o m c thn cht bt mt dn vo vo mt hm m ti.
Trn u mi ngi c qun mt bng hoc bng en.
Cui ng hm l kho bu ca nh vua.

39

Tp ch online ca cng ng nhng ngi yu Ton

Thn cht cho php ba tn trm m m mt ra. Khi m mt


bn chng ch thy c mu khn ca hai ng bn v khng
thy mu ca mnh. Thn cht ni k no on ng sm nht
mu khn ca mnh s ginh c kho tng. Bng khng s b
git. Lut chi ca thn cht cn quy nh nu c k no thy
khn bt u ca hai tn ng bn c mu en th phi gi tay
ln. Theo bn c tn trm m no c th ly c kho bu ca
nh vua khng?

2.11. Bi ton i nn s 11: Qua i t thn


C 20 t t c nhn mt c hi cng sng st nh sau: 20
ngi ny c xp thnh vng trn, b che mt v mi ngi
c i nn trng hoc en. T t ch c bit c t nht
mt nn en trong s 20 nn c i. Sau khi m mt, mi
ngi thy c mu nn ca 19 ngi cn li. Sau mi pht,
nu c ngi nghim ra c mu nn ca mnh th ngi ny
s c php on. Nu sau 20 pht, nu khng ai on ra,
ton b s b x t. Nu nh trong 20 pht c ngi on sai,
h cng b x t. H ch c t do nu nh trong 20 pht phi
c ngi on, v tt c cc ngi on u phi on ng.
Hy tm chin thut sao cho tt c u sng st.
hai bi ton i nn tip theo, chng ti gii thiu cc trng
hp m rng m s lng hoc ngi chi, hoc s nn
c nng ln v hn (m c).

2.12. Bi ton i nn s 12: V hn ngi chi


C v hn (m c) ngi chi xp thnh mt hng, trong
mi ngi c i mt nn c mu trng hoc en v ngi
ng sau thy c ton b nn ca nhng ngi ng trc.
Ln lt mi ngi t sau ra trc s ni ln mu nn ca
mnh. Hy tm chin thut s ngi on ng l cao nht.
Lu , c li gii cho bi ton ny, c gi cn phi s dng
tin chn.

2.13. Bi ton i nn s 13: V hn nn


Bi ton ny c ra bi Lionel Levine (i hc Cornell) vo
nm 2011 nh sau: Bn ngi cng tham gia mt tr chi on
40

Tp ch online ca cng ng nhng ngi yu Ton

nn nh sau: Ngi dn tr s i v hn cc nn c mu trng


hoc en ln u mi ngi vi xc sut nn trng v en l
nh nhau v bng 21 . Cc nn ca mi ngi c nh s ln
lt 1, 2, . . . Mi ngi chi ch thy c ton b nn ca 3
ngi khc nhng nn ca mnh th h khng thy.
Mi ngi s c pht mt t giy v h c php ghi vo
mt con s, ng vi ch s ca nn ca h m h on l mu
en. Sau khi nhn tr li, ngi dn tr s kim tra s c
ghi trn giy ca mi ngi.

Nu c 4 ngi cng on ng (tc l 4 ngi u ghi c con


s ng vi nn mu en ca mnh), h thng tr chi, ngc li,
ch cn mt ngi on khng ng, h thua. Bn ngi c
tho lun trc chin thut trc khi chi v khng c bt k
trao i no sau . H cng khng bit c thi im m
nhng ngi khc a giy cho ngi dn tr. Hy tm chin
thut xc sut thng l cao nht.
V d: h u ghi s 2015 vo cc mnh giy. Khi , c hi chin
1
v xc sut nn th 2015 ca mi ngi l 12 .
thng s l 16
Tng qut ha cho N ngi liu cch gii c khc?
Nhng bi ton i nn khc: Trong nhng bi trc, tt c
u lin quan n vic on mu ca nn, trong nhm bi cui
cng ny chng ti gii thiu vi c gi mt vi dng khc ca
bi ton i nn.

2.14. Bi ton i nn s 14: Bi ton 3 chic nn


Ba ngi chi, mi ngi c i mi chic nn, trn mi chic
nn c ghi mt s nguyn dng. Mi ngi ch thy 2 s ca 2
ngi chi khc m khng bit s ca mnh. H c cho bit
41

Tp ch online ca cng ng nhng ngi yu Ton

l 1 trong 3 s l tng ca 2 s cn li. Ln lt tng ngi,


hoc on ra con s ca mnh, hoc chn b qua. Sau y l
on on s ca h:
Ngi 1: b qua.
Ngi 2: b qua.
Ngi 3: b qua.
Ngi 1: b qua.
Ngi 2: b qua.
Ngi 3: b qua.
Ngi 1: b qua.
Ngi 2: b qua.
Ngi 3: s ca ti l 60.
Hi rng con s trn 2 nn cn li c th l bao nhiu?

2.15. Bi ton i nn s 15: Xp hng


C 10 ngi tham gia tr chi nh sau: mi ngi s c i
mt chic nn, trn c mt con s nguyn dng. Ngi chi
khng c cho bit gii hn ca cc s, h ch bit 10 s ny
phn bit vi nhau. Mi ngi khng bit s ca mnh nhng
thy c s ca 9 ngi cn li. Sau khi quan st xong cc s
ca nhng ngi khc, mi ngi s phi chn nn ca mnh
l mu trng hoc en. Vic chn mu ny cng khng c
cho cc ngi chi khc bit.
Sau khi chn xong mu nn, nhng ngi chi s c vxp
thnh mt hng, theo th t tng dn ca gi tr con s trn
nn. Nu nh h c th xp thnh mt hng trng/en xen k
nhau, h chin thng tr chi, ngc li h tht bi. Hy tm
chin thut xc sut chin thng l cao nht.
42

Tp ch online ca cng ng nhng ngi yu Ton

3. Li kt
Bi ton i nn th 15 trn cng kt thc chuyn mc
k ny. Chng ti hi vng rng sau cuc du ngon xuyn sut
hn na th k pht trin ca nhng bi ton i nn, tp ch
ca chng ti c th gii thiu c vi c gi v p v s
hp dn ca nhm bi ton ny. Bn thn mi bi ton i nn
thng l nhng th thch ton hc hng tun, nn trnh
lm mt i cm xc ca nhng c gi mong mun th sc,
chng ti ch chn ng gi hoc p n vn tt ca mi bi
v s trnh by li gii chi tit vo nhng s tip theo.
Chng ti tin rng vi phn li gii, c gi s c tip tc
chuyn hnh trnh k th cng nhng ng dng thc t t
nhm bi ton gii tr ny. Chng ti cng rt hoan nghnh
mi ng gp ca qu v c gi v li gii cng nh nhng
phin bn khc ca nhm bi ton i nn.
thm phn th v cho mt s bi ton, chng ti t li
phn ln tnh tit bi nhng vn gi nguyn bn cht ton
hc. tin tra cu v tham kho, c gi c th truy tm li
ngun gc ca tng bi thng qua nhng ti liu sau:
1) A Dozen Hat Problems: Cho cc bi 1, 2, 3, 4, 5, 8, 10, 11.
2) Colored Hats and Logic Puzzles: Cho cc bi 1, 2, 3, 4, 5,
6, 8, 10 v 11.
3) A Line of Sages: Cho cc bi 7 v 9.
4) An introduction to infinite hat problems: Cho bi 12, 13.
5) Problem of the week 1179: Cho bi 13.
6) The Three-Hat Problem: Cho bi 14.
7) Another black and white hats puzzle: Cho bi 15.

43

Tp ch online ca cng ng nhng ngi yu Ton

4. Gi li gii
Bi 1. Ngi 1 chn mu ngc li vi mu nn ca ngi 2;
Ngi 2 chn mu ca ngi 1.
Bi 2. Ghp thnh tng cp v p dng bi 1.
Bi 3. Gn s cc mu t 0 n 9 v tng linh hn t 0 n 9.
Khi linh hn th k s on mu sao cho mu v tng 9
mu khc bng k (mod 10).
Bi 4. p n pht biu t bi 6.
Bi 5. Da trn t tng t bi 3, lin kt vi m Hamming.
Bi 6. C 8 trng hp cho 3 ngi vi 2 mu nn, khi chin
thut hin ti s tht bi 2 trng hp cc nn cng mu - hoc Xanh-Xanh-Xanh v thnh cng 6 trng hp
cn li. Liu c chin thut lun tht bi cp trng hp
Xanh-- v -Xanh-Xanh nhng thnh cng 6 trng
hp cn li? Vi 2 cp cn li (Xanh--Xanh, -Xanh-) v
(Xanh-Xanh-, --Xanh), liu c chin thut tng ng?
Khi , nu ta ngu nhin chn chin thut xut pht th vic
chn gi tr ban u khng ngu nhin s gp tht bi. Xc
sut thnh cng c bo ton l 75%.
Bi 7. Cho trng hp 2 mu: ngi cui s ni mu en nu
s nn en anh ta thy l s l v ni trng nu s nn en l
chn. Nhng ngi khc, cn c vo s on c.
Vi trng hp M < N mu, ngi cui s chn mu l tng
(mod N) nhng mu anh ta quan st c.
Bi 8. Vi la chn b qua, kh nng thng ln n
khi tt c nn u cng mu trng (hoc en).

1023
.
1024

Ch sai

Bi 9. S dng t hp thay v modulo. Chng ti s phn tch


b ba bi 7, 8 v 9 ny trong nhng s tip theo.
Bi 10. Nu tt c u nn trng, s khng c tn no gi tay,
v bn chng s suy ra c nn mnh mu trng.
Nu ch c mt nn en, c hai tn i nn trng s cng lc
bit nn mnh mu trng v tn cn li khng gi tay.
44

Tp ch online ca cng ng nhng ngi yu Ton

Nu c hai nn en, c ba tn s cng gi tay. Lc ny, nhng


tn i nn en s cng bit mnh i nn en, v chng s suy
lun: nu nn mnh mu trng, th tn i nn en kia khng
th gi tay.
Tnh hung khng th on c ngay lp tc l khi c 3 u
i nn en. Nhng sau s chn ch ca c 3, c 3 s cng
on c tt c u i nn en.
Bi 11. Nu mt ngi thy k nn trng, anh ta s on nn
mnh mu en pht th (20 k). Sau khi c ngi on
th khng ai on na.
Bi 12, 13. Chng ti s phn tch chi tit vo cc s tip theo.
Bi 14. Cc p n c th c l:
[25, 35, 60], [35, 25, 60], [42, 18, 60], [18, 42, 60]
[10, 50, 60], [50, 10, 60], [44, 16, 60], [16, 44, 60].
Bi 15. Tn ti chin thut lun lun thng. Hy th vi nhng
trng hp nh.

45

Tp ch online ca cng ng nhng ngi yu Ton

46

Tp ch online ca cng ng nhng ngi yu Ton

V BI HNH HC THI VMO 2015


Trn Quang Hng (Trng THPT Chuyn KHTN, HQG H Ni)

Tm tt
Bi vit s xoay quanh khai thc bi hnh hc thi quc gia
Vit Nam ngy u tin.

K thi hc sinh gii quc gia Vit Nam nm 2015 c bi ton


hnh hc nh sau:
Bi ton 1. Cho ng trn (O) v hai im B, C c nh trn
(O) vi BC khng l ng knh. Mt im A thay i trn (O)
sao cho tam gic ABC nhn. Gi E, F ln lt l chn ng cao
k t B, C ca tam gic ABC. (I) l ng trn thay i i qua cc
im E, F v c tm l I.
q
cot B
DB
.
a) Gi s (I) tip xc BC ti D. Chng minh rng DC = cot
C

b) Gi s (I) ct cnh BC ti M, N. Gi H l trc tm tam gic


ABC v P, Q l giao im ca (I) vi ng trn ngoi tip
tam gic HBC. ng trn (K) i qua P, Q tip xc (O) ti
T vi T , A cng pha BC. Chng minh rng phn gic trong
ca gc MT N lun i qua im c nh.

Nhn xt. y l bi ton v tr s 4 l bi c nh gi l


kh. Hai ca bi ton khng lin quan nhiu ti nhau, chng
ti s gii v phn tch tng . Vi b) ca bi ton thc cht
cc yu t v trc tm v chn ng cao l khng cn thit.
Chng ti xin a ra mt bi ton tng qut hn v thc ra v
mt cu hnh s n gin hn ng thi pht biu li cho thy
ngha thc ca n.
Bi ton 2. Cho BC l dy cung ca ng trn (O). ng trn
(K) bt k qua B, C. P, Q l hai im thuc (K) v trong (O).
ng trn (L) qua P, Q tip xc trong (O) ti A sao cho A, K
khc pha BC. ng trn (S) qua P, Q ct BC ti M, N. Chng
minh rng BAM = CAN.
47

Tp ch online ca cng ng nhng ngi yu Ton

Li gii. Theo tnh cht tm ng phng d thy tip tuyn


chung ti A ca (O) v (L), PQ v BC ng quy ti T .
A

L
O
Q

P
S
T

C
N

T d c T A2 = T P.T Q = T M.T N. T d suy ra ng trn


ngoi tip tam gic AMN cng tip xc (O). T y, ta d dng
suy ra BAM = CAN (iu phi chng minh).
Nhn xt. Bi ton l p dng trc tip ca cc tnh cht v
phng tch v trc ng phng. Bi ton c th thay th iu
kin tip xc thnh ct nhau nh sau
Bi ton 3. Cho XY l dy cung ca ng trn (O). ng trn
(K) bt k qua X, Y. ng trn (L) ct (O) ti Z, T v ct (K) ti
P, Q. ng trn (S) qua P, Q ct BC ti M, N. Chng minh rng
XZM = YT N.

48

Tp ch online ca cng ng nhng ngi yu Ton

Li gii. Ta cng d thy XY, ZT , PQ ng quy ti R. Suy ra


RM RN = RP RQ = RZ RT .

Z
L
O

P
S
R

Kt qu ny chng t t gic ZT MN ni tip. T , ta c


XZM = RZM RZZ = T NM T YM = YT N.
Ta c iu phi chng minh.
Nhn xt. Bi ton m rng tip theo ny xem ra cn n gin
hn c trng hp tip xc. Thc ra iu chng ti mun ni
nhng bi ton sau ny l khi tng qut bi ton th cng
l mt cch hay cho chng ta tm ra li gii n gin hn l cc
trng hp ring. Khi nhn qua ci nhn tng qut b bt cc
d kin khng cn thit bi ton tr nn khng kh na. Ta c
th vit li bi ton theo cch khc mang tnh i xng hn:

49

Tp ch online ca cng ng nhng ngi yu Ton

Bi ton 4. Cho ng trn (K) v (L) ct nhau ti A, B. Mt


ng trn bt k ct (K) ti M, N ct (L) ti P, Q v ct AB ti
S, T . Mt ng trn qua M, N ct (L) ti E, F. Mt ng trn qua
P, Q ct (K) ti G, H.
a) Chng minh rng E, F, G, H cng thuc mt ng trn.
b) Chng minh rng SEA = T FB v SGA = T HB.
c) Gi s G, E, M, P, S, A cng pha vi KL. Chng minh rng
HBF + GAE = HT F + GSE.

G
E
M

A
K

H
F

B
Q

N
T

Li gii n gin ch p dng bi tp trn. Chng ti nhn thy


bi ton ny ngha nm nhiu cu a). Tuy rng theo nh
gi th a) l dng g im nhng thc ra n sau n c
nhiu yu t th v. Chng ta thy l vic pht biu kt lun
bng mt biu thc lng gic khng p. Ta hon ton c th
KB
cotB
= KC
vi AK ng cao t
thay th biu thc lng gic cotC
DB2
KB
A, nh vy ta cn chng minh DC2 = KC . n y ta c th
xut bi tng qut hn nh sau:
Bi ton 5. Cho tam gic ABC. Mt ng trn (K) qua B, C ct
CA, AB ti E, F. BE ct CF ti H. AH ct BC ti D. Mt ng trn
qua E, F tip xc on BC ti T . Chng minh rng
T B2
DB
=
.
2
TC
DC
50

Tp ch online ca cng ng nhng ngi yu Ton

Li gii 1. Gi ng trn ngoi tip tam gic DEF ct CA, AB


ti M, N khc E, F.
A

E
M

N
H
F
B
T

Ta c EMN = EFN = ECB, suy ra MN k BC. T :


T B2
BF AN
BF AE
DB
BF BN
=

=
.
=
2
TC
CE CM
CE AM
CE AF
DC
Ta c iu phi chng minh.
A

H
K

51

Tp ch online ca cng ng nhng ngi yu Ton

Li gii 2. Gi EF ct BC ti G. Gi S i xng T qua G. T tnh


cht phng tch ta thy GS2 = GT 2 = GE GF = GB GC, suy ra
hng (ST , BC) = 1 cng d c hng (GD, BC) = 1. T ta c
CB CG = CT CS v BC BG = BT BS, suy ra
DB
BG
BG BC
BT .BS
BT BS
T B2
=
=
=
=

= 2.
TC
DC
CG
CG CB
CT .CS
CT CS
T d c iu phi chng minh.
Nhn xt. Li gii th nht mang nhiu tnh hnh hc v s
cp. Tuy vy khi bi ton nhn qua cch ca hng iu ha kh
d hiu, tt c hon ton l bin i t s trn ng thng.
C th gii m khng dng im S tuy vy vic dng thm hng
iu ha (BC, ST ) s dng cc h thc bit trn hng iu
ha lm ta gii quyt bi ton nhanh hn. Bi ton hon ton
cha dng li vic tng qut. Nu ta k bi ton gc,
khi dng ra cc tip im tng t trn CA, AB s dn n mt
bi ton ng quy rt th v nh sau:
Bi ton 6. Cho tam gic ABC nhn, ng cao AD, BE, CF.
ng trn qua E, F tip xc on BC ti X. Tng t c Y, Z.
Chng minh rng AX, BY, CZ ng quy.
tng tng t khi tip im ngoi cnh:
Bi ton 7. Cho tam gic ABC nhn, ng cao AD, BE, CF.
ng trn qua E, F tip xc BC ti X ngoi on BC. Tng
t c Y, Z. Chng minh rng X, Y, Z thng hng.
Kt hp c hai ca bi ton trn ta c mt bi ton rt th v
nh sau (tham kho [2]):
Bi ton 8. Cho tam gic ABC nhn, ng cao AD, BE, CF ng
quy ti H. Hai ng trn ng trn qua E, F tip xc BC ti
X1 , X2 sao cho X1 nm gia B, C. Tng t c Y1 , Y2 , Z1 , Z2 . Chng
minh rng:
a) AX1 , BY1 , CZ1 ng quy ti P.
b) X2 , Y2 , Z2 thng hng trn ng thng d.
c) PH v d vung gc.

52

Tp ch online ca cng ng nhng ngi yu Ton

Li gii. Cc a) v b) c cc bi trn, ta tp trung chng


minh c) l th v nht ca bi ton ny. Theo tnh cht tip
tuyn d thy EF ct BC ti G l trung im X1 X2 . Gi M l trung
im BC ch E, F, D, M cng nm trn ng trn Euler ca
tam gic ABC nn ta c GX21 = GX22 = GE GF = GD GM. T
hng (X1 X2 , DM) = 1. Ta cng c hng (BC, DG) = 1. Suy ra
DH DA = DB DC = DG DM = DX1 DX2 .
A

E
Z1
F

P
H
K

X2

X1
B

Z2

T H l trc tm tam gic AX1 X2 nn X2 H vung gc AX1 ti


K. Gi Q l hnh chiu ca X2 ln PH, t kt qu trn, ta c
HP HQ = HK HX2 = HD HA = HB HE = HC HF = k.
T y, bng cch chng minh tng t, ta c hnh chiu ca
Y2 , Z2 ln PH cng l Q. Nh vy PH vung gc d ti Q.
53

Tp ch online ca cng ng nhng ngi yu Ton

Ta hon ton c th c bi ton o nh sau:


Bi ton 9. Cho tam gic ABC vi cc im D, E, F thuc cnh
BC, CA, AB sao cho AD, BE, CF ng quy. Gi s c ng trn
DB
B2
(K) qua E, F tip xc on BC ti T sao cho TT C
2 = DC . Chng minh
rng bn im B, C, E, F thuc mt ng trn.
Li gii 1. T gi thit AD, BE, CF ng quy v ch t gic
MENF ni tip, ta suy ra
T B2
DB
EA FB
EA AM FB AN
FB AN
=
=

.
2
TC
DC
EC FA
FA AN EC AM
EC AM

(1)

E
M

N
H
F
B
T

Mt khc, ta li c

T (1) v (2) suy ra

BN
CM

T B2
BF BN
=
.
T C2
CE CM
AN
= AM
. Vy MN k BC. T :

(2)

ACB = AMN = EFA,


suy ra t gic BCEF ni tip. Ta c iu phi chng minh.
Li gii 2. Gi (L) l ng trn qua B, C v tip xc ng
trn ngoi tip tam gic T EF ti S. Tip tuyn chung ti S ct BC
ti G. Ta d thy ST l phn gic BSC, suy ra
GB
T B2
DB
SB2
=
=
=
.
2
2
GC
SC
TC
DC
54

Tp ch online ca cng ng nhng ngi yu Ton

E
S

H
L
K

Nh th, ta c (BC, DG) = 1 m AD, BE, CF ng quy nn suy


ra EF i qua G. T GE GF = GS2 = GB GC suy ra E, F, B, C
thuc mt ng trn. Ta c iu phi chng minh.
T bi ton tng qut trn, ta li c th tng qut bi ton ng
quy hn na nh sau:
Bi ton 10. Cho tam gic ABC cc im K, L, N ln lt thuc
trung trc BC, CA, AB sao cho AK, BL, CN ng quy. ng trn
(K) qua B, C ct on CA, AB ti Ab , Ac . ng trn qua Ab , Ac
tip xc cnh BC ti Aa . Tng t c Bb , Cc . Chng minh rng
AAa , BBb , CCc ng quy.
Nu thay cc yu t tip xc thnh ct nhau, ta cng c mt s
bi ton tng t, cc bn hy lm nh cc bi luyn tp:
Bi ton 11. Cho tam gic ABC c E, F ln lt thuc on
CA, AB. BE ct CF ti H. AH ct BC ti D. S l mt im trn
on BC. ng trn ngoi tip tam gic SEF ct BC ti T khc S.
DB
BSBT
Chng minh rng CSCT
= DC
khi v ch khi B, C, E, F cng thuc
mt ng trn.

55

Tp ch online ca cng ng nhng ngi yu Ton

Bi ton 12. Cho tam gic ABC nhn c ng cao AD, BE, CF
ng quy ti H. Cc im X, Y, Z thuc on BC, CA, AB sao cho
AX, BY, CZ ng quy. ng trn ngoi tip tam gic XEF ct BC
ti U khc X. Tng t c cc im V, W. Chng minh rng:
a) AU, BV, CW ng quy ti P.
b) YZ, ZX, XY ln lt ct BC, CA, AB theo ba im thuc mt
ng thng vung gc vi PH.

Ti liu tham kho


[1] thi VMO nm 2015.
[2] Cc bi vit ca Buratinogigle:
artofproblemsolving.com/Forum/viewtopic.php?f=47&t=620287

56

Tp ch online ca cng ng nhng ngi yu Ton

V BI BT NG THC
TRONG THI VMO 2015
V Quc B Cn (H Ni)

Tm tt
Trong k thi chn hc sinh gii Quc gia mn Ton nm
2015, thi ngy th nht c bi ton bt ng thc sau:
Bi ton 1. Cho a, b, c > 0. Chng minh rng
3(a2 + b2 + c2 ) > P > (a + b + c)2 ,


vi P = (a+b+c) ab+ bc+ ca +(ab)2 +(bc)2 +(ca)2 .

Bi vit ny chng ti trnh by cc kin ca mnh v bi


ton cng nh nu ra cc hng tip cn khc nhau i
n li gii. Bn cnh cc phn tch bnh lun, chng ti
cng s xut mt s bi ton vi tng tng t cho
tng hng tip cn bn c c th t rn luyn thm.
cui bi vit, chng ti s gii thiu ngun gc, pht
biu v gii bi ton tng qut ca bi VMO ni trn.

1. Nhn xt chung
Vi kin ch quan ca mnh, chng ti cho rng y l mt
bi ton kh hp l tng xng vi v tr ca n trong thi.
Trong thi gian 180 pht, cc th sinh phi lm 4 bi ton vi
cc th loi: Gii tch, i s, T hp v Hnh hc.
S lng cu hi kh nhiu nhng thi gian lm bi li hn
ch, th nn cc bi ton u tin khng th ra qu kh v nh
th s to p lc cho th sinh.
Bi ton ny mc trung bnh, khng d cng khng kh.
Hnh thc pht biu cng gn gng, n gin ch khng cng
57

Tp ch online ca cng ng nhng ngi yu Ton

knh phc tp so vi VMO nm 2014. Ngoi ra, bi ton ny


cng c kh nhiu hng tip cn ch khng mo mc phc
tp nh thi nm ngoi. Chnh v th, vic chn n lm bi
s 2 l kh ph hp.
Tuy nhin, iu khng c ngha l bi ton ny thc s tt.
tng ca n khng mi nu khng mun ni l kh quen
thuc vi cc em hc sinh. V vy, do quen dng nn nhiu em
trng t c th nhn vo ngay v gii m khng cn phi ngh
suy nhiu. R rng iu ny s khin cho vic nh gi cht
lng cng nh kt qu s khng c khch quan. S tht
tuyt nu thi l nhng bi ton vi tng mi m nhng
li nh nhng, tinh t v khng mo mc. Mong rng cc
VMO sp ti s p ng c iu ny.

2. Cc hng tip cn cho bi ton


V tri ca bt ng thc kh n gin. Dng pht biu ca
n vi tng cc bnh phng gi cho ta ngh n ng nht
thc Lagrange mt hng ng thc quen thuc c dng
chng minh bt ng thc Cauchy-Schwarz:
X
X
 X
2
n
n
n
X
2
2
ai
bi
ai bi =
(ai bj aj bi )2 > 0.
i=1

i=1

i=1

16i<j6n

C th hn, ta c ng thc sau:


3(a2 + b2 + c2 ) (a + b + c)2 = (a b)2 + (b c)2 + (c a)2 .
Do , bt ng thc v tri c th c vit di dng:


(a + b + c)2 > (a + b + c) ab + bc + ca .

n y th c l bn no cng s ngh n vic s dng bt


ng
thucx2 + y2 + z2 > xy + yz + zx (p dng cho

thc quen
x = a, y = b v z = c) hon tt php chng minh.
y, ta s dnh s quan tm nhiu hn cho bt ng thc
v phi. Nhn xt ban u cho thy y l mt bt ng thc
tng i cht v du bng xy ra ti hai trng hp a = b = c
v a = b, c = 0 (cng cc hon v tng ng). Do , ta cn phi
rt cn trng trong cc nh gi ca mnh.
58

Tp ch online ca cng ng nhng ngi yu Ton

Ngoi ra, ta cng thy rng ch kh ca bi ton chnh l cc


cn thc. Nu ta c th ph c du cn a bt ng thc
v dng n gin hn th chc chn bi ton cng s tr nn
sng sa hn. n y, c hai tng chnh nh sau:
1. t n ph kh cn: y l mt hng i kh t nhin
v cc cn thc y cng n gin, cc biu thc di
du cn ch
c dngbc mt.
Do , ch cn mt ln t
n ph x = a, y = b, z = c l ta c th kh c ht
cc cn thc v a v xt mt bt ng thc thun nht
bc 4 i vi x, y, z. Bc ca bt ng thc mi cng khng
qu cao nn y l hng i hon ton kh thi.
2. S dng nh gi kh cn: y l tng thng thy
khi x l cc bi ton c cn. Vn c t ra y l
ta phi la chn nh gi cht sao cho cc iu kin
du bng phi c m bo.
Cc hng tip cn c trnh by di y hu ht u s
dng hai tng trn lm t tng ch o:

2.1. Hng 1: Khai trin trc tip


y c l l
hng it nhin
nht cho bi ton ny. Ta ch
vic t x = a, y = b, z = c ri nhn tung ht ra. Khi ,
bt ng thc cn chng minh c th c vit li di dng:
X
X
X
X
x4 + xyz
x+
xy(x2 + y2 ) > 4
x2 y2 .
(1)
n y, nu bn no c tm hiu s ngh ngay n bt ng
thc Schur bc 4:
x2 (x y)(x z) + y2 (y z)(y x) + z2 (z x)(z y) > 0.
Dng khai trin ca n chnh l:
X
X
X
x4 + xyz
x>
xy(x2 + y2 ).

(2)

S tng ng gia hai bt ng thc (1) v (2) gi cho ta ngh


n vic dng (2) nh gi cho (1). Ngoi ra, (2) cng c du
bng ti x = y = z v x = y, z = 0 (cng cc hon v) tng ng
vi trng hp ng thc ca (1). Do , y s l mt nh gi
59

Tp ch online ca cng ng nhng ngi yu Ton

kh n v ta c th yn tm v an ton ca n. Tht vy, sau


khi nh gi, ta ch cn xt bt ng thc:
X
X
X
X
2
xy(x2 + y2 ) > 4
x2 y2
xy(x2 + y2 ) > 2
x2 y2
v n ch l mt h qu trc tip ca bt ng thc AM-GM:
X
X
X
xy(x2 + y2 ) >
(xy 2xy) = 2
x2 y2 .
Li bnh. t n ph l mt trong nhng k nng c bn cn
c trong bt ng thc. Nhiu bi ton c hnh thc cng knh
phc tp, tuy nhin sau nhng bc t n ph n gin, ta
c th a bi ton tr v dng mi m nhiu tng (m
trong cng c th l gc ca bi ton) s c phi by ra.
C nhiu kiu t n ph, trong c ba kiu sau rt thng
dng: t n ph lm n gin hnh thc bi ton, t n
ph thun nht ha hoc i xng ha, v t n ph lng
gic da vo du hiu t iu kin gi thit.
Di y l mt s v d:
Bi ton 2. Cho x, y, z l cc s thc dng. Chng minh rng
Xp
p
(x + y)(x + z) > x + y + z + 3(xy + yz + zx).

Li gii. t a = y + z, b = z + x v c = x + y. Khi , ta d
thy a, b, c l ba cnh ca mt tam gic v:
x=

b2 + c2 a2
,
2

y=

c2 + a2 b2
,
2

z=

a2 + b2 c2
.
2

Thay vo, ta vit c bt ng thc di dng:


r 
X
X
X
X 
2
2
2
2
ab
a > 3 2
ab
a4 ,

hay
X
X
p
2
ab
a2 > 3(a + b + c)(b + c a)(c + a b)(a + b c).

n y, ta li t a = n+p, b = p+m v c = m+n vi m, n, p > 0.


Bt ng thc c vit li thnh:
p
mn + np + pm > 3mnp(m + n + p).
Mt kt qu qu quen thuc.

60

Tp ch online ca cng ng nhng ngi yu Ton

Bi ton 3 (IMO, 2001). Cho a, b, c > 0. Chng minh rng


a
b
c

+ 2
+ 2
> 1.
2
a + 8bc
b + 8ca
c + 8ab
c
Li gii. t x = a2a+8bc , y = b2b+8ca v z = c2 +8ab
. Khi ,
bng cc bin i n gin, ta d thy 0 < x, y, z < 1 v:

8bc
1 x2
=
,
a2
x2

8ca
1 y2
=
,
b2
y2

8ab
1 z2
=
.
c2
z2

T suy ra
512x2 y2 z2 = (1 x2 )(1 y2 )(1 z2 ).
Theo yu cu ca bi ton, ta cn chng minh x + y + z > 1. Nu
iu ny khng ng, tc x + y + z < 1, th ta c
1 x2 > (x + y + z)2 x2 = (y + z)(2x + y + z).
nh gi tng t cho cc biu thc cn li, ta thu c
hY
ihY
i
512x2 y2 z2 >
(y + z)
(2x + y + z) .
Bng cch s dng bt ng thc quen thuc:
(m + n)(n + p)(p + m) > 8mnp,

m, n, p > 0

ln lt cho cc b (x, y, z) v (x + y, y + z, z + x), ta c


Y
Y
(2x + y + z) > 8
(x + y) > 64xyz.
T suy ra
hY
ihY
i
(y + z)
(2x + y + z) > 8xyz 64xyz = 512x2 y2 z2 .
Mu thun nhn c cho ta kt qu bi ton.

Bi ton 4. Cho a, b, c l cc s thc tha mn iu kin abc = 1.


Chng minh rng
1
1
1
+
+
> 1.
2
2
1+a+a
1+b+b
1 + c + c2

61

Tp ch online ca cng ng nhng ngi yu Ton

Li gii. Do abc = 1 nn ta c th chng minh c tn ti cc


, b = yzx2 v c = xy
(chng hn, ta
s thc x, y, z tha mn a = yz
x2
z2
1
1
c th chn x =
, z = 13c ). Khi , bt ng thc cn
3 a, y =
3
b
chng minh c th c vit li thnh:
y4
z4
x4
+
+
> 1.
x4 + x2 yz + y2 z2 y4 + y2 zx + z2 x2 z4 + z2 xy + x2 y2
n y, bng cch s dng bt ng thc Cauchy-Schwarz:
(x2 + y2 + z2 )2
VT > 4
,
(x + y4 + z4 ) + xyz(x + y + z) + (x2 y2 + y2 z2 + z2 x2 )
ta a c bi ton v xt mt kt qu qu thuc:
x2 y2 + y2 z2 + z2 x2 > xyz(x + y + z).
Bi ton 5. Cho x, y, z l cc s thc dng tha mn iu kin
xy + yz + zx + 2xyz = 1. Gi s z = max{x, y, z}, chng minh rng
1 1 1
(2z 1)2
+ + 4(x + y + z) >
.
x y z
z(2z + 1)
Li gii. Gi thit xy+yz +zx+2xyz = 1 c th c vit li di
1
1
1
dng x+1
+ y+1
+ z+1
= 2. T , ta d dng chng minh c
tn ti cc s dng a, b, c sao cho:
x=

a
,
b+c

y=

b
,
c+a

z=

c
.
a+b

Ngoi ra, do z = max{x, y, z} nn ta c c = max{a, b, c}. Bt ng


thc cn chng minh c vit li thnh:
 a
b+c c+a a+b
b
c  (2c a b)2
+
+
4
+
+
>
.
a
b
c
b+c c+a a+b
c(2c + a + b)
Do

a
b

a
c

4a
b+c

a(bc)2
bc(b+c)

nn bt ng thc tng ng vi:

b(c a)2
c(a b)2
(2c a b)2
a(c b)2
+
+
>
.
bc(b + c) ca(c + a) ab(a + b)
c(2c + a + b)
V ta s chng minh bt ng thc mnh hn l:
a(c b)2
b(c a)2
(2c a b)2
+
>
,
bc(b + c) ca(c + a)
c(2c + a + b)
62

Tp ch online ca cng ng nhng ngi yu Ton

hay
(2c a b)2
a(c b)2 b(c a)2
+
>
.
b(b + c)
a(c + a)
2c + a + b
S dng bt ng thc Cauchy-Schwarz dng cng mu, ta c
q
hp
i2
a
b
(c b) + a (c a)
b
VT >
.
2c + a + b
T , bi ton c a v chng minh
r
r
a
b
(c b) +
(c a) > 2c a b,
b
a
hay

r

a
+
b

b
2 c + a + b 2 ab > 0.
a

Bt ng thc ny hin nhin ng theo AM-GM.


Bi ton 6 (Vit Nam TST, 2001). Cho x, y, z l cc s thc
dng tha mn 2x + 4y + 7z = 2xyz. Tm gi tr nh nht ca:
P = x + y + z.

7a, y = 27 b, z = 2 7 7 c, ta c a + b + c = abc v:

7
P=
(14a + 7b + 4c).
14

Do a, b, c > 0 v a + b + c = abc nn tn ti A, B, C 0, 2 tha
mn A + B + C = v a = tanA, b = tanB, c = tanC, suy ra

7
P=
(14 tanA + 7 tanB + 4 tanC).
14

Li gii. t x =

Biu thc P c dng tng hm. iu ny gi cho ta nh n bt


ng thc tip tuyn nh sau: Nu hm s f(x) kh vi bc hai
v li trn khong (a, b) th vi mi x, y (a, b), ta u c
f(x) > f(y) + f 0 (y) (x y).
Do hm s f(x) = tanx kh vi bc hai v li trn 0,

bt ng thc trn, vi mi x, y 0, 2 , ta c

nn theo

tanx > tany + (tany) 0 (x y) = tan y + (tan2 y + 1)(x y).


63

Tp ch online ca cng ng nhng ngi yu Ton

Trong bt ng
 thc trn, ln
 lt thay cps (x, y) bi cc cp
A, arctan 37 , B, arctan 57 v C, arctan 7 , ta thu c
3
16 
3 
tanA > +
A arctan ,
7
7
7
5
32 
5 
tanB > +
B arctan ,
7
7
7



tanC > 7 + 8 C arctan 7 .

T suy ra (ch rng arctan 37 + arctan 57 + arctan 7 = ):




X


7
3
5
P>
15 7 + 32
A arctan arctan arctan 7
14
7
7
h
i

7
15
=
15 7 + 32(A + B + C ) = .
14
2
ng thc xy ra khi v ch khi x = 3, y = 52 , z = 2.
Nhn xt. Bt ng thc tip tuyn l mt trong nhng kt qu
quan trng ca hm li. N l mu cht xy dng nn bt
ng thc Karamata, mt cng c rt mnh x l cc bt
ng thc dng tng hm. Bn c c th tm c thm v hai
kt qu th v ny trong bi vit chuyn ca chng ti Ti
liu Chuyn Ton, Gii tch 12 (Nh xut bn Gio Dc, 2011).

Cc s arctan 37 , arctan 57 , arctan 7 c s dng trn khng


phi l nhng s ngu nhin m c. V yu cu bi ton l
tm min nn ta cn phi nh gi P ln hn hoc bng mt
hng s no . Do , khi s dng bt ng thc tip tuyn
nh gi, ta cn chn cc hng s thch hp sao cho h s
ca A, B, C phi bng nhau c th tn dng c gi thit
A + B + C = v bin i v b thnh hng s.
Nh vy, cc s c s dng trong li gii trn thc cht chnh
l nghim thu c t h phng trnh:

0 < x, y, z < , x + y + z =
2
14(tan2 x + 1) = 7(tan2 y + 1) = 4(tan2 z + 1)
64

Tp ch online ca cng ng nhng ngi yu Ton

Bi ton 7 (Kin Giang, 2014).


P Cho 2014 s thc x1 , x2 , . . . , x2014

thuc on [1, 1] tha mn 2014
i=1 xi > 1. Chng minh rng tn
ti mt s nguyn dng k sao cho:
k

2014
X
X


xi
xi 6 1.

i=1

i=k+1

Li gii. Ta thay 2014 bi mt s n > 2 tng qut hn. Vi mi


P
1 6 k 6 n, ta t ak = ki=1 xi , khi t gi thit suy ra
|ak ak1 | 6 1 < |an |, k = 1, . . . , n ( y a0 = 0).
Theo yu cu bi, ta cn chng minh tn ti k N sao cho
|an 2ak | 6 1.

(3)

Nu c s 1 6 k 6 n 1 sao cho (an 2ak )(an 2ak+1 ) 6 0 th




|an 2ak |+|an 2ak+1 | = (an 2ak )(an 2ak+1 ) = 2|ak+1 ak | 6 2.
T suy ra s nh nht trong hai s |an 2ak |, |an 2ak+1 | s
c gi tr khng vt qu 1 v bi ton c chng minh.

Xt trng hp (an 2ak )(an 2ak+1 ) > 0 vi mi 1 6 k 6 n 1.


Ta chng minh (3) bng phn chng. Gi s khng tn ti s k
ni trn, khi ta c
|an 2ak | > 1,

1 6 k 6 n 1.

(4)

Nu an > 2an1 th do (an 2an1 )(an 2an ) > 0 nn ta suy ra


an < 0. Mt khc, do |an | > 1 nn ta c an < 1.
Do an 2ak v an 2ak+1 c cng du vi mi 1 6 k 6 n 1 nn
t bt ng thc an > 2an1 , ta cng suy ra an > 2a1 v do
1 + 2a1 < an < 1 (theo (4)), tc a1 < 1. Kt qu ny a n
|a1 | > 1, mu thun vi gi thit bi ton.
L lun tng t, trng hp an < 2an1 cng khng th xy ra.
Nhng mu thun thu c cho ta kt qu ca bi ton.

65

Tp ch online ca cng ng nhng ngi yu Ton

Sau y l mt s bi ton khc dnh cho bn c t luyn:


Bi ton 8 (VMO, 2005). Xt cc s thc x, y tha mn iu kin:
p

x 3 x + 1 = 3 y + 2 y.

Hy tm gi tr ln nht v gi tr nh nht ca biu thc P = x+y.

Bi ton 9 (Olympic N sinh Trung Quc, 2004). Cho a, b, c l


cc s thc dng. Tm gi tr nh nht ca biu thc:
P=

4b
8c
a + 3c
+

.
a + 2b + c a + b + 2c a + b + 3c

Bi ton 10 (Th Nh K, 2006). Cho x, y, z l cc s thc dng


tha mn xy + yz + zx = 1. Chng minh rng

2

27
(x + y)(y + z)(z + x) >
x + y + y + z + z + x > 6 3.
4
Bi ton 11. Cho a, b, c l cc s thc dng. Chng minh rng
a
b
c
1
+
+
6 .
4a + 4b + c 4b + 4c + a 4c + 4a + b
3
Bi ton 12 (IMO, 1983). Cho a, b, c l di ba cnh ca mt
tam gic. Chng minh rng
a2 b(a b) + b2 c(b c) + c2 a(c a) > 0.
Bi ton 13 (Chn i tuyn Moldova, 2006). Cho a, b, c l
di ba cnh ca mt tam gic. Gi p l na chu vi ca tam gic
, chng minh bt ng thc sau:
r
r
r
(p b)(p c)
(p c)(p a)
(p a)(p b)
+b
+c
> p.
a
bc
ca
ab
Bi ton 14 (IMO, 2008). Cho x, y, z 6= 1 l cc s thc tha mn
iu kin xyz = 1. Chng minh rng
 x 2  y 2  z 2
+
+
> 1.
x1
y1
z1
66

Tp ch online ca cng ng nhng ngi yu Ton

Bi ton 15. Cho a, b, c l cc s dng tha mn abc = 1.


Chng minh rng
1
1
2
1
+
+
+
> 1.
(1 + a)2 (1 + b)2 (1 + c)2 (1 + a)(1 + b)(1 + c)
Bi ton 16. Cho a, b, c l cc s thc dng tha mn abc = 1.
Chng minh rng
1
1
1
a)
+
+
6 1.
5a + 4
5c + 4
5b + 4
b)

1
1
1

+
+
6 1.
1 + 3a + 1 1 + 3b + 1 1 + 3c + 1

Bi ton 17. Cho x1 , x2 , . . . , xn l cc s thc dng tha mn:


1
1
1
+
+ +
= 1.
1 + x1 1 + x2
1 + xn
Chng minh rng
 1

1
1 
x1 + x2 + + xn > (n 1) + + +
.
x1
x2
xn
Bi ton 18. Cho x1 , x2 , . . . , xn l cc s thc dng tha mn:
1
1
1
n
+
+ +
= .
1 + x1 1 + x2
1 + xn
2
Tm gi tr nh nht ca biu thc:
P=

n X
n
X
i=1 j=1

1
.
xi + xj

Bi ton 19 (Nga, 2004). Cho s t nhin n > 3. Xt cc s thc


dng x1 , x2 , . . . , xn c tch bng 1. Chng minh rng
1
1
1
+
+ +
> 1.
1 + x1 + x1 x2 1 + x2 + x2 x3
1 + xn + xn x1

67

Tp ch online ca cng ng nhng ngi yu Ton

Bi ton 20 (Trung Quc, 2004). Cho a, b, c, d l cc s thc


dng tha mn abcd = 1. Chng minh rng
1
1
1
1
+
+
+
> 1.
2
2
2
(1 + a)
(1 + b)
(1 + c)
(1 + d)2
Bi ton 21. Cho a, b, c > 0 tha mn abc = 1. Chng minh rng
a
b
c
+
+
> 1.
a+b+1 b+c+1 c+a+1
Bi ton 22. Cho x, y, z l cc s thc dng tha mn iu kin
x2 + y2 + z2 + 2xyz = 1.
Chng minh cc bt ng thc sau:
1
a) xyz 6 .
8
3
b) x + y + z 6 .
2
c) xy + yz + zx 6

3
6 x2 + y2 + z2 .
4

d) xy + yz + zx 6

1
+ 2xyz.
2

Bi ton 23. Cho x, y, z l cc s thc dng tha mn iu kin


xyz = x + y + z + 2.
Chng minh cc bt ng thc sau:
a) xy + yz + zx > 2(x + y + z).
b)

3
x+ y+ z6
xyz.
2

Bi ton 24 (APMO, 2004). Cho a, b, c l cc s thc dng.


Chng minh rng
(a2 + 2)(b2 + 2)(c2 + 2) > 9(ab + bc + ca).

68

Tp ch online ca cng ng nhng ngi yu Ton

Bi ton 25 (Trung Quc, 1996). Cho n N , x0 = 0, xi > 0 vi


mi i = 1, 2, . . . , n v x1 + x2 + + xn = 1. Chng minh rng
16

n
X
i=1

xi

< .
2
1 + x0 + x1 + + xi1 xi + + xn

Bi ton 26 (Trung
Cho a1 , a2 , . . . , a2n l cc s
P2n1 Quc, 2003).
2
thc tha mn i=1 (ai ai+1 ) = 1. Tm gi tr ln nht ca:
P = (an+1 + an+2 + + a2n ) (a1 + a2 + + an ).
Bi ton 27 (IMO, 2006). Tm s M nh nht bt ng thc:


ab(a2 b2 ) + bc(b2 c2 ) + ca(c2 a2 ) 6 M(a2 + b2 + c2 )2

ng vi mi s thc a, b, c.

Trc khi kt thc phn ny, xin c ni thm mt cht v bt


ng thc Schur bc 4:
X
X
X
x4 + xyz
x>
xy(x2 + y2 ).
y l mt bt ng thc rt cht (cht hn c bt ng thc
Schur bc 3), n ng vi mi b s thc x, y, z ch khng
i hi cc bin phi khng m. Tht vy, bng cch s dng
phng php SOS c trnh by phn sau, ta vit c
X

x2 (x y)(x z) =

1X
(x y)2 (x + y z)2 > 0.
2

Chnh v iu ny, bt ng thc c th c dng x l rt


hiu qu cho cc bi ton i xng ba bin vi du bng bin,
c bit l cc bt ng thc thun nht bc 4.
Di y l mt s v d:
Bi ton 28. Cho a, b, c l cc s thc dng. Chng minh rng
(b + c)2 (c + a)2 (a + b)2
+ 2
+ 2
> 6.
a2 + bc
b + ca
c + ab

69

Tp ch online ca cng ng nhng ngi yu Ton

Li gii. S dng bt ng thc Cauchy-Schwarz, ta c



2
(b + c)2 + (c + a)2 + (a + b)2
.
VT >
(b + c)2 (a2 + bc) + (c + a)2 (b2 + ca) + (a + b)2 (c2 + ab)
T , bi ton c a v chng minh
X
2(a2 + b2 + c2 + ab + bc + ca)2 > 3
(b + c)2 (a2 + bc).
Sau khi khai trin v rt gn, bt ng thc trn c dng:
X
X
X
X
2
a4 +
ab(a2 + b2 ) + 2abc
a>6
a2 b2 ,
hay
hX
i
X
X
X
X
2
a4 + abc
a
ab(a2 + b2 ) + 3
ab(a2 + b2 ) > 6
a2 b 2 .

n y, bng cch s dng bt ng thc Schur bc 4 v mt


s nh gi n gin, ta d c iu phi chng minh.
Bi ton 29. Cho a, b, c l cc s thc khng m tha mn khng
c hai s no ng thi bng 0. Chng minh rng
2a2

1
1
8
1
+ 2
+ 2
>
.
+ bc 2b + ca 2c + ab
(a + b + c)2

Li gii. S dng bt ng thc Cauchy-Schwarz, ta c


4(a + b + c)2
.
VT >
(b + c)2 (2a2 + bc) + (c + a)2 (2b2 + ca) + (a + b)2 (2c2 + ab)
T , bi ton c a v chng minh


(a+b+c)4 > 2 (b+c)2 (2a2 +bc)+(c+a)2 (2b2 +ca)+(a+b)2 (2c2 +ab) .

Sau khi khai trin v rt gn, bt ng thc trn c dng:


X
X
X
X
a4 + 2
ab(a2 + b2 ) + 4abc
a>6
a2 b2 .

n y, bng cch s dng bt ng thc Schur bc 4, ta a


c v xt mt bt ng thc mi l:
X
X
X
3
ab(a2 + b2 ) + 3abc
a>6
a2 b 2 .
Mt kt qu kh hin nhin.
70

Tp ch online ca cng ng nhng ngi yu Ton

Nhn xt. Cch nh gi bng bt ng thc Cauchy-Schwarz


c mt k thut ring ca n ch khng phi ta ngu nhin
chn thm cc i lng (b + c)2 , (c + a)2 , (a + b)2 nhn vo.
Ch rng bt ng thc cho c du bng ti trng hp
a = b, c = 0 (v cc hon
gi theo li thng
Pv) 1nn nu nh
9
P
> (2a2 +bc) ), ta s khng m
thng (chng hn nh
2a2 +bc
bo c du bng. Th nn, ta cn phi c s iu chnh.
thu c li gii nh trn, chng ti s dng CauchySchwarz vi km thm tham s ph:
X
hX
i
1
2
2
(2a + bc)(ma + nb + nc) > (m + 2n)2 (a + b + c)2
2
2a + bc
ri chn m, n sao cho h t l khi xt du bng cng tha mn
ti trng hp a = b, c = 0 (v cc hon v). K thut ny ging
nh mt php cn bng h s cho cc bt ng thc m ta
on trc c du bng.
Ging nh trn, chng ti cng xin a ra mt s bi tp
bn c t rn luyn thm k nng ny:
Bi ton 30. Cho a, b, c l cc s thc dng. Chng minh rng
X
21
1
.
>
2
2
2
2
2
a + ab + b
2(a + b + c ) + 5(ab + bc + ca)
Bi ton 31. Cho a, b, c l cc s thc dng. Chng minh rng
X
1
9
.
>
4a2 ab + 4b2
7(a2 + b2 + c2 )
Bi ton 32. Cho a, b, c l cc s thc khng m tha mn khng
c hai s no ng thi bng 0. Chng minh rng
1
1
1
1
+
+
>
.
2
2
2
22a + 5bc 22b + 5ca 22c + 5ab
(a + b + c)2
Bi ton 33. Cho a, b, c l cc s thc khng m tha mn khng
c hai s no ng thi bng 0. Chng minh rng
2a2 bc
2b2 ca
2c2 ab
+
+
> 3.
b2 bc + c2 c2 ca + a2 a2 ab + b2
Bi ton 34. Cho a, b, c l cc s thc khng m tha mn khng
c hai s no ng thi bng 0. Chng minh rng
a2 bc
b2 ca
c2 ab
+
+
> 0.
2b2 3bc + 2c2 2c2 3ca + 2a2 2a2 3ab + 2b2
71

Tp ch online ca cng ng nhng ngi yu Ton

2.2. Hng 2: Phng php SOS


y l hng i t nhin th hai sau phng php khai trin.
Trc ht, ta cng s t n ph x, y, z nh hng 1 trn
kh cn tin cho vic quan st. Ta a bi ton v chng minh:
X
(x2 + y2 + z2 )(xy + yz + zx) +
(x2 y2 )2 > (x2 + y2 + z2 )2 .
Trong bt ng thc trn, c hai s hng cng cha nhn t
x2 + y2 + z2 . Mt cch t nhin, ta ngh n vic ghp hai s hng
vi nhau. Lc ny, bt ng thc c vit li thnh:
X
(x2 y2 )2 > (x2 + y2 + z2 )(x2 + y2 + z2 xy yz zx).
S xut hin ca tng bnh phng
phn tch qu quen thuc:
X

x2

xy =

(x2 y2 )2 bn v tri v

1X
(x y)2
2

gi cho ta ngh ngay n vic dng phng php phn tch


bnh phng SOS x l bi ton. C th, ta vit c bt
ng thc cn chng minh di dng:
Sx (y z)2 + Sy (z x)2 + Sz (x y)2 > 0,
trong Sx = f(x, y, z) = 2(y+z)2 (x2 +y2 +z2 ) = y2 +z2 +4yzx2 ,
cn Sy = f(y, z, x), Sz = f(z, x, y) c nh ngha tng t.
n y, ta ch vic s dng cc tiu chun ca phng php
l c. Gi s x > y > z, khi ta c
Sy = z2 + x2 + 4zx y2 > 0,

Sz = x2 + y2 + 4xy z2 > 0

v
Sx + Sy = 2z2 + 4zx + 4yz > 0.
Do x z > y z > 0 nn (z x)2 > (y z)2 . T suy ra
Sx (y z)2 + Sy (z x)2 + Sz (x y)2 > (Sx + Sy )(y z)2 > 0.
Li bnh. Mt iu cn ch l khi s dng phng php SOS,
cc bn cn phi chng minh li cc tiu chun ca n. Nhiu
bn cu th ch ghi gn l Sx + Sy > 0, Sy + Sz > 0 ri suy ra iu
phi chng minh. Nh th l cha c.
72

Tp ch online ca cng ng nhng ngi yu Ton

Ngoi cch s dng cc tiu chun SOS nh trn, ta cng c


cch bin i m khng phi s dng tiu chun no da trn
ng nht thc n gin:
X
(x y)2 (x z)(y z) = 0.
(3)
C th thy im mu cht gy kh khn trong vic x l tng:
X
(x y)2 (x2 + y2 + 4xy z2 ) > 0
chnh l phn s m mi s hng, chng hn nh z2 trong
s hng (x y)2 (x2 + y2 + 4xy z2 ). Nu ta em cng vi tng
P
(x y)2 (x z)(y z) vi mt s lng thch hp vo s lm tng
s lng z2 ln s hng ny v rt c th s thu c mt i
lng khng m. C th, ta hy vng s c s k sao cho:
x2 + y2 + 4xy z2 + k(x z)(y z) > 0
(x + y)2 + (2 + k)xy kz(x + y) + (k 1)z2 > 0.
Quan st mt cht, cho th thy ngay nu chn k = 2 th ta s
vit c biu thc (x + y)2 kz(x + y) + (k 1)z2 di dng bnh
phng. T , ta thu c mt li gii ngn gn th v sau:
Bt ng thc cn chng minh tng ng vi
X
X
(x y)2 (x2 + y2 + 4xy z2 ) + 2
(x y)2 (x z)(y z) > 0,
hay

X


(x y)2 (x + y z)2 + 4xy > 0.

ng nht thc (3) gip chng ta x l c bi ton theo


mt li SOS rt th v a n mt bt ng thc hin nhin.
y cng l mt kinh nghim ca chng ti tch ly c khi
tm hiu v phng php SOS. Tt nhin, ng nht (3) ch hiu
qu cc bt ng thc i xng bc 4. Vi cc bt ng thc
bc cao, chng ta cn mt ng nht thc tng qut hn
tng cng tnh hiu qu. Chng ta c mt kt qu th v sau
(bn c c th t chng minh): Cho f(x, y, z) l mt a thc i
xng vi hai bin x, y. Khi , ta c th phn tch:
X

(xy)2 (xz)(yz)f(x, y, z) = (xy)2 (yz)2 (zx)2 g(x, y, z)
trong g(x, y, z) l mt a thc i xng vi ba bin x, y, z.
73

Tp ch online ca cng ng nhng ngi yu Ton

Nh vo ng nht thc trn m chng ti x l thnh cng


rt nhiu bt ng thc bng phng php SOS rt n gin
ch khng cn phi dng tiu chun phc tp no.
Di y l mt s v d:
Bi ton 35. Cho a, b, c > 0 c tng bng 3. Chng minh rng
1

1 1
9
+ + 3 > 8(a2 + b2 + c2 3).
a b c
Li gii. Ta c cc bin i quen thuc sau:

1
1
X (a b)2
1 1
1 1
+ + 3 = (a + b + c)
+ +
9=
,
3
a b c
a b c
ab
1
1X
a2 + b2 + c2 3 = a2 + b2 + c2 (a + b + c)2 =
(a b)2 .
3
3
Do , bt ng thc c th c vit li di dng:
 9

X
(a b)2
8 > 0,
ab
hay

c(a b)2 (9 8ab) > 0.

n y, ta c rng:
9 8ab = (a + b + c)2 8ab
= (a + b)2 8ab + 2c(a + b) + c2
= (a + b)2 6c(a + b) + 9c2 8(a c)(b c)
= (a + b 3c)2 8(a c)(b c).
T , ta c th vit li bt ng thc nh sau:
X
X
c(a b)2 (a + b 3c)2 8
c(a b)2 (a c)(b c) > 0.
P
P
V
c(a b)2 (a c)(b c) = 0 v
c(a b)2 (a + b 3c)2 > 0 nn
bt ng thc cui hin nhin ng.
Bi ton 36 (Iran, 1996). Cho a, b, c l cc s thc khng m
tha mn khng c hai s no ng thi bng 0. Chng minh rng


1
1
9
1
(ab + bc + ca)
+
+
> .
2
2
2
(a + b)
(b + c)
(c + a)
4
74

Tp ch online ca cng ng nhng ngi yu Ton

Li gii. Bt ng thc cn chng minh tng ng vi



X  4(ab + bc + ca)
3 > 0.
(a + b)2
Ta c
VT =

X 4bc + 4ca 3a2 3b2 2ab

(a + b)2
X (c a)(3a + b) (b c)(a + 3b)
=
(a + b)2
X (c a)(3a + b) X (b c)(a + 3b)
=

(a + b)2
(a + b)2


X
3b + c
3a + c
=
(a b)

(b + c)2 (a + c)2
X (a b)2 (3ab + ac + bc c2 )
.
=
(a + c)2 (b + c)2

T , bt ng thc c a v chng minh


X
(a2 b2 )2 (3ab + ac + bc c2 ) > 0.
Do 3ab + ac + bc c2 = 4ab (a c)(b c) nn bt ng thc trn
c th c vit li thnh:
X
X
4
ab(a2 b2 )2
(a2 b2 )2 (a c)(b c) > 0.
Mt khc, ta li c
X
(a2 b2 )2 (a c)(b c) = (a b)2 (b c)2 (c a)2
nn bt ng thc trn tng ng vi
X
4
ab(a2 b2 )2 > (a b)2 (b c)2 (c a)2 .
n y, s dng bt ng thc Cauchy-Schwarz, ta c

2
4 ab(a2 b2 ) + bc(b2 c2 ) + ca(c2 a2 )
VT >
ab + bc + ca
2
4(a + b + c) (a b)2 (b c)2 (c a)2
=
ab + bc + ca
2
> 12(a b) (b c)2 (c a)2 ,
t d dng suy ra kt qu cn chng minh.
75

Tp ch online ca cng ng nhng ngi yu Ton

Bi ton 37. Cho a, b, c l cc s thc. Chng minh rng


a2

bc
ca
3
ab
+ 2
+ 2
6 .
2
2
2
2
2
2
+ b + 3c
b + c + 3a
c + a + 3b
5

Li gii. Bt ng thc cn chng minh tng ng vi


X
1


5ab
> 0.
a2 + b2 + 3c2

n y, bng cch tch SOS tng t nh trn, ta vit c


bt ng thc di dng:
X
(a b)2 (9a2 + 9b2 + 5c2 2ab 10ac 10bc)(a2 + b2 + 3c2 ) > 0.
Ta c
9a2 + 9b2 + 5c2 2ab 10c(a + b) = (3a + 3b 5c)2 20(a c)(b c)
v
X
(a b)2 (a c)(b c)(a2 + b2 + 3c2 ) = 2(a b)2 (b c)2 (c a)2 ,
do bt ng thc trn tng ng vi
X
(a b)2 (3a + 3b 5c)2 (a2 + b2 + 3c2 ) > 40(a b)2 (b c)2 (c a)2 .
S dng bt ng thc Cauchy-Schwarz, ta c
1X
(a b)2 (3a + 3b 5c)2 (a + b 3c)2
5
i2
1 hX
(a b)(3a + 3b 5c)(a + b 3c)
>
15
1024
=
(a b)2 (b c)2 (c a)2 ,
15

VT >

t d dng suy ra kt qu cn chng minh.


Ch rng mi mt bt ng thc trong ba v d nu trn u
c nhng hai trng hp du bng. iu ny chng t chng
l nhng bt ng thc kh cht. Th nhng, vi phng php
tch SOS mi ny, cng vic x l li tr nn kh nh nhng.

76

Tp ch online ca cng ng nhng ngi yu Ton

Di y, chng ti xin c nu thm mt s bi ton p dng


khc bn c th sc v so snh vi phng php c.
Bi ton 38 (Vit Nam TST, 2006). Cho a, b, c l di ba cnh
ca mt tam gic. Chng minh rng
 a
1
1 1
b
c 
+ +
>6
+
+
.
(a + b + c)
a b c
b+c c+a a+b

Bi ton 39. Cho a, b, c l cc s thc khng m tha mn khng


c hai s no ng thi bng 0. Chng minh rng
1
1
1
6
+ 2
+ 2
> 2
.
2
2
2
2a + bc 2b + ca 2c + ab
a + b + c + ab + bc + ca
Bi ton 40. Cho a, b, c l cc s thc dng. Chng minh rng
b
c
3(a2 + b2 + c2 ) 1
a
+
+
>
+ .
b+c c+a a+b
(a + b + c)2
2
Bi ton 41 (Lm cht Vit Nam TST 1996). Cho a, b, c l cc
s thc. Chng minh bt ng thc sau:

4
(a + b)4 + (b + c)4 + (c + a)4 > a4 + b4 + c4 + (a + b + c)4 .
7
Bi ton 42 (Tng qut chn Olympic Iran 2009). Cho cc s
thc a, b, c c tng bng 3. Chng minh rng vi mi k > 58 , ta c

1
1
1
3
+
+
6
.
a2 + b2 + k b2 + c2 + k c2 + a2 + k
2+k
(Trng hp k = 2 km thm iu kin a, b, c > 0 chnh l bi bt
ng thc trong chn i tuyn Iran nm 2009).
Bi ton 43 (Tng qut Olympic Ton Ba Lan 1996). Cho
a, b, c l cc s thc c tng bng 1. Chng minh rng vi mi s
thc k tha mn 9k2 26k + 1 6 0, ta u c
a
b
c
9
+ 2
+ 2
6
.
2
a +k b +k c +k
1 + 9k
(Trng hp k = 1 km thm iu kin a, b, c > 34 chnh l bi
bt ng thc trong thi Olympic Ton Ba Lan nm 1996).
Bi ton 44. Cho p > 0 v cc s thc a, b, c. t:
X p(3 p)a2 + 2(1 p)bc 3(1 + p)(2 p)

F(a, b, c) =
.
pa2 + b2 + c2
2+p
Chng minh rng (p 1) F(a, b, c) > 0.
77

Tp ch online ca cng ng nhng ngi yu Ton

2.3. Hng 3: nh gi kh cn
Mt hng i khc thay cho t n ph l tm cch nh gi
ph
C th, ta s tm cc nh gi thch hp cho
thc.
cn

ab, bc, ca vi chiu > ph du cn. Thng th vi cc


dng cn tch nh th ny, cch ph cn thng dng l s
dng bt ng thc AM-GM. Tuy nhin, bi ton ny, n li
cho nh gi vi chiu ngc li:

a+b
ab 6
,
2

b+c
bc 6
,
2

c+a
ca 6
2

khng phi chiu ta cn. C cch no iu chnh khng nh?


Mt tng th v y l s dng nghch o. Nh bit,
vi bt ng thc dng th nghch
o ca n s o chiu.
ri nh gi:
Do , ta c th ngh n vic vit ab thnh ab
ab

2ab
ab >
.
a+b

(4)

Nh vy l s ph c cn thc vi chiu ta mun. Tuy nhin,


mt iu cn lu y l s 0 khng c nghch o. Th nn
nu mt trong cc cn thc c mt s bng 0 th ta khng th
dng cch ny c. Do , cn phi xt trng hp loi tr
tnh hung ngoi mun ny.
Nu trong a, b, c c mt s bng 0, chng hn c = 0, th bt ng
thc cn chng minh s tr thnh:

(a + b) ab + (a b)2 + a2 + b2 > (a + b)2 .

Cng vic y l kh n gin v ta c (a + b) ab > 2ab.


Tip theo, ta xt trng hp a, b, c > 0. Lc ny, ta c th s
dng c (4). Bi ton c a v chng minh
X X ab  X
X 2
+
(a b)2 >
a .
2
a
a+b
Sau khi thu gn, n c dng:
X X ab 
X
X
2
a
>4
ab
a2 .
a+b
78

Tp ch online ca cng ng nhng ngi yu Ton

V a + b + c c th tch ra cc i lng aP
+ b, b + c, c + a lin
ab
nn ta c th
quan n mu ca cc s hng ca tng
a+b
x l rt gn v tri theo cch sau:
VT = 2
=2

X ab(a + b + c)
X

a+b
ab + 2abc

=2

1
.
a+b


ab 1 +

c 
a+b

Khi , bt ng thc c th vit li thnh:


 1
1
1 
2abc
+
+
> 2(ab + bc + ca) a2 b2 c2 .
a+b b+c c+a
n y th tng t nhin l s dng bt ng thc CauchySchwarz dng cng mu lm gim s lng cc phn thc:
1
1
1
9
+
+
>
a+b b+c c+a
2(a + b + c)
v a bi ton v xt mt bt ng thc mi:
9abc
> 2(ab + bc + ca) a2 b2 c2 .
a+b+c
Tuy nhin, y chnh l bt ng thc Schur bc ba.
Li bnh. y, chng ti mun ch vi cc bn v cch tch
cc tch ab, bc, ca, abc c s dng trong li gii trn.
Nh ta bit, nhng bt ng thc m trong cc trng hp
du bng ca chng c trng hp khng ti tm th thng
kh nh gi hn cc bt ng thc bnh thng. Nguyn nhn
l cc b hon v. Mt bt ng thc i xng (hoc hon v)
nu c du bng ti b (A, B, C) th cng s t c du bng
ti cc hon v ca n l (B, C, A) v (C, A, B). Do , nh
gi thnh cng th ta phi tm c mt nh gi sao cho n
m bo c c ba trng hp. R rng rt kh!
i vi cc bi ton c du bng ti bin th cch tch trn
cho ta mt k thut x l c bit hiu qu. Tht vy, gi s
ta cn chng minh bt ng thc f(a, b, c) > 0 vi du bng l
a = kb (k 6= 0), c = 0 (v cc hon v) chng hn ( y ch xin
ly v d mt trng hp c th phn tch, cn nhiu trng
79

Tp ch online ca cng ng nhng ngi yu Ton

hp khc cng c th x l tng t). Khi , nu vit c bt


ng thc trn di dng:
ab g(a, b, c) + bc g(b, c, a) + ca g(c, a, b) > 0
th ta ch cn quan tm nh gi biu thc i din g(a, b, c)
theo du bng a = kb, c = 0 l m khng cn ch nhiu
n cc hon v ca b ny. Nu nh gi thnh cng th sau
khi nhn thm ab vo hai v, ta s thu c mt nh gi
cho s hng ab g(a, b, c) vi du bng xy ra ti ab = 0 v
a = kb, c = 0. Hin nhin nh gi ny s m bo c c ba
trng hp hon v ca a = kb, c = 0.
T y, ta thy rng cc iu kin s cng thun li hn nu ta
tch ra c s hng c dng abc h(a, b, c). Lc ny, h(a, b, c)
c th c nh gi kh l v t, bi l ti trng hp bin
th tch abc bng 0 mt ri, th nn khi nhn vo th kiu g
cng m bo c du bng bin. Sau y l mt s v d m
chng ti p dng thnh cng tng ny x l:
Bi ton 45 (VMO, 1996). Cho a, b, c l cc s thc dng tha
mn iu kin ab + bc + ca + abc = 4. Chng minh rng
a + b + c > ab + bc + ca.
Li gii. iu kin gi thit c th c vit li thnh:
1
1
1
+
+
= 1.
a+2 b+2 c+2
T , ta chng minh c tn ti cc s thc dng x, y, z
2y
2x
2z
sao cho a = y+z
, b = z+x
v c = x+y
. Khi , bt ng thc cn
chng minh c th c vit li thnh:
X
x
y
z
xy
+
+
>2
.
y+z z+x x+y
(y + z)(z + x)
S dng bt ng thc AM-GM, ta c
i X xy
X h
X xy
1
1
VP 6
xy
+
=
+
(y + z)2 (z + x)2
(y + z)2
(z + x)2
X xy
X zx
X xy + zx
=
+
=
= VT .
(y + z)2
(y + z)2
(y + z)2
Bi ton c chng minh xong.
80

Tp ch online ca cng ng nhng ngi yu Ton

Bi ton 46. Cho a, b, c l cc s khng m. Chng minh rng


X
X 2

X
a a2 + 2bc > 2 3
a +2 31
ab.
Li gii. D thy bt ng thc hin nhin ng nu trong
a, b, c c hai s cng bng 0, do ta ch cn xt trng hp
(a + b)(b + c)(c + a) > 0. Khi , bt ng thc cn chng minh
c th c vit li di dng:
X
X 


 X
a a2 + 2bc a >
31 2
ab
a2
X 
X

 X
1

>
3

1
2
ab

a2 .
2abc
2
a + 2bc + a

n y, bng cch s dng bt ng thc Cauchy-Schwarz:


X
1
9

> P 2
2
a + 2bc + a
a + 2bc + a + b + c
9
>p P
2
3 (a + 2bc) + a + b + c


9 31
=
2(a + b + c)
ta a c bi ton v chng minh:
9abc
> 2(ab + bc + ca) a2 b2 c2 .
a+b+c
y chnh l bt ng thc Schur bc ba quen thuc.

Bi ton 47. Cho x, y, z l cc s thc dng. Chng minh rng


(x + y + z)2 (x2 + y2 )(y2 + z2 )(z2 + x2 ) > 8(x2 y2 + y2 z2 + z2 x2 )2 .
Li gii. t a = x2 , b = y2 v c = z2 . Khi , s dng bt ng
thc AM-GM, ta c

(x + y + z)2 = a + b + c + 2 ab + 2 bc + 2 ca
4ab
4bc
4ca
>a+b+c+
+
+
.
a+b b+c c+a
Do , ta ch cn chng minh:
X
(a+b+c)(a+b)(b+c)(c+a)+4
ab(a+c)(b+c) > 8(ab+bc+ca)2 .
81

Tp ch online ca cng ng nhng ngi yu Ton

Sau khi khai trin v rt gn, bt ng thc trn c dng:


ab(a2 + b2 ) + bc(b2 + c2 ) + ca(c2 + a2 ) > 2(a2 b2 + b2 c2 + c2 a2 ),
hay
ab(a b)2 + bc(b c)2 + ca(c a)2 > 0.
Bi ton 48. Cho a, b, c l cc s thc dng. Chng minh rng
a
b
c
1
+
+
6 .
4a + 4b + c 4b + 4c + a 4c + 4a + b
3
Li gii. Nhn hai v bt ng thc cho 4(a + b + c) vi ch :
3ca
4a(a + b + c)
=a+
,
4a + 4b + c
4a + 4b + c
ta bin i c n v dng:
ca
ab
bc
a+b+c
+
+
6
.
4a + 4b + c 4b + 4c + a 4c + 4a + b
9
n y, s dng bt ng thc Cauchy-Schwarz, ta c
1
1
1 2
1 
=
6
+
.
4a + 4b + c
2(2a + b) + (2b + c)
9 2a + b 2b + c
T suy ra
2 X ca
1 X ca
+
9
2a + b 9
2b + c
2 X ab
1 X ca
=
+
9
2b + c 9
2b + c
1 X 2ab + ca
= VP.
=
9
2b + c

VT 6

Bi ton c chng minh xong.


Bi ton 49 (IMO, 1999). Cho s t nhin n > 2. Tm hng s C
nh nht sao cho bt ng thc:
X
xi xj (x2i + x2j ) 6 C(x1 + x2 + + xn )4
16i<j6n

ng vi mi x1 , . . . , xn > 0. Vi hng s tm c, hy xc nh
xem ng thc xy ra khi no?
82

Tp ch online ca cng ng nhng ngi yu Ton

P
Li gii. t A = 16i<j6n xi xj (x2i + x2j ). Khi , bng cch s
dng bt ng thc AM-GM, ta c
A6

X
16i<j6n

  X

X
2
2
2
xk =
xi xj xi + xj +
k6=i, k6=j

16i<j6n

xi xj

X
n

x2i

i=1

!2
P
Pn

X

n
X
1
1 2 16i<j6n xi xj + i=1 x2i
2
=
xi 6
xi xj
2
2 16i<j6n
2
2
i=1

2 2
Pn
 n 4
x
i=1 i
1
1 X

=
xi .
=
2
2
8 i=1

Du bng xy ra khi v ch khi xi xj (x2i +x2j ) = xi xj (x21 +x22 + +x2n )


P
vi mi i < j v x21 + x22 + + x2n = 2 16i<j6n xi xj . c c
iu ny th trong cc s xi phi c n 2 s bng 0, hai s cn
li bng nhau. V c th tm c b s xi du bng c th
xy ra nn ta d dng suy ra Cmin = 81 .
Sau y l mt s bi ton khc bn c t rn luyn:
Bi ton 50. Cho a, b, c l cc s khng m. Chng minh rng

a a2 + 3bc + b b2 + 3ca + c c2 + 3ab > 2(ab + bc + ca).


Bi ton 51 (M, 2001). Cho a, b, c l cc s thc khng m tha
mn iu kin a2 + b2 + c2 + abc = 4. Chng minh rng
0 6 ab + bc + ca abc 6 2.
Bi ton 52. Cho a, b, c l cc s thc khng m tha mn khng
c hai s no ng thi bng 0. Chng minh rng
a2 + b2 b2 + c2 c2 + a2
3(a2 + b2 + c2 )
+
+
6
.
a+b
b+c
c+a
a+b+c
Bi ton 53. Cho a, b, c l cc s thc khng m tha mn iu
kin ab + bc + ca = 1. Chng minh rng
a
b
c
3abc
+
+
> a2 + b 2 + c 2 + p
.
3
b+c c+a a+b
(a + b)(b + c)(c + a)
83

Tp ch online ca cng ng nhng ngi yu Ton

Bi ton 54. Cho n N . Xt cc s thc khng m a, b, c thay


i tha mn a + b + c = 1. Tm gi tr ln nht ca biu thc:
P=

(bc)n
(ca)n
(ab)n
+
+
.
1 ab 1 bc 1 ca

Bi ton 55. Cho a, b, c l cc s thc khng m tha mn iu


kin a2 + b2 + c2 = 1. Chng minh rng
b+c
c+a
a+b
+
+
6 3(a + b + c).
1 ab 1 bc 1 ca
Bi ton 56. Cho a, b, c l cc s thc khng m tha mn iu
kin a + b + c = 3. Chng minh rng
h a(b + c) b(c + a) c(a + b) i
5
+
+
> 4(ab + bc + ca) + 3abc.
a+1
b+1
c+1
Bi ton 57. Cho a, b, c l cc s thc dng. Chng minh rng
a4
b4
c4
a+b+c
.
+
+
>
3
3
3
3
3
3
a +b
b +c
c +a
2
Bi ton 58. Cho a, b, c l cc s thc khng m tha mn iu
kin a + b + c = 1. Chng minh rng

3
a b2 + 4c2 + b c2 + 4a2 + c a2 + 4b2 6 .
4

2.4. Hng 4: S dng hm li


Chc hn bn c yu Ton u bit n tnh cht th v sau
ca hm li: Nu hm s f(x) lin tc v li trn on [a, b] th
gi tr ln nht ca n s t c mt trong hai im x = a hoc
x = b, cn i vi hm lm th s l gi tr nh nht.
Th nhng, li khng c nhiu bn ngh n vic s dng tnh
cht ny vo gii ton. Mt trong nhng nguyn nhn c l l
tnh cht ca hm li. Nh trn cp, hm li s t
cc i ti bin v hm lm s t cc tiu ti bin. Nhng
84

Tp ch online ca cng ng nhng ngi yu Ton

bi ton ny th li khng c bin r rng, cc bin c bin di


nhng li khng c bin trn.
Mt iu na cng cn phi ni n l hu ht cc bn hc
sinh u ch c tm nhn v m m cha c n ci nhn vi
m. C ngha l bi cho bt ng thc bao nhiu bin th
cc bn ch nhn bng ng by nhiu bin ch khng ngh n
tm nhn khc i. l mt tm nhn sai lm. Trn thc t,
c c li gii thnh cng th ta nn bt u bng nhng th
nh nht nht, ch n mi kha cnh.
Bt ng thc cho c dng i xng vi ba bin a, b, c, th
th n cng l bt ng thc i xng vi hai bin bt k no
trong ba bin trn. Chn hn cc bn vn con nh chng
ta hc t cp 2 rt nhiu rng cc bi ton i xng hai
n c th c x l hiu qu bng php t n ph tng-tch
S = x + y, P = xy nh vo quan h ca chng: S2 > 4P.
C th hn, nu ta c nh c th bt ng thc s c dng i
xng vi a v b. Khi , nh php t S = a + b, P = ab, ta c
th chuyn bt ng thc v dng:
g(S, P) > 0.
Khi , nu ta c nh S na th y s ch cn l mt bt ng
thc vi mt bin l P v lc ny
 P c chn min vi
S2
bin trn ln bin di l 0, 4 .

n y, nu ta c th suy xt c tnh n iu hoc tnh li


lm ca gP (S, P) th cng c th a ra c kt lun v tnh
cht cc cc tr ca n ri t i n li gii.
Li gii chi tit theo hng ny nh sau: t f(a, b, c) = VT VP.
Khi , ta phi chng minh f(a, b, c) > 0. C nh c v S = a + b.
2
t P = ab th ta c 0 6 P 6 S4 . Ta c bin i:
q
q

a + b = a + b + 2 ab = S + 2 P
v (a b)2 + (b c)2 + (c a)2 (a + b + c)2 = h(P), trong h(P)
l mt biu thc bc nht ca P. Do :


q

f(a, b, c) = (S + c)
P + S + 2 P + h(P) = g(P).
85

Tp ch online ca cng ng nhng ngi yu Ton

Nu c hai s a, b u bng 0 th bt ng thc hin nhin ng


nn ta ch cn xt trng hp S > 0 l (l lun ny l m
2
bo khong 0, S4 tn ti, m bo cho vic xt o hm ca

2
g(P)). D thy hm s g(P) lin tc trn 0, S4 . Ngoi ra, vi mi
2
P 0, S4 , ta tnh c:
1

1
g 0 (P) = (S + c) + p P
2 P 2 S+2 P

+k

trong k l hs cao nht ca h(P). R rng


g 0(P) l hm gim

2
2
ngt trn 0, S4 nn g(P) l hm lm trn 0, S4 . T suy ra


S2
g(P) > min g(0), g
4




S2
P 0,
.
4

Nh th, chng minh


 bt ng thc cho, ta ch cn chng
S2
minh g(0) > 0 v g 4 > 0. Mt khc, ta li d thy:
g(0) = f(S, 0, c),

S2
g
2


S S
=f , ,c .
2 2

Do , t nhng l lun trn, c th thy rng ta ch cn xt


bt ng thc ti hai trng hp: c mt s bng 0 hoc c hai
s bng nhau, l . hng 3, ta chng minh c bt
ng thc ng ti trng hp th nht. Nh vy, ta ch cn
phi kim tra trng hp th hai na l c.
Gi s a = b. Khi , bt ng thc tr thnh

(2a + c) a + 2 ac + 2(a c)2 > (2a + c)2 .

Sau khi khai trin v rt gn, ta phi chng minh

4a ac + 2c ac + c2 > 7ac.

V ln c dng tng, cn v b c dng tch gi cho ta ngh ngay


n bt ng thc AM-GM nh gi:
q

4 2
7
2
4a ac + 2c ac + c > 7 a ac c ac c2 = 7ac.
86

Tp ch online ca cng ng nhng ngi yu Ton

Li bnh. Bng cch s dng hm li v bt ng thc Karamata1 , ta s c thm cch nhn nhn tng quan hn cho nhiu
vn , nm bt c bn cht tt hn.
Bi ton 59. Cho a, b, c [1, 2]. Chng minh rng


1 1
1
+ +
6 10.
(a + b + c)
a b c
Li gii. C nh b, c. t f(a) = (a + b + c)
f 0 (a) =

1 1 b+c
+
,
b c
a2

f 00 (a) =

1
a

1
b

1
c

2(b + c)
> 0.
a3


, ta c

Do vy f(a) l hm li, suy ra gi tr ln nht ca f(a) trn on


[1, 2] s t c ti bin, tc l ti a = 1 hoc a = 2.
Lp lun tng t, ta cng c kt lun nh th vi b v c. T
vi ch s i xng ca cc bin, ta suy ra ch cn kim tra
tnh ng n ca bt ng thc trong cc trng hp sau:
1. C ba bin bng nhau (cng bng 1 hoc 2);
2. C hai bin bng 1 v mt bin bng 2;
3. C hai bin bng 2 v mt bin bng 1.
Nhng tnh ton n gin cho thy trong c ba trng hp ny,
bt ng thc ca ta u ng.
1

Bt ng thc Karamata c xy dng da trn khi nim b tri v


tnh cht tip tuyn ca hm li:
Cho hai b s khng tng A = (a1 , a2 , . . . , an ) v B = (b1 , b2 , . . . , bn ), trong
ai , bi u cng thuc vo mt min I. Ta ni rng A tri hn B, k hiu A  B,
nu cc iu kin sau c tha mn ng thi:

a1 > b1

a1 + a2 > b1 + b2
.........

a1 + a2 + + an1 > b1 + b2 + + bn1

a1 + a2 + + an1 + an = b1 + b2 + + bn1 + bn
Lc ny, nu f(x) l mt hm kh vi bc hai v li trn I th:
f(a1 ) + f(a2 ) + + f(an ) > f(b1 ) + f(b2 ) + + f(bn ).

87

Tp ch online ca cng ng nhng ngi yu Ton

Bi ton 60. Cho x1 , x2 , . . . , xn [0, 1]. Chng minh rng


(1 + x1 + x2 + + xn )2 > 4(x21 + x22 + + x2n ).
Li gii. C nh x2 , . . . , xn . t:
f(x1 ) = 4(x21 + x22 + + x2n ) (x1 + x2 + + xn + 1)2 .
Ta thy ngay f 00 (x1 ) = 8 2 = 4 > 0, suy ra f(x1 ) l hm li. V
nh vy, f(x1 ) t gi tr ln nht khi x1 = 0 hoc x1 = 1, suy
ra ta ch cn xc lp tnh ng n ca hai bt ng thc, thu
c bt ng thc ban u vi x1 = 0 v x1 = 1.
Lp li cc l lun ny vi mi mt trong hai bt ng thc
thu c, ta nhn c kt qu tng t: ch cn kim tra tnh
ng n ca mi mt trong hai bt ng thc ti x2 = 0 v
x2 = 1. Tip tc l lun tng t, ta thu c l ch cn kim
tra tnh ng n ca tt c 2n bt ng thc, thu c t bt
ng thc ban u vi mt phn (c th l rng) cc bin s
bng 0 v phn cn li bng 1. Do tnh i xng ca bt ng
thc ban u, c th xt:
x1 = x2 = = xk = 1,

xk+1 = xk+2 = = xn = 0,

0 6 k 6 n.

Vi cc gi tr nh vy ca bin s, bt ng thc c dng:


(k + 1)2 > 4k.
Mt kt qu hin nhin, n tng ng vi (k 1)2 > 0.
Bi ton 61 (Bt ng thc Turkevici). Cho a, b, c, d l cc s
thc khng m. Chng minh rng
a4 + b4 + c4 + d4 + 2abcd > a2 b2 + a2 c2 + a2 d2 + b2 c2 + b2 d2 + c2 d2 .
Li gii. t x = a2 , y = b2 , z = c2 v t = d2 . Khi , bt ng
thc cn chng minh tng ng vi
p
x2 + y2 + z2 + t2 + 2 xyzt > xy + xz + xt + yz + yt + zt,
hay

p
(x + y)2 + z2 + t2 + 2 xyzt > 3xy + (x + y)(z + t) + zt.
88

Tp ch online ca cng ng nhng ngi yu Ton

C nh z, t, x + y = S v t xy = P th ta c 0 6 P 6 S4 . Bt
ng thc trn c th vit li di dng:

f(P) = 2 ztP + S2 + z2 + t2 3P S(z + t) zt.

V g(P) = P l hm lin tc v lm trn [0, +) nn ta cng d


thy f(P) cng l hm lin tc v lm trn [0, +). Do :

 2 


S
S2
f(P) > min f(0), f
, P 0,
.
4
4
Nh vy, chng minh bt ng
 thc cho, ta ch cn xt
S2
hai bt ng thc f(0) > 0 v f 4 > 0. Mt cch tng ng, ta
a c bi ton v xt ti hai trng hp: xy = 0 v x = y.
Tng t, ta cng ch cn xt hai trng hp: zt = 0 v z = t. Kt
hp cc l lun li, ta thy ch cn xt 2 trng hp ln sau:
Trng hp 1: Trong 4 s x, y, z, t c mt s bng 0. Gi s
t = 0, khi bt ng thc tr thnh:
x2 + y2 + z2 > xy + yz + zx.
Mt kt qu qu quen thuc.
Trng hp 2: x = y v z = t. Khi , ta d thy bt ng thc
c th vit c di dng (x z)2 > 0.
Bi ton 62 (VMO, 1992). Cho n (n > 2) s thc x1 , x2 , . . . , xn
thuc on [1, 1] c tng bng n 3. Chng minh rng
x21 + x22 + + x2n 6 n 1.
Li gii. Do tnh i xng nn khng mt tnh tng qut, ta c
th gi s 1 > x1 > > xn > 1. Ta chng minh:
(1, 1, . . . , 1, 0, 1)  (x1 , x2 , . . . , xn2 , xn1 , xn ).
D thy, t gi thit ta c

1 > x1

1 + 1 > x1 + x2

1 + 1 + + 1 > x1 + x2 + + xn2

1 + 1 + + 1 + 0 + (1) = x + x + + x
1
2
n2 + xn1 + xn
89

(5)

Tp ch online ca cng ng nhng ngi yu Ton

Do vy, chng minh (5), ta ch cn chng minh


n 2 = 1 + 1 + + 1 + 0 > x1 + x2 + + xn1 = n 3 xn .
Bt ng thc ny tng ng vi xn > 1, hin nhin ng
theo gi thit. Vy (5) c chng minh.
T , s dng bt ng thc Karamata vi ch rng hm
f(x) = x2 lin tc v li trn R, ta c
(n2)f(1)+f(0)+f(1) > f(x1 )+f(x2 )+ +f(xn2 )+f(xn1 )+f(xn ),
hay
x21 + x22 + + x2n 6 n 1.
Bt ng thc c chng minh.
Bi ton 63 (Vit Nam TST, 2011). Cho s t nhin n > 3. Xt n
s thc x1 , x2 , . . . , xn tha mn ng thi cc iu kin:
i) x1 > x2 > > xn ;
ii) x1 + x2 + + xn = 0;
iii) x21 + x22 + + x2n = n(n 1).
Tm gi tr ln nht v gi tr nh nht ca tng S = x1 + x2 .
Li gii. a) Tm max S: T gi thit, ta c
x1 + x2 = (x3 + x4 + + xn ).
T , s dng bt ng thc Cauchy-Schwarz, ta thu c
S2 = (x3 + x4 + + xn )2 6 (n 2)(x23 + x24 + + x2n )




S2
2
2
.
= (n 2) n(n 1) (x1 + x2 ) 6 (n 2) n(n 1)
2
p
Gii bt phng trnh trn, ta c S 6 2(n 1)(n 2). Mt
khc, d thy ng thc xy ra khi:
p
p
2(n 1)(n 2)
2(n 1)(n 2)
x1 = x2 =
, x3 = = xn =
.
2
n2
p
Do , ta i n kt lun maxS = 2(n 1)(n 2).
90

Tp ch online ca cng ng nhng ngi yu Ton

b) Tm min S: Ta xt hai trng hp sau.


Trng hp n = 3: Do x1 > x2 > x3 nn ta c (x3 x1 )(x3 x2 ) > 0,
t suy ra x23 (x1 + x2 )x3 + x1 x2 > 0. M x3 = (x1 + x2 ) nn bt
ng thc trn cho php ta c lng:
x1 x2 > 2(x1 + x2 )2 .
S dng nh gi ny v gi thit x21 + x22 + x23 = 6, ta thu c
2(x1 + x2 )2 = (x1 + x2 )2 + (x1 + x2 )2 = (x1 + x2 )2 + x23
= 2x1 x2 + 6 > 4(x1 + x2 )2 + 6.
T suy ra S = x1 + x2 > 1 (ch rng S > 0). Mt khc, d thy
du bng c th t c khi x1 = 2 v x2 = x3 = 1 nn p s
ca bi ton trong trng hp ny l 1.
Trng hp n 4: D thy vi gi thit ca bi th:

x2 , x2 , . . . , x2 , x2 + x3 + + xn (n 2)x2  (x2 , x3 , . . . , xn ).

Mt khc, ta li c ch rng hm s f(x) = x2 lin tc v li


trn R. Do , theo bt ng thc Karamata th:

2
x22 + x23 + + x2n 6 (n 2)x22 + x2 + x3 + + xn (n 2)x2

2
= (n 2)x22 + x1 + (n 2)x2 .

Kt hp vi gi thit iii), ta thu c


2
x21 + (n 2)x22 + x1 + (n 2)x2 > n(n 1).

(6)

By gi, ta s chng minh:


2
n(n 1)
(x1 + x2 )2 > x21 + (n 2)x22 + x1 + (n 2)x2 .
4

(7)

C nh x1 v t g(x2 ) = VT VP. Ta thy g(x2 ) l hm bc hai


vi h s cao nht bng n(n1)
(n2)(n2)2 = (n1)(3n8)
<0
4
4
nn n l hm lm. Ta li c ch rng:
x1 + (n 1)x2 > x1 + x2 + + xn = 0,
91

Tp ch online ca cng ng nhng ngi yu Ton

x1
th nn t gi thit ta suy ra n1
6 x2 6 x1 . V g(x2 ) l hm lm

x1
nn gi tr nh nht ca n s l gi tr nh nht ca g n1
v g(x1 ). Kim tra trc tip, ta c


x1  n2 (n 4)x21
> 0.
g
=
n1
4(n 1)
 x1

Do g(x2 ) > 0, x2 n1
, x1 . nh gi (7) c chng minh.
g(x1 ) = 0,

Kt hp hai bt ng thc (6) v (7), ta thu c ngay S > 2 (ch


rng S > 0). Mt khc, d thy du bng c th t c khi
x1 = x2 = = xn1 = 1 v xn = 1 n nn ta suy ra kt qu ca
bi ton trong trng hp ny l 2.
Sau y l mt s bi ton t luyn:
Bi ton 64. Cho a, b, c [0, 1]. Chng minh rng
b
c
5
a
+
+
+ abc 6 .
1 + bc 1 + ca 1 + ab
2
Bi ton 65. Cho a, b, c [0, 1]. Chng minh rng
a
b
c
+
+
+ (1 a)(1 b)(1 c) 6 1.
1+b+c 1+c+a 1+a+b
Bi ton 66. Cho a, b, c, d [1, 2]. Chng minh rng
a+c
3
a+b c+d
+

6 .
c+d a+b b+d
2
Bi ton 67. Cho a, b, c l cc s thc khng m tha mn iu
kin a + b + c = 3. Chng minh rng
2(a2 + b2 + c2 ) + 3abc > 9.
Bi ton 68. Cho a, b, c l cc s thc khng m tha mn khng
c hai s no ng thi bng 0 v a + b + c = 1. Chng minh rng

bc +

b 
c  1
a 
ca +
ab +
6 .
b+c
c+a
a+b
4
92

Tp ch online ca cng ng nhng ngi yu Ton

Bi ton 69. Cho a, b, c, d l cc s thc khng m tha mn


iu kin a + b + c + d = 4. Chng minh rng
1
1
1
1
1
2
1
+
+
+
+
+
6 .
10 ab 10 ac 10 ac 10 bc 10 bd 10 cd
3
Bi ton 70. Cho a, b, c l cc s thc khng m tha mn khng
c hai s no ng thi bng 0. Tm hng s k ln nht sao cho
bt ng thc sau lun ng:


b
c
3 k max (a b)2 , (b c)2 , (c a)2
a
+
+
> +
.
b+c c+a a+b
2
ab + bc + ca
Bi ton 71 (Kim tra i tuyn KHTN, 2014). Chng minh bt
ng thc sau ng vi mi a > b > c > d > 0:

b 
c 
d  81
a 
1+
1+
1+
> .
1+
b+c
c+d
d+a
a+b
16
Bi ton 72. Cho x1 , x2 , . . . , xn l cc s thc dng tha mn
iu kin x1 + x2 + + xn = 1. Chng minh rng

n
1 + x1 1 + x2
1 + xn
n+1

>
.
1 x1 1 x2
1 xn
n1
Bi ton 73 (Trung Quc, 2005). Cho a, b, c l cc s thc
dng c tng bng 1. Chng minh rng
10(a3 + b3 + c3 ) 9(a5 + b5 + c5 ) > 1.
Bi ton 74. Cho x1 , x2 , . . . , xn l cc s thc khng m tha
mn iu kin x1 + x2 + + xn = n. Chng minh rng
2(x31 + x32 + + x3n ) + n2 6 (2n + 1)(x21 + x22 + + x2n ).
Bi ton 75 (Trung Quc, 2013). Cho s t nhin n > 2. Xt hai
b s thc khng m bt k (a1 , a2 , . . . , an ) v (b1 , b2 , . . . , bn ).
Chng minh rng
v
u n


n
n
2
n
X
X
uY
n
2
2t
n
a
+
b
>
n
(a2i + b2i ).
i
i
(n 1)n1 i=1
i=1
i=1
93

Tp ch online ca cng ng nhng ngi yu Ton

Bi ton 76 (M, 1994). Cho a1 , a2 , . . .


, an l cc s thc dng
tha mn iu kin a1 + a2 + + ak > k vi mi k = 1, 2, . . . , n.
Chng minh rng


1
1
1
2
2
2
a1 + a2 + + an >
1 + + +
.
4
2
n


Bi ton 77 (Trung Quc, 1997). Cho x1 , x2 , . . . , x1997 13 , 3

tha mn iu kin x1 + x2 + + x1997 = 318 3. Tm gi tr ln


12
12
nht ca tng S = x12
1 + x2 + + x1997 .
Bi ton 78. Cho a, b, c l cc s thc khng m tha mn iu
kin a + b + c = 1. Chng minh rng
r
r
r
1a
1b
1c
2
+
+
61+ .
1+a
1+b
1+c
3

3. Xut x ca bi ton
Qua tm hiu v nghin cu ti liu, chng ti suy on rng
bi ton ny c xut x t bi ton sau y:
Bi ton 79. Cho a, b, c l cc s thc khng m tha mn iu
kin a + b + c = 1. Chng minh rng
p
p
p

a + (b c)2 + b + (c a)2 + c + (a b)2 > 3.


(8)

Tt nhin, y ch l nhng suy on c tnh ch quan, nhng


nu suy xt k, cc bn s thy bi VMO 2015 chnh l mt
mu cht quan trng trong chng minh bt ng thc (8). Tht
vy, bnh phng hai v ca (8), ta thy n tng ng vi
X q
 X

a + (b c)2 b + (a c)2 +
(a b)2 > 2.
2
n y, s dng bt ng thc Cauchy-Schwarz, ta c
X q
 X


a + (b c)2 b + (a c)2 >
ab + (a c)(b c)
X
X
ab +
(a c)(b c)
=
X
1X
=
ab +
(a b)2 .
2
94

Tp ch online ca cng ng nhng ngi yu Ton

Do , chng minh (8), ta ch cn chng minh c


X
X
ab +
(a b)2 > 1,
hay

 X
X 2
X X

2
a
ab +
(a b) >
a .

y chnh l bt ng thc v phi trong bi s 2 ca VMO


nm nay. Cn v tri c l tc gi t thm ra vi mc ch
g im cho cc th sinh tham d k thi.
Ni ring v bt ng thc (8), n cng c mt xut x rt th
v... t hnh hc. Chnh xc hn l t s tng t ha mt bt
ng thc v ng trung tuyn ca tam gic. Ta bit rng,
trong mt tam gic vi di ba cnh l a, b, c th:
ma + mb + mc 6 2p.
Xt cc tam gic c na chu vi p = 1. t a = y + z, b = z + x v
c = x + y vi x, y, z > 0. Khi , ta c x + y + z = 1 v:
2(z + x)2 + 2(x + y)2 (y + z)2
2b2 + 2c2 a2
=
4
4
2
(y z)2
(y z)
=x+
.
= x(x + y + z) +
4
4

m2a =

T php bin i ny, ta thu c bt ng thc


r
r
r
(y z)2
(z x)2
(x y)2
x+
+ y+
+ z+
6 2.
4
4
4
C th thy bt ng thc (8) chnh l mt s tng t ha
bng cch thay i h s ca cc bnh phng di du cn.
Vi cch lm ny v mt s th thut thay i cu trc biu
thc, ta cng c th to ra c nhiu bt ng thc anh em
khc ca (8) nh sau (bn c c th th sc):
Bi ton 80. Cho a, b, c l cc s thc khng m tha mn iu
kin a + b + c = 1. Chng minh rng
r
r
X
(b c)2
1X
a+
6 3+
(a b)2 .
4
2
95

Tp ch online ca cng ng nhng ngi yu Ton

Bi ton 81. Cho a, b, c l cc s thc khng m tha mn iu


kin a + b + c = 1. Chng minh rng
r


X
(b c)2
3 X
6 3+ 1
a+
|a b|.
4
2
Bi ton 82 (Crux Mathematicorum). Cho a, b, c l cc s thc
khng m tha mn iu kin a + b + c = 1. Chng minh rng
1
1
1
q
+q
+q
> 5.
2
2
(ca)
(ab)2
a + (bc)
b
+
c
+
4
4
4
Bi ton 83. Cho a, b, c l cc s thc khng m tha mn
iu

3
kin a + b + c = 1. Chng minh rng vi mi 0 6 k 6 1 2 , ta c
p
p
p

a + k(b c)2 + b + k(c a)2 + c + k(a b)2 6 3.

4. Bi ton tng qut


Nh cp, phn cui ny, chng ti xin c xut
mt tng qut cho bt ng thc v phi trong VMO nm
nay nh sau (trng hp bi VMO chnh l ng vi n = 3):
Bi ton 84 (Tng qut VMO 2015). Cho s t nhin n > 2. Xt
n s thc khng m x1 , x2 , . . . , xn . Chng minh rng
X
 X

n
n
X
X
2

xi xj + (n 2)
x2i > 4
xi
xi xj .
n 1 i=1
16i<j6n
i=1
16i<j6n

Li gii. t ai = xi , i = 1, 2, . . . , n. Khi , bt ng thc


cn chng minh tng ng vi
X
 X

n
n
X
X
2
2
ai
ai aj + (n 2)
a4i > 4
a2i a2j ,
n 1 i=1
16i<j6n
i=1
16i<j6n
hay
X
2
X
 X

n
n
n
X
2
4
2
2
a
ai aj + n
ai > 2
ai .
n 1 i=1 i
16i<j6n
i=1
i=1
96

(1)

Tp ch online ca cng ng nhng ngi yu Ton

Nu tt c cc s ai u bng 0 th bt ng thc trn hin


nhin ng. Do ta ch cn xt trng hp a1 + + an > 0.
Lc ny, vi ch tnh thun nht ca bt ng thc, ta c
th chun ha cho a1 + a2 + + an = n.
Vi iu kin ny, ta d dng nh gi c
n 6 a21 + a22 + + a2n 6 n2 .
Do vy, ta c th t a21 +a22 + +a2n = n+n(n1)t2 vi 0 6 t 6 1.
2 Pn
P
Pn
V 2 16i<j6n ai aj =
i=1 a2i = n(n 1)(1 t2 ) nn
i=1 ai
bt ng thc (1) c th c vit li di dng:
2

n 1 + (n 1)t (1 t ) + n

n
X
i=1


2
a4i > 2n2 1 + (n 1)t2 ,

hay



a41 + a42 + + a4n > n 1 + (n 1)t2 1 + (2n 1)t2 .

By gi, ta chng minh cc b sau:

B 1. Gi s a1 , a2 , . . . , an l cc s thc tha mn ng thi


cc iu kin a1 +a2 + +an = n v a21 +a22 + +a2n = n+n(n1)t2
(t > 0). Khi , vi mi i = 1, 2, . . . , n, ta c
1 (n 1)t 6 ai 6 1 + (n 1)t.
Chng minh. Do cc bin c vai tr nh nhau nn ta ch cn
chng minh bt ng thc ng vi i = 1 l . S dng bt
ng thc Cauchy-Schwarz, ta c
n + n(n 1)t2 > a21 +

(a2 + + an )2
(n a1 )2
= a21 +
.
n1
n1

Gii bt phng trnh vi n a1 , ta thu c ngay kt qu.

B 2. Gi s a1 , a2 , . . . , an l cc s thc tha mn cc iu
kin ca b 1. Khi , ta c bt ng thc sau:

3
a31 + a32 + + a3n > (n 1)(1 + t)3 + 1 (n 1)t .
97

Tp ch online ca cng ng nhng ngi yu Ton

Chng minh. D thy vi t = 0 th a1 = a2 = = an = 1 v bt


ng thc hin nhin ng nn trong chng minh di y (v
cho c b sau), ta ch xt trng hp t > 0. Ta c
n
X

a3i

i=1

n
X
i=1
n
X
i=1

a2i

n
 
X
ai + (n 1)t 1 + 1 (n 1)t
a2i
i=1






a2i ai + (n 1)t 1 + n 1 (n 1)t 1 + (n 1)t2 .

Theo b 1, d thy ai + (n 1)t 1 > 0. Do vy, ta c iu


kin s dng bt ng thc Cauchy-Schwarz nh sau:
X
X

n
n




2
ai ai + (n 1)t 1
ai + (n 1)t 1 >
i=1

>

X
n
i=1

Do

Pn 
i=1

n
X
i=1

i=1


ai ai + (n 1)t 1


ai + (n 1)t 1 = n(n 1)t v:

n
n

 X

X
ai ai + (n 1)t 1 =
a2i + (n 1)t 1
ai
i=1

 i=1

= n + n(n 1)t + n (n 1)t 1
= n(n 1)t(1 + t)
2

nn t nh gi trn, ta suy ra
n
X
i=1



a2i ai + (n 1)t 1 > n(n 1)t(1 + t)2 .

V nh th, ta c
n
X
i=1




a3i > n(n 1)t(1 + t)2 + n 1 (n 1)t 1 + (n 1)t2

3
= (n 1)(1 + t)3 + 1 (n 1)t .

B 3. Gi s a1 , a2 , . . . , an l cc s thc khng m tha mn


1
th ta c
cc iu kin ca b 1. Khi , nu 0 6 t 6 n1

4
a41 + a42 + + a4n > (n 1)(1 + t)4 + 1 (n 1)t .
98

Tp ch online ca cng ng nhng ngi yu Ton

Chng minh. Ta c bin i sau:


n
X

a4i

n
X

i=1

a3i

i=1

n
 
X
ai + (n 1)t 1 + 1 (n 1)t
a3i .

(2)

i=1

S dng bt ng thc Holder, ta c


X
X

n
n



 2
3
ai ai + (n 1)t 1
>
ai + (n 1)t 1
i=1

>

X
n
i=1

i=1


ai ai + (n 1)t 1

T , s dng cc kt qu bin i ca b 2, ta suy ra:


n
X
i=1



a3i ai + (n 1)t 1 > n(n 1)t(1 + t)3 .

S dng kt qu ny kt hp vi b 2 vo bin i (2), ta thu


c ngay kt qu cn chng minh.

1
Tr li bi ton cho, c th thy rng: Nu 0 6 t 6 n1
th
bng cch s dng b 3, ta ch cn chng minh

4



(n 1)(1 + t)4 + 1 (n 1)t > n 1 + (n 1)t2 1 + (2n 1)t2 .

Bt ng thc ny tng ng vi



nt2 1 (n 1)t 3n 4 (n 1)(n 4)t > 0,
hin nhin ng do 0 6 t 6

Vi

1
n1

1
.
n1

< t 6 1: S dng bt ng thc Holder, ta c


X
n
i=1

a4i

 X
i=1

ai

2

>

X
n
i=1

a2i

3

t ta suy ra

3
a41 + a42 + + a4n > n 1 + (n 1)t2 .

V nh th, ta ch cn chng minh c



3



n 1 + (n 1)t2 > n 1 + (n 1)t2 1 + (2n 1)t2 .
99

Tp ch online ca cng ng nhng ngi yu Ton

Bt ng thc ny tng ng vi



t2 (n 1)t 1 (n 1)t + 1 > 0,
hin nhin ng do t >

1
.
n1

Nhn xt. C th thy bi ton tng qut l mt bt ng thc


kh cht vi du bng xy ra ti hai trng hp a1 = = an v
a1 = = an1 , an = 0 (v cc hon v). Ngoi ra, cc cng c x
l cho bt ng thc nhiu bin vn cn nhiu hn ch. Chnh
iu ny to nn kh ca bi ton.
Ngoi li gii trn, chng ta cng c th s dng hm li v
mt b dn bin ph chng minh. Li gii theo cch ny
s gn gng hn. Tuy nhin, y chng ti chn cch di hn
trnh by cng c nguyn nhn ca n.
B 1 trong li gii cho php ta chn min bin theo tham
s vi nhng nh gi rt cht, t nu ta bit cch s dng
hp l s thu c nhiu kt qu th v. Chng hn, trong cc
b 2 v 3, chng ti s dng k thut thm bt ca
bt ng thc Cauchy-Schwarz kt hp vi b 1 lm tng
tnh hiu qu ca cc nh gi.
Bn c no quan tm c th tm c bi vit Nh m khng
nh ca chng ti trong Chuyn Ton hc s 9 ca trng
PTNK (Tp. H Ch Minh), pht hnh thng 10 nm 2010.

100

Tp ch online ca cng ng nhng ngi yu Ton

PHN TCH V M RNG


TRONG CC BI TON T HP
L Phc L (Tp HCM)

Tm tt
Nh chng ta u bit, cc bi ton t hp i hi t duy
nhiu, nht l t duy tru tng, sng to khng theo
li mn. Trong phng thi c thi gian t, p lc cao, t c
th sinh no dm mo him lm t hp khi cc cu i
s, gii tch (thng c m hnh sn) vn cha xong. Tuy
nhin, ni i th cng phi ni li, t hp cng nh hnh
hc v s hc, bn cnh cc ni dung nh tnh th vn c
cc cu nh lng. Chng hn nh xt ring trong ch
hnh phng, nu bn no hc hnh cha tt nhng li s
dng tt cc cng c ph tr nh i s, lng gic, ta
, . . . vo bi ton th ch cn mt t c gng no
a bi ton nh tnh thun ty v nh lng l coi nh
c th t tin x l c rt nhiu bi kh. Tuy nhin,
li l mt cu chuyn di khc.
i vo vn chnh, chng ta c th thy rng t hp
cng th, bn cnh cc bi chng minh i hi s dng
cc lp lun logic, cc nguyn l t hp mt cch bi bn
th vn c cc bi nh lng nh th.

1. Phn tch tm li gii


Trong phn ny, chng ta s cng xem xt mt s bi ton lin
quan n cc tr ri rc v thng qua cc nh gi vi s nh,
trng hp c bit c gng pht hin ra quy lut ri t
gii quyt c vn . Ch rng c mt s bi ch nu hng
x l ch khng i su vo li gii chi tit.
Bi ton 1. Trn mt bn trn, c n > 3 ngi ngi. Bit rng
trong s h, ai cng lun ni di hoc lun ni tht v ngay lc
101

Tp ch online ca cng ng nhng ngi yu Ton

ny, h ni rng: C 2 ngi ngi cnh ti u l k ni di. Hi


trn bn c nhiu nht v t nht bao nhiu ngi ni di?
Li gii. y l mt bi mnh pht trin t mt cu vui dnh
cho hc sinh Tiu hc trong trng hp n = 5 (hon ton c th
th cc trng hp nh).
Tt nhin bi ton ny khng phi qu d dng nhn vo l
ra ngay, nht l khi i hi phi xy dng mt cch bi bn.
Chng ta hy th vi cc trng hp nh c gng tm cch
x l v c th d dng hnh dung cch tip cn hn.
Vi n = 3 th min = 2, max = 2. (Ch rng ph nh ca
vi mi l tn ti!)
Vi n = 4 th min = 2, max = 2.
Vi n = 5 th min = 3, max = 3.
Cc gi tr u th ln nht v nh nht bng nhau ri, nhng
nh th th ng ngha vi vic tn ti duy nht s lng ngi
ni di trn bn, c v khng hp l lm, ta th tip tc:
Vi n = 6 th min = 3, max = 4.
Vi n = 7 th min = 4, max = 4.
Vi n = 8 th min = 4, max = 5.
Vi n = 9 th min = 5, max = 6.
Vi n = 10 th min = 5, max = 6.
n y chc cc bn c th d dng nhn ra quy lut ca hai
dy min v max. Vi dy min th 2, 2, 3, 3, 4, 4, 5, 5 rt d nhn
ra. Tuy nhin, phi m t c cng thc tng qut ca n ch
khng th dng li . Mt kinh nghim cho thy rng khi s
thay i theo cc cm c di 2 nh th th 90% cng thc c
dng phn nguyn ca mt hm tuyn tnh theo n khi chia cho
2. Ta c th lm chm hn mt cht:
Vi n = 2k th min = k.
Vi n = 2k 1 th min = k.
102

Tp ch online ca cng ng nhng ngi yu Ton

Nh th, cng thc c th vit gp li l: min =


Tng t vi dy max, ta on c max =

 2n 
3

 n+1 
2

Nh th, n y, ta c c g trong tay? Nu vit ht ni


dung trn vo th c im no cha? Tht kh ni nhng c
l mun c im th ta phi c gng thm na. C th chng
minh cc gi tr kia tt nht l iu khng d nhng trc mt,
vic xy dng kh n gin.
Ta ch cn chia trng hp ra v "bt chc" theo cch lm
vi cc s nh. Chng hn trng hp n = 2k th cho nhng
ngi ni di tht ngi xen k l c c ngay kt qu. y
cng l mt kinh nghim na khi x l dng ny. Khi chng
minh iu kin , tc l xy dng cu hnh tha mn th phi
chia tng trng hp ra xt cho d. Nu nguyn cng thc
dng phn nguyn th rt kh, nhiu khi khng xy dng c.
Cn phn chng minh iu kin cn th cng tng i nh
nhng v c th ni chnh kt qu trn gi cho phn ny.
Ta ch nu nhn xt di y l coi nh xong:
1. Trong 2 ngi lin tip, phi c t nht mt ngi ni di.
2. Trong 3 ngi lin tip, phi c t nht mt ngi ni tht.
Hai nhn xt ny c v rt hin nhin nhng d ngh ra c
hay khng th cng ty ngi. Tnh kh d n y cng khng
cn khch quan na khi mi chuyn r rng c.
Bi ton 2. To dng n > 2 que dim xp thnh cc con s
nh hnh bn di:

Hi s ln nht v s nh nht m To c th nhn c khi xp


cc que dim l bao nhiu?

103

Tp ch online ca cng ng nhng ngi yu Ton

Li gii. bi ny, ta ch cn dng mt tng tham lam


(greedy strategy) n gin sau: s cng c nhiu ch s th
cng ln v c cng t ch s th cng nh. Nh m trng
hp s ln nht, To c th d dng thc hin c nh sau:
Nu n chn th xp

n
2

s 1.

Nu n l th xp 1 s 7 u tin v

n3
2

s 1.

R rng cch xp ny cho nhiu ch s nht v hin nhin s


tng ng s ln nht.
Tuy nhin, trng hp nh nht li khng n gin nh vy.
Mnh phi vit n n gn 30 mi d on c quy lut. Cn
l do ti sao c s khc nhau ny gia ln nht v nh nht th
d thi, v c im ca s lng cc que dim xp cc s.
Gi f(n) l s nh nht thu c. Ta th lit k cc kt qu xp
c bng tay nh sau:
f(2) = 1, f(3) = 7, f(4) = 4, f(5) = 2, f(6) = 0, f(7) = 8, f(8) = 10,
f(9) = 18, f(10) = 22, f(11) = 22, f(12) = 28, f(13) = 80, f(14) = 88,
f(15) = 108, f(16) = 188, f(17) = 200, f(18) = 208, f(19) = 288,
f(20) = 688, f(21) = 888, f(22) = 1088, f(23) = 1888, f(24) = 2008.
n y ta mi thy c c mt quy lut bt u r rng t
14 v n c chu k 7. Vic chng minh hu nh ch mang tnh
hnh thc (quy np) v kt qu trn qu c th.
Chng ta th sc vi mt bi tng t di y thy r hn
ngha ca vic lit k ny:
Bi ton 3. Cho s nguyn dng n. Tm s nguyn dng nh
nht c n ch s v chia ht ng thi cho 2, 3, 5, 7.
Li gii. Ch rng s chia ht cho 2, 3, 5, 7 th chia ht cho
210. Cng bng cch th tng t nh trn, ta gi f(n) l s
tha mn ng vi n cho trc th c kt qu nh sau:
f(1) = 0, f(2) khng tn ti, f(3) = 210, (4) = 1050,
f(5) = 10080, f(6) = 100170, f(7) = 1000020, f(8) = 10000200,
f(9) = 100000110, f(10) = 1000000050.
104

Tp ch online ca cng ng nhng ngi yu Ton

n y, ta thy ngay c im ca cc s cn tm l gm 1 ch
s 1 hng cao nht, tip theo l mt lot s 0, s cui cng l
0 v cc s hng chc, hng trm s thay i ph hp c
s chia ht cho 7. n f(10), ta thy quy lut quy v ging
.
f(4). L do n gin l v 10k+6 10k = 10k (106 1) .. 7 vi mi k
nguyn dng, theo nh l Fermat nh.
Nh vy, bng vic xy dng nh trn v tnh tun hon ca
cng thc, ta c th d dng hon tt bi ton.
Ta xt v d tip theo, cng thc phc tp hn nhiu:
Bi ton 4. C mt con th n n c c rt trong mt s ngy theo
quy lut sau:
1) Ngy u tin v ngy cui cng, n phi n 1 c ci.
2) Hai ngy lin tip nhau, con th phi n s c ci chnh lch
khng qu 1.
Hi s ngy t nht m con th n ht s c ci l bao nhiu?
Li gii. M t ca bi ton kh r rng v vic xy dng tng
i n gin, thm ch l trong trng hp n kh ln. Gi f(n)
l s ngy t nht cn tm, ta c dy sau:
f(1) = 1, f(2) = 2, f(3) = 3, f(4) = 3, f(5) = 4, f(6) = 4, f(7) = 5,
f(8) = 5, f(9) = 5, f(10) = 6, f(11) = 6, f(12) = 6, f(13) = 7,
f(14) = 7, f(15) = 7, f(16) = 7, f(17) = 8, f(18) = 8.
Quan st quy lut ca dy s, ta thy rng:
Gi tr ca f thay i ti cc s c dng k2 + 1.
Trong khong t (k 1)2 + 1 n k2 , gi tr ca f thay i
mt ln na ti im chnh gia.
T suy ra:
Vi (k 1)2 + (k 1) + 1 6 n 6 k2 th gi tr f(n) = 2k 1.
Vi k2 + 1 6 n 6 k2 + k th gi tr f(n) = 2k.
105

Tp ch online ca cng ng nhng ngi yu Ton

Cch xy dng th cng d thy do ta c th da theo m hnh


tam gic Pascal v la chn khi no cn phi c tam gic nh
nhn, nh bng ph hp.


4n 3 .
Nu tm cng thc trn mt cch tng qut th ta c
R rng bng mt lp lun th v no , ta c th tm ra c
cng thc ny nhng y, mi vic hon ton c th lm mt
cch th cng v trong thi gian khng qu lu. R rng cng
vic ny ng c xem xt v p dng!
v d di y, chnh vic kim tra kt qu trong trng hp
nh cho mnh cch chng minh cho bi ton.
Bi ton 5. Cho s nguyn dng n. Xt dy s nguyn dng
hu hn (ak ) gm k s hng sao cho vi mi 1 6 i < j 6 n th tn
ti t vi 1 6 t 6 k sao cho at = i, at+1 = j hoc at = j, at+1 = i.
Hi gi tr nh nht ca k l bao nhiu?
Li gii. Bi ton pht biu hi rc ri nhng nu ngm ngh
k th mi chuyn cng tng i r rng ( yu cu rng mi
cp s t 1 n n u phi l hai s cnh nhau no ca dy).
Ta th xt cc trng hp nh:
Vi n = 1 th k = 1.
Vi n = 2 th k = 2 ng vi dy 1, 2.
Vi n = 3 th k = 4 ng vi dy 1, 2, 3, 1.
Vi n = 4 th k = 8 ng vi dy 1, 2, 3, 4, 1, 3, 2, 4.
Vi n = 5th k = 11 ng vi dy 1, 2, 3, 4, 5, 1, 4, 2, 5, 3, 1.
Trong qu trnh xy dng ny, ta s gp mt s vn sau:
Do c tt c C2n cp s v c C22 = 1, C23 = 3 nn d on kt qu
l C2n + 1. Trng hp n = 5 hon ton hp l nhng n = 4 th li
khng xy dng c vi k = 7.
kim tra cn thn v trc quan, th v ra m hnh c cc s
v cc cnh ni chng nu cp tha mn iu kin bi.
T pht hin ra bi ton ny c lin h mt thit vi bi ton
v chu trnh Euler trong th y v dy s (ak ) trn chnh
l s lit k th t nh trong chu trnh .
106

Tp ch online ca cng ng nhng ngi yu Ton

Vic cng thm 1 y l do dy khng to thnh m hnh khp


kn nn cn lit k d ra 1 nh. T iu kin v tn ti ng
i Euler, ta thy cn phi chia thnh 2 trng hp chn l mi
c th gii quyt c bi ton. C th l:
Nu n l th tt c cc nh u bc chn nn tha mn
2
.
iu kin c chu trnh Euler, s k cn tm l C2n +1 = n n+2
2
Nu n chn th tt c cc nh u bc l nn phi thm
vo n2 cnh na c c a th c cc bc u chn
2
v s k cn tm l C2n + n2 = n2 .
Bi ton 6 (VNTST, 2006). Trong khng gian cho 2006 im m
trong khng c 4 im no ng phng. Ngi ta ni tt c cc
im li bi cc on thng. S t nhin m gi l s tt nu ta
c th gn cho mi on thng trong cc on thng ni bi
mt s t nhin khng vt qu m sao cho mi tam gic to bi
ba im bt k trong s cc im u c hai cnh c gn bi
hai s bng nhau v cnh cn li gn bi s ln hn hai s .
Tm s tt c gi tr nh nht.
Li gii. Cng nh nhiu bi ton khc, bi ny, ta thy s
2006 khng c ngha ln lm v th tng qut trong trng
hp n > 2 ty . T tng chia nh phn l mt trong cc i
din quan trng ca ng dng chin lc chia tr, thay v x
l bi ton ln, ta chia n ra v gii quyt tng phn nh.
bi ny, ta s ch ra mt tnh hung m khi khng phn tch
thu u, da theo cc xy dng cc b v th vi vi s nh,
ta s d dng ri vo mt ng nhn no dn n kt qu sai.
Ta cng tin hnh tng t nh cc v d nu, xt cc tnh
hung vi s nh:
Vi n = 2 th ch cn m = 0 l c.
Vi n = 3 th cn n 2 s nh nn chn m = 1.
Vi n = 4 th cng ch cn 2 s nn m = 1.
Vi n = 5 th ta cn 3 s v m = 2.
n y d thy rng lun tn ti mt cnh no c nh
s 0, gi s cnh ni A v B. Ta chia n 2 nh cn li thnh
107

Tp ch online ca cng ng nhng ngi yu Ton

2 phn th r rng mi nh u phi ni vi A hoc B bi


cnh nh s 0. Ta li chia chng thnh 2 tp hp: X cha cc
nh ni vi A bi cnh nh s 0, Y cha cc nh ni vi B bi
cc cnh nh s 0.

Khi ta c th cho:
T mi nh tp X sang mi nh tp Y, cnh ni vi nhau
nh s 0.
T nh A sang tp Y v t nh B sang tp X, cnh ni vi
nhau nh s 1.



Cn li trong X v Y, ta cn s dng max f |X| , f |Y| . Tuy
nhin, do cn chn nh nht nn




f(n) = 2 + min max f |X| , f |Y|
.

Hn na, f(n) l hm n iu v |X| + |Y| = n 2 nn



h




n 1i
.
min max f |X| , f |Y|
=f
2
T i n kt lun:
h

n 1i
f(n) = 2 + f
.
2

Lp lun c v ph hp nhng ng tic l kt lun ny li sai


v vi phn v d khi xy dng trong trng hp n = 7, n = 8, ta
d dng pht hin ra vn nm ch cch xy dng m hnh.
108

Tp ch online ca cng ng nhng ngi yu Ton

Trn thc t, ta c th chia ra ngay t u ch khng cn phi


xt thm im A, B v cng thc ng l:
h

n 1i
f(n) = 1 + f
.
2
Bn c hy th t phn tch xem ti sao cng thc truy hi
li c thay th nh trn nh!
Bi ny cho ta thy rng trong nhiu trng hp, ta xy dng
cc m hnh da theo kinh nghim nhng cng c lc cn phi
xt cc gi tr c th trong trng hp nh th mi pht hin ra
c vn .
Ngoi ra, vic xy dng cho cc gi tr nh to thnh dy ri
tm quy lut nh nu trn khng ch p dng c cho dng
ton cc tr ri rc m mt s bi ton nh tnh khc vn c.
Th th xt bi ton kinh in sau:
Bi ton 7. C hai ngi A, B chi tr chi bc si v ban u
c n vin si, A i trc. Mi ngi c th bc 2, 3 hoc 6 vin v
ai bc c vin cui cng th thng. Nu ch cn 1 vin th ngi
chi lt bc v thng lun. Hi vi nhng gi tr no ca n
th A c chin lc thng?
Li gii. Trc khi tin hnh xy dng tng t nh trn, cc
bn c th cn hai nh ngha sau v tr chi t hp cn bng
(tc l hai bn c cch chi nh nhau):
V tr thng: l v tr m tn ti mt cch i n v tr thua
(v tr y ni gi tr n hin ti, thng ny ch ngi
chi hin ti v i n ch s si cn li sau lt chi ).
V tr thua: l v tr m lun phi i n mt v tr thng.
C v hi m h, ta th phn tch vi bi ton c th trn. t
f(n) : N {0, 1} v n nhn gi tr 1 khi A c chin lc thng,
0 khi B c chin lc thng. Hin nhin A thng th B thua v
ngc li nn ta c f(1) = 1, f(2) = 1, f(3) = 1.
n y tnh f(4), ta thy rng nu A bc 2 hoc 3 th u
dn n v tr thng ca i phng nn theo nh ngha trn,
4 chnh l v tr thua v f(4) = 0.
109

C th, ta tnh tip c:

Tp ch online ca cng ng nhng ngi yu Ton

f(5) = 0, f(6) = 1, f(7) = 1, f(8) = 1, f(9) = 0, f(10) = 0, . . .


D thy quy lut: nu n chia 5 d 0, 1, 2 th A thng; v ngc
li th B thng.
ngh ra c iu ny th r rng khng d nhng on
ra c th li qu d dng. Chng minh c thc hin mt
cch nhanh chng bng quy np.
Bi ton 8 (Cuc thi Brilliant.org). Trn mt phng ta Oxy,
xt tp hp S gm cc im tha mn:


(x, y) | 0 6 x, y 6 999; x, y Z .
Hai bn A, B chi mt tr chi nh sau: Ban u, c 1 qun c
v tr no thuc tp hp S v h di chuyn qun c theo cch
sau:
Bn A i trc v c th di chuyn qun c xung di 1
n v hoc sang tri 2 n v.
Bn B i sau v c th di chuyn qun c xung di 2 n
v hoc sang tri 1 n v.
Hai bn lun phin chi v ai buc phi di chuyn qun c khi
gc phn t th nht th thua. Hi c tt c bao nhiu v tr m A
c chin lc thng tr chi?
Li gii. Ta s thc hin kim tra trc tip cc v tr gn gc
ta th tm mt quy lut no :

110

Tp ch online ca cng ng nhng ngi yu Ton

Trong hnh trn, ta quy c cc im trng l ngi A c chin


lc thng, im en l ngi B c chin lc thng. Ta thy
rng da vo mt s kim tra trc tip vt cn vi cc v tr
(0, 0), (0, 1), (1, 0), (2, 0), . . . th c quy lut l cc v tr thng
thua hnh vung 3 3 ngay st gc ta c lp li.
iu ny c th gii thch c thng qua c im: A sang tri
1 n v th B sang tri 2 n v, tng cng l 3 n v; tng t
khi i xung di. T d dng gii quyt c bi ton.
Di y l mt s bi ton tng t, cc bn c th t lp li
cc thao tc trn d on kt qu, mt cng vic ht sc th
v, th cng nhng li mang n nhng hiu qu bt ng.
Bi ton 9. C n chic cc c p thnh mt vng trn v di
1 trong cc chic cc ny, c mt ng xu. mi lt, ngi chi
c th chn ra 4 chic cc lin tip v m ln. Nu c ng xu th
tr chi kt thc. Nu khng th ngi chi s tr 4 chic cc v v
tr c v bng mt cch no , ng xu s di chuyn sang mt
trong hai cc k n. Ngi chi lun suy lun, phn tch k trong
qu trnh bc. Hi trong trng hp xu nht th s ln cn phi
thao tc l bao nhiu?
Bi ton 10. mt ca hng n, ngi ch ch s dng cc ng
tin c gi tr l ly tha t nhin ca 3. C mt ngi khch cn
mua mt mn hng c gi tr l n. D chun b sn mt s loi
ng tin c gi tr l ly tha ca 3 (mi loi c s lng ty )
nhng khng may l vi cc loi ng tin khng th no tr
c va ng gi tin n. Ngi bn cng khng mun thi li
tin tha. Th l ngi khch nh phi tr nhiu hn gi tr ca
mn hng. tit kim, ng tr s tin m > n vi m nh nht
c th. Hi trong cc cch tr s tin m , s ng tin nhiu
nht s dng l bao nhiu?
Bi ton 11. m mt cnh ca, tn trm cn phi bm ng
th t ca n ci nt m ngi ch quy nh trc. Nu bm nt
ng th t th th cc nt s gi nguyn v tr, nu ch cn bm
sai mt nt th ton b cc nt bm s b bt ln v tn trm
s phi th li t u.
Chng hn vi n = 3 v th t cc nt cn bm l 2, 3, 1. Nu
ban u, tn trm bm nt 1 hoc 3 th nt s bt ln ngay lp
111

Tp ch online ca cng ng nhng ngi yu Ton

tc. Nu hn bm nt 2 th nt s gi nguyn (do nt 2 c th t


u tin). Tip theo, nu hn bm nt 1 th do sai th t nn c
nt 2 bm trc v nt 1 s b bt ln. Nu hn bm nt
3 tip theo nt 2 th c hai nt s gi nguyn v cn li hn ch
cn bm nt 1. D mt tn trm c thng minh n u th trong
tnh hung xu nht, hn cng phi bm nt 7 ln. Hi vi n nt
trong trng hp xu nht, tn trm phi bm nt bao nhiu ln
m c ci ca?
Bi ton 12. Hai ngi chi mt tr chi nh sau: u tin, c
mt s n c chn t tp hp {2, 3, 4, . . . , 999}. Ngi chi th
nht chn mt im trong mt phng c ta l (x, y) sao cho
n 6 x, y 6 n. Hai ngi thay phin nhau chn n s tha mn
iu kin trn. Tip theo, h tin hnh ni cc im ny li. Ngi
th nht chn ra hai im vo ni chung bi mt ng cong sao
cho ng ny khng i qua bt c im no trong cc im cn
li v cng khng ct bt c ng cong no c trc . C nh
th. Hi trong 998 s ban u, c bao nhiu s ngi th nht
c chin lc thng?

2. Xy dng m hnh trong gii Ton


Song nh l mt cng c mnh gii nhiu bi ton chng
minh v bi ton m trong t hp. tng chnh ca phng
php ny chnh l thay i cch tip cn trong bi bng mt
con ng, mt cch nhn khc c cc c im tng ng vi
gi thit ban u m vi n, ta c th d dng x l hn. Di
y, chng ta s cng phn tch mt s dng Ton v vic xy
dng m hnh thng gp.

2.1. S dng biu Venn


Bi ton 13. C bao nhiu cch chia tp hp S c n phn t
thnh 2 tp con (c tnh tp rng) sao cho hp ca chng bng S?
Li gii. Ta thy rng vn t ra y kh tng qut v
cc tng m bng truy hi, chng minh bng quy np xut
hin u tin trong trng hp ny. Tuy nhin, nu chng ta
th v mt m hnh ra hnh dung th vn c th sng t
112

Tp ch online ca cng ng nhng ngi yu Ton

hn: Ta c th biu din hai tp A, B nh th bi cc vng trn


v cng vic cn lm l ng vi mi phn t x S, xp n vo
trong cc vng trn . R rng ta c 4 cch xp:
x thuc A nhng khng thuc B.
x thuc B nhng khng thuc A.
x thuc c A v B (nm trong phn giao).
x khng thuc c A v B.
Nhng v yu cu ca bi ton l hp ca hai tp con l S nn
khng th tnh trng hp th 4 trn c. Do , ng vi
mi phn t, ta c ng 3 cch xp vo hai tp hp.
T y, ta c th tnh ra p s ca bi ton l:
3n + 1
3n 1
+1=
.
2
2
Gii thch thm v kt qu ny, ta thy rng c mt vn cn
gii quyt khi m l cc trng hp b trng nhau. Nu ban
u cho sn hai tp con A v B ri th kt qu s l 3n r
rng. Tuy nhin, yu cu y l chia tp S ra thnh hai tp
con, nh th th vic chia y c tnh i xng gia A v B
(tc l cch m x thuc A nhng khng thuc B, x thuc B
nhng khng thuc A trn l ging nhau).
Ch thm c mt trng hp c bit l khi A = B th buc
phi c A = B = S nn ch c mt cch chia. B trng hp
ra, chia i s trng hp ri li cng n vo th s c s
cch chia cn tm.
Cng thc y hon ton c th kim tra vi cc gi tr n nh.
Da vo phn tch trn, cc bn th gii cc bi ton sau: C
bao nhiu cch phn hoch tp hp S gm n phn t thnh hai
tp con? (tp hp S phn hoch thnh hai tp hp A v B khi
A B = , A B = S).

113

Tp ch online ca cng ng nhng ngi yu Ton

M hnh quen thuc trn chnh l biu Venn trong l thuyt


tp hp, tip theo l mt bi ton lin quan c gii bng cch
v m hnh tng t:
Bi ton 14. Khi iu tra kt qu hc tp ca mt lp hc c 45
hc sinh cc mn Ton, L, Ha, ngi ta thy rng:
C 8 hc sinh gii ng 1 trong 3 mn.
C 17 hc sinh gii mn Ton v L nhng khng gii Ha.
C 6 hc sinh gii L v Ha nhng khng gii Ton.
C 12 hc sinh gii Ha v Ton nhng khng gii L.
C 1 hc sinh khng gii mn no.
Hi c tt c bao nhiu hc sinh gii c 3 mn?
Li gii. Di y l mt li gii theo hng lp lun trc tip:
Gi T l tp hp cc th sinh v A, B, C ln lt l cc th sinh
gii c cu Ton, L, Ha. Theo gi thit th:

|A\(B C)| + |B\(C A)| + |C\(A B)| = 19


|(A B)\C| = 18, |(B C)\A| = 10, |(C A)\B| = 12

|T
\(A B C)| = 1
Suy ra |A B C| = 45 1 = 44. Ta cng c
|A\(B C)| = |A| |A (B C)| = |A| (|A| + |B C| |A B C|)
= |A B C| |B C| = |A B C| |B| |C| + |B C|
v
|(B C)\A| = |B C| |(B C) A| = |B C| |A B C| .
Do :
|A\(B C)|+|(B C)\A| = |A B C||A B C||B||C|+2 |B C| .
Tng t, ta cng c
|B\(C A)|+|(C A)\B| = |A B C||A B C||C||A|+2 |C A| ,
|C\(A B)|+|(A B)\C| = |A B C||A B C||A||B|+2 |A B| .
114

Tp ch online ca cng ng nhng ngi yu Ton

Cng cc ng thc trn li, ta c


3 |A B C| 3 |A B C| 2 (|A B C| |A B C|) = 43,
hay
|A B C| |A B C| = 43.

T kt qu ny, ta suy ra |A B C| = 44 43 = 1. Vy c ng 1
hc sinh gii c 3 mn.
Cch khc. R rng vic lp lun trn khng sng sa lm v
i hi bin i cc php tnh trn tp hp kh phc tp. Ta s
dng biu Venn gii quyt bi ton ny nhanh gn hn.
K hiu tp hp cc hc sinh gii nh cc vng trong m hnh
trn. Theo gi thit, ta c

a + b + c = 8
f = 17, d = 6, e = 12

a + b + c + d + e + f + g = 45 1 = 44
Suy ra
g = 44 (8 + 17 + 6 + 12) = 1.
Vy c ng 1 hc sinh gii c 3 mn.
Bi ton 15. Trong mt k thi Ton, ngi ta cho 3 bi 1, 2, 3 v
c 100 th sinh gii c t nht mt bi. Trong cc th sinh gii
c bi 2 th s th sinh gii c bi 1 nhiu gp i s th sinh
gii c bi 3, cn s th ch gii c bi 2 th nhiu gp ba ln
s th sinh cn li. S th sinh gii c t nht mt bi nhng
khng gii c bi 2 l 40. Hi c t nht bao nhiu th sinh gii
c c bi 2 ln bi 3?

115

Tp ch online ca cng ng nhng ngi yu Ton

Li gii. Tng t bi trn, ta k hiu cc vng nh hnh v.


Ta s biu din ln lt cc quan h trong bi cho. S th
sinh gii c t nht 1 bi l a + b + c + d + e + f + g = 100.
Trong cc th sinh gii c bi 2, s th sinh gii c bi 1
nhiu gp i s th sinh gii c bi 3 th:
f + g = 2(d + g) f = 2d + g.
Trong cc th sinh gii c bi 2, s th ch gii c bi 2 th
nhiu gp ba ln s th sinh cn li th b = 3(f + g + d).
S th sinh gii c t nht mt bi nhng khng gii c bi
2 l a + e + c = 40. Do , b + d + f + g = 60 v:

f + g + d = 15
3d + 2g = 15.
f = 2d + g
Ta c s th sinh gii c bi 2 v 3 l:
d+g=

1
1
(3d + 3g) > (3d + 2g) = 5.
3
3

Vy c t nht 5 th sinh gii c c bi 2 ln bi 3.


Di y l mt s bi tp tng t:
Bi ton 16 (IMO, 1996). Trong mt cuc thi ton, c tng cng
3 bi ton l A, B, C. Bit rng c 25 th sinh gii c t nht mt
trong cc bi. Trong cc th sinh khng gii c bi A, s th sinh
gii c bi B nhiu gp i s th sinh gii c bi C. S th
sinh ch gii c bi A nhiu hn 1 n v so vi s th khc gii
c bi A v mt bi khc na. Ngoi ra, c mt na trong s
cc th sinh gii c ch mt bi l khng gii c bi A. Hi c
bao nhiu ngi gii c bi B?

116

Tp ch online ca cng ng nhng ngi yu Ton

Bi ton 17. Cho tp hp S = {1, 2, 3, . . . , n} l tp hp n s


nguyn dng u tin.
a) Hy tm s cch chia S thnh 3 tp con A, B, C sau cho A
B 6= , B C 6= , C A 6= v A B C = .
b) Hy tm s cc b ba cc tp con A, B, C tha mn
A B C = S v B S = .
c) Hy tm s cc b bn cc tp con A, B, C, D tha mn
A B C D = S v B S D = .
d) (Gp g Ton hc 2013) Chng minh rng c 5n b ba tp
hp c th t (X, Y, Z) tha mn ng thi cc tnh cht:
i) X, Y, Z S.

ii) X ((Y Z) (S\Y))

iii) Y ((Z X) (S\Z))

iv) Z ((X Y) (S\X)) .

2.2. Xy dng bng vung


Tip theo, chng ta s tm hiu mt m hnh kh thng dng
na gii cc bi Ton m l xy dng mt bng thch hp
v m theo hai chiu ca bng . Trc tin, ta th xt bi
trong thi chn i tuyn ca trng Ph Thng Nng Khiu
TPHCM nm 2011:
Bi ton 18. Cho hm s f : N N N tha mn f(0, 0) = 0 v
h i  
b
a
.

,
khi a + b .. 2
f
2
2
h i  
f(a, b) =

a
b
.

1 + f
,
khi (a + b 1) .. 2
2
2
a) C bao nhiu s t nhin m sao cho f(2011, m) = 5?
b) Cho s l p v s n N sao cho 1 < p < 2n v A l tp hp
gm p s t nhin khng vt qu 2n 1. Chng minh rng
 2

X
p 1
.
f(a, b) 6 n
4
{a, b}A

117

Tp ch online ca cng ng nhng ngi yu Ton

Li gii. a) Ta xt li gii bng cch xy dng bng nh sau:


Hng 1 gm 2 : 2011 v k.
 
Hng 2 gm 2 : 1005 v k2 .
h
i
Hng 3 gm 2 : 502 v [k/2]
.
2

C nh th, ta c 9 hng tip theo m cc s ct u tin


l 251, 125, 62, 31, 15, 7, 3, 1, 0 v cc s ct tip theo c xy
dng bng cch ly phn nguyn ca mt na s lin trc.
Ta c f(2011, m) ng bng s hng m trong 2 s trong hng
khc tnh chn l (ch rng vi mi cch quy nh tnh chn
l cho s mi hng th lun tn ti k tha mn). C C511 cch
chn ra 5 hng nh th v y cng l p s cn tm.
b) Gi s p s chn ra l a1 , a2 , . . . , ap . Khi :
   a 
f (ai , aj ) = f a2i , 2j nu ai , aj c cng tnh chn l.
   a 
f (ai , aj ) = f a2i , 2j + 1 nu ai , aj khc tnh chn l.

Ta li tip tc xt bng nh sau (c p ct, v nhiu nht l n + 1


hng, trong hng cui cng th cc s u bng 0).
Hng u tin gm ap v a1 .
 
a 
Hng th hai gm 2p v a21 v c th.

Xt hai s ai , , aj , ti hng bt k m hai phn t tng ng ca


ai , aj khc tnh chn l th f ca hai phn t bng f ca hai
phn t ngay di n cng 1. Hin nhin sau nhiu nht n bc
th t n f(0, 0) = 0.
Do , f (ai , aj ) ng bng vi s lng phn t tng ng ca
hai s nm trong cng mt hng m khc tnh chn l.
Xt 1 hng bt k gm p (tr hng cui cng), c s lng
s
cc s i mt khc tnh chn l nhiu nht th s c p1
2
p+1
khc tnh chn l vi 2 s cn li. Khi qua hng ny, th
P
2
f(a, b) tng thm p 41 . V c n ct nh vy nn
 2

X
p 1
max
f(a, b) = n
.
4
Ta c iu phi chng minh.
118

Tp ch online ca cng ng nhng ngi yu Ton

Bi ton tip theo kh in hnh cho phng php ny xut


hin trong k thi chn i tuyn Vit Nam d thi IMO 2001 (s
2001 xut hin trong bi ton c th thay bng mt s nguyn
dng bt k no khc).
Bi ton 19 (VNTST, 2001). Cho dy s (an ) tha mn
0 < an+1 an 6 2001.
Chng minh rng tn ti v s cp s nguyn dng (p, q) tha
mn nu p < q th ap | aq .
Li gii. T cch xc nh dy, ta thy rng s hng tip theo
s khng ln hn s hng lin trc n cng thm 2001 n v.
Nh th, ch cn xt 2002 s nguyn dng lin tip th s c t
nht mt s hng thuc dy, t s l a. Ta xy dng bng
nh sau (vi k hiu a(i, j ) l s hng i, ct j):
a(1, 1) = a, a(1, 2) = a + 1, . . . , a(1, 2002) = a + 2001;
Q
Q
a(2, 1) = 2002
a(1, i) + a(1, 1) , a(2, 2) = 2002
i=1
i=1 a(1, i) + a(1, 2) , . . . ,
Q2002
a(2, 2001) = i=1 a(1, i) + a(1, 2001) ;
...;

Q
a(2002, 1) = 2001
a
+ a(2001, 1) , . . . ,
i=1
Q2002(2001, i)
a(2002, 2002) = i=1 a(2001, i) + a(2001, 2002) .
Bng ny c tt c 2002 ct v 2002 hng; trong , mi s hng
hng th 2 tr i bng tch ca tt c s hng hng lin trc
n cng vi s hng cng ct vi n. R rng, trong mi hng,
c t nht mt s hng thuc dy cho. T theo nguyn l
Dirichlet, ta c iu phi chng minh.
Bi ton 20. Petya v Vasya chi tr chi nh sau: Ban u trn
bn c 11 ng si, mi ng c 10 vin si. Hai ngi thay phin
nhau i, bt u t Petya. Mi mt nc i, ngi chi bc 1, 2
hoc 3 vin si, nhng Petya mi ln bc tt c cc vin si t
mt ng si bt k, cn Vasya lun bc cc vin si t cc ng
khc nhau (nu nh Vasya bc nhiu hn mt vin). Ngi no
n lt mnh khng i c na s thua. Hi ai l ngi lun
m bo c thng li, khng ph thuc vo cch i ngi kia?

119

Tp ch online ca cng ng nhng ngi yu Ton

Li gii. Ta xp 11 ng si thnh bng 10 11 (11 ct tng


ng vi 11 ng si v mi ng c 10 vin si) ri gi 10 vin si
ct u l b A, 10 vin si trn ng cho ca hnh vung
to bi 10 ct sau l b B.

Ta s chng minh rng Vasya c chin lc thng bng cch


bc si nh sau: Mi khi Petya bc x vin si A th Vasya s
bc tng ng x vin si B. Khi Petya bc x vin si mt
trong 10 ct cn li (theo chiu dc) th Vasya s bc x vin si
tng ng (i xng qua ng cho theo chiu ngang).
Tip theo, ta xt bi ton trong thi VMO 2015 va qua. Mt
bi ton kh di v rc ri m thng qua vic m hnh ha, ta
c th x l mt cch nh nhng hn rt nhiu.
Bi ton 21 (VMO, 2015). C m hc sinh n v n hc sinh nam
(m, n > 2) tham gia mt lin hoan song ca. Ti lin hoan song ca,
mi bui biu din mt chng trnh vn ngh. Mi chng trnh
vn ngh bao gm mt s bi ht song ca nam-n m trong ,
mi i nam-n ch ht vi nhau khng qu mt bi v mi hc
sinh u c ht t nht mt bi.
Hai chng trnh c coi l khc nhau nu c mt cp namn ht vi nhau chng trnh ny nhng khng ht vi nhau
chng trnh kia. Lin hoan song ca ch kt thc khi tt c cc
chng trnh khc nhau c th c u c biu din, mi chng
trnh c biu din ng mt ln.
120

Tp ch online ca cng ng nhng ngi yu Ton

a) Mt chng trnh c gi l l thuc vo hc sinh X nu nh


hy tt c cc bi song ca m X tham gia th c t nht mt
hc sinh khc khng c ht bi no trong chng trnh
. Chng minh rng trong tt c cc chng trnh l thuc
vo X th s chng trnh c s l bi ht bng s chng
trnh c s chn bi ht.
b) Chng minh rng ban t chc lin hoan c th sp xp cc
bui biu din sao cho s cc bi ht ti hai bui biu din
lin tip bt k khng cng tnh chn l.
Li gii. a) Ta nh s cc hc sinh n theo th t t 1 n m
v cc hc sinh nam t 1 n n. ng vi mi chng trnh vn
ngh, ta biu din vic ghp cp ca cc cp nam n song ca
thnh mt bng m n gm m hng n ct. Bng s c nh
s 1 hoc 0, trong nm hng i ct j c in s:
S 1 nu bn hc sinh n th i v bn hc sinh nam th
j c ht song ca vi nhau.
S 0 nu bn hc sinh n th i v bn hc sinh nam th
j khng ht song ca vi nhau.
0
1
0
1
1

0
1
0
0
0

1
0
1
1
1

0
0
0
1
1

1
0
0
1
1

Mt bng gi l tt nu trn mi hng v mi ct u phi c t


nht mt s 1. R rng theo bi th tt c cc bng biu din
cho chng trnh u l tt v hc sinh no cng c biu din.
Xt mt hc sinh X no , gi s l n; trng hp hc sinh
nam chng minh tng t. Chng trnh no l thuc hc
sinh X nu nh trn bng tng ng ca n, tn ti t nht 1
ct c ng mt s 1 nm trn hng ca X, ta gi bng ny l l
thuc X v ct nh th l ct l thuc X.
Ta cn chng minh rng, trong cc bng l thuc X, s bng c
s cc s 1 chn bng s bng c s cc s 1 l.
121

Tp ch online ca cng ng nhng ngi yu Ton

Tht vy, xt trng hp trong bng c k ct l thuc X th r


rng k < n v nu khng, ngc li, k = n th ton b cc trn
hng X u l 1, cn tt c cc cn li ca bng u l 0. Do
m > 2 nn tn ti mt dng ton l s 0, mu thun.
Vi k < n, ta b k ct ra khi bng th trn bng s mt i
ng k s 1. Mi trong n k cn li ca hng X s c
in s 0 hoc 1 ty v cc ct cn li u cn t nht mt s
1 na khng thuc hng X. Do , nu ta b lun hng X i th
bng cn li vn l tt. Suy ra s bng l thuc X trong trng
hp ny s l 2nk nhn vi s lng bng tt c kch thc
(m 1) (n k) cn li. Trong mi bng , ta chn mt bt
k ca hng X v thay i s t 0 1, 1 0 th s dn n thay
i tnh chn l ca s cc s 1 trn bng. Do , r rng tn
ti mt song nh i t tp hp cc bng l thuc X c s cc s
1 chn n tp hp cc bng l thuc X c s cc s 1 l.
Do , s lng hai loi bng ny l bng nhau. ng vi mi
k = 1, n 1 v cc cch chn k ct ph thuc X th s lng
bng c s 1 l v chn u bng nhau, v th nn tng s bng
c s cc s 1 l bng vi bng c s cc s 1 chn. Ta c iu
phi chng minh.
b) Tip theo, ta t f(m, n) v g(m, n) ln lt l s cc bng tt
m n c chn v l cc s 1. Xt mt hc sinh n ty , t l
X. Ta xt cc trng hp sau:
Nu tn ti mt ct no l thuc X th theo cu a, s
bng c s cc 1 chn bng s bng c s cc s 1 l, t
gi tr ny l h(m, n).
Nu khng tn ti ct no l thuc X th b hng tng
ng ca X i, ta cn li mt bng tt c m 1 hng, n ct.
Mt khc, s trng hp m hng X c s l v c s chn
in s 1 ln lt l
X
L=
Can
a1 (mod 2)

v
C=

Can (do hng X khng th ton l s 0).

a0 (mod 2), a>0

122

Tp ch online ca cng ng nhng ngi yu Ton

n
Ta bit rng (1 + x)n = C0n +C1n x+C2n x2 + +Cn
n x nn vi x = 1,
ta c ngay L = C + 1. Ch l tnh chn l ca s cc s 1 thuc
dng X s quyt nh n tnh chn l ca bng cn li nn ta
c cng thc truy hi sau:

f(m, n) = h(m, n) + L g(m 1, n) + C f(m 1, n)
g(m, n) = h(m, n) + L f(m 1, n) + C g(m 1, n)

Do :


f(m, n) g(m, n) = (L C) g(m 1, n) f(m 1, n)
= g(m 1, n) f(m 1, n).
Lp li qu trnh ny n khi s hng v s ct nh nht c th,
tc l m = n = 2, ta c


f(m, n) g(m, n) = (1)m+n4 f(2, 2) g(2, 2) .
m trc tip, ta thy c f(2, 2) = 3, g(2, 2) = 4 nn suy ra
f(m, n) g(m, n) = (1)m+n3 .
T ta thy rng s lng ca hai loi bng khng vt qu
1 v c th sp xp cc bng theo th t chn, l an xen
c iu kin bi. Ta cng c iu phi chng minh.
tin hnh x l bi ton, ta cn phi tm cch m hnh ha
n thnh dng thch hp, trong , mi rng buc u c th
biu din c. y, vic lp thnh bng nh phn cho mt
chng trnh c th nh trn c l l la chn sng sa, n
gin nht. Cch ny cng quen thuc vi hu ht cc hc sinh
v bi ton v t mu bng, in s trn bng cng tng i
quen thuc (t nht l trong kim tra ca Vin Ton trong
thng 12 va qua cng c mt bi nh th).
Bn cnh , ta cng c th dng cc cch tip cn khc nh:
Dng bipartite graph hay cn gi l th lng phn,
th 2 phe ri thao tc trn cc nh v cnh.
Dng cch chia thnh cc b (nam, n, chng trnh) ri
m bng 2 cch v tnh tng, hoc cch ni khc l xt
hm s f(a, b, c) : (A B C) {0, 1} vi A, B, C ln lt l
tp hp n, nam v cc chng trnh.
123

Tp ch online ca cng ng nhng ngi yu Ton

Suy cho cng, cc cch ny cng l song nh, chuyn i cch


tip cn nhng bn cht vn th: bng tt chnh l mt ma trn
cnh k ca th v n cng chnh l tng hp cc gi tr m
hm trn nhn c vi cng mt gi tr c C.
Mt nhn xt c bn nhng mang tnh quyt nh trong c hai
a v b ca bi ton l: Trong cc tp hp con ca mt tp hp
c n > 1 phn t, s tp hp con c l phn t bng s tp hp
con c chn phn t v cng bng 2n1 . ny c th gii quyt
d dng bng song nh hoc m trc tip v dng nh thc
Newton.
So snh vi cc bi ton trong nhng nm gn y, c th coi
y l bi ton m phn a h tr rt tt cho phn b vi vic chia
mt trng hp ln thnh hai trng hp nh. Phn a va c
tc dng gi , va ng gp mt phn vo lp lun ca phn
b. Ci kh ca bi ton ny c l ch s dng tng truy
hi quy np, v i n cng.
Chng ta s t ra cu hi sau y mt cch t nhin: C
tt c bao nhiu chng trnh trong lin hoan vn ngh? y
l mt bi ton khng d v khng cho ra kt qu dng tng
minh nhng c th c th sinh no m trong bi thi ca
mnh, c gng m s lng ny.
Ta c th gii quyt bng nguyn l b tr nh sau (ta vn xt
bng c m hng v n ct): Ta gi mt bng m mi hng u c
t nht mt s 1 l bng tt theo hng, gi tp hp cc bng
ny l P. Gi A l tp hp cc bng tt theo hng nhng li c
mt ct no khng c s 1 no, Ai l tp hp cc bng tt
theo hng nhng ct th i li khng c s 1 no. Ta thy:
A = A1 A2 A3 An .
R rng s bng tt cn tm chnh l |P| |A|. Trc ht, ta tnh
s bng tt theo hng. Ch rng mi c hai cch in l 0
hoc 1 nn hng c n s c 2n cch, tuy nhin, loi tr trng
hp tt c cc u l 0 ra th c 2n 1 cch. Cc hng khc
cng tng t th nn c tng cng (2n 1)m bng tt theo hng
hay |P| = (2n 1)m . Do cc ct bnh ng vi nhau nn ta c th
m i din mt trng hp no suy ra cc trng hp
cn li.
124

Tp ch online ca cng ng nhng ngi yu Ton

Gi s c k ct no trong cc ct 1, 2, 3, . . . , n l khng cha


s 1 no vi 1 6 k < n. Trn mi hng s cn li n k in
vo cc s 0 hoc 1 sao cho c t nht mt s 1 (do k thuc
cc ct kia u c in s 0), s cch in cho mi hng l
m
2nk 1 v cho c bng l (2nk 1) . T , theo nguyn l b
tr, ta c
|A| = |A1 A2 A3 An | =

n
X
k=1

m
Ckn (1)k 2nk 1 .

Do , ta c s bng tt cn tm l:
n

(2 1)

n
X

nk

Cn (1) (2

1)

n
X

Ckn (1)k (2nk 1) .

k=0

k=1

Vi tnh bnh ng ca m, n, ta cng suy ra c kt qu sau:


n
X

nk

Cn (1) (2

1)

m
X
k=0

k=0

n
Ckm (1)k 2mk 1 .

Di y l mt s bi tp p dng:
Bi ton 22. Cho cc s thc dng phn bit x1 , x2 , x3 , . . . , xn .
Chng minh rng tn ti s thc a sao cho cc tng sau a + x1 ,
a + x2 , a + x3 , . . . , a + xn u l cc s v t.
Bi ton 23 (Russia, 1996). Duma, c 1600 i biu tham
gia vo 16000 y ban v mi y ban c ng 80 ngi tham gia.
Chng minh rng, trong cc y ban trn, c 2 y ban no m
c t nht 4 i biu tham gia chung.
Bi ton 24 (CMO, 1996). C 8 ca s tham gia vn ngh vi tng
cng m bui ha nhc. Mi bui c 4 ca s tham gia v s ln tham
gia ca mi cp ca s l nh nhau. Tnh gi tr nh nht ca m.

2.3. Chuyn i m hnh


Trn thc t, nhiu bi ton c th c s chuyn i qua li
gia vic pht biu dng tp hp nh trn v vic xy dng m
hnh. in hnh nh bi ton trong VMO trn c th
125

Tp ch online ca cng ng nhng ngi yu Ton

xut pht t mt bi ton trong th no xy dng theo


tnh hung thc t trn. Vic chuyn i m hnh c tinh
t hay khng ty thuc vo tnh hung s dng tng ng bao
nhiu so vi bi ton gc.
Xut pht t bi ton quen thuc: Chng minh rng khng th
lt mt nn nh 10 10 bng cch vin gch 1 4.
Cch gii ca bi ny l t mu cc c dng (l, l) trong bng,
nh th th t c 25 . Tuy nhin, mi vin gch 14 nh trn
phi chim 2 c t mu, tc l s c t mu nm trong
phi chn. iu mu thun ny cho thy ta khng th lt
gch nn nh theo cch trn c.
Bi ton ny c th thay s 10 (s chn nhng khng chia ht
cho 4) thnh cc s tng t nh 50, 2010, 2014, . . . th vn cho
cu tr li tng t vi cch gii tng t.
Ta th thay i bi ton theo kiu ni cc im trong mt phng
thu c bi ton sau:
Bi ton 25. Trong mt phng, cho tp hp A gm 20102 im
phn bit c nh s t 1 n 20102 sao cho ba im bt k no
trong chng cng khng thng hng. Mt t gic (li hoc lm)
c gi l p nu cc nh ca n thuc A v c nh s
bng 4 s tha mn mt trong hai iu kin sau:
l 4 s t nhin cch nhau 2010 n v.
l 4 s t nhin lin tip v nu trong c cha s chia
ht cho 2010 th s phi l ln nht.
Ni tt c cc im thuc tp hp A li vi nhau sao cho im
no thuc A cng thuc ng mt t gic. Tm s ln nht t gic
p c to thnh.
R rng cch ni cc im nh trn c th c m phng bng
4 s nm trn mt vin gch 1 4 nh trn.
Ta c th gii chi tit bi ton ny nh sau:
Li gii. Xt mt bng vung gm 2010 2010 vung con
c in cc s theo th t t trn xung v t tri sang (xem
trang tip) nh sau:
126

Tp ch online ca cng ng nhng ngi yu Ton

1
2
3
...
2011 2012 2013 . . .
4021 4022 4023 . . .
...
...
... ...
...
...
... ...

2009 2010
4019 4020
6029 6030
...
...
. . . 20102

Trc ht, ta s chng minh rng khng th chia tt c 20102


im cho thnh cc t gic p c. R rng 4 s trn
cc nh ca cc t gic p tng ng vi 4 s b che i trn
bng vung khi t mt mnh ba hnh ch nht kch thc
1 4 vo . Ta s chng minh rng khng th che ht ton b
bng vung ny bng cc hnh ch nht 1 4. Tht vy, ta
t mu cc vung nm ct chn v hng chn. Do bng c
2
20102 vung nn s vung b t mu l 2010
, l s l.
4
Gi s ngc li rng ta c th lp kn c c bng vung
bng cc mnh ba. Khi , mi mnh ba s che i hoc hai
vung hoc khng c vung no ca bng vung, tc l
lun c mt s chn vung b che i; do , s vung b
che i trn bng l mt s chn. T ta thy c mu thun.
Do , khng th che ht bng vung ny bng cc hnh ch
nht 1 4 c.
By gi, gi k l s t gic p ln nht cn tm th k 6 1010024.
Ta s chng minh k = 1010024 bng cch ch ra cch dng cc
mnh ba che kn bng vung. Tht vy, chia bng vung
thnh hai phn:
Phn 1 gm 2008 ct u, ta xp cc mnh ba theo cc
hng, mi hng c ng 502 mnh ba. Khi , ta s c th
che kn ht phn 1 bi cc mnh ba.
Phn 2 gm 2 ct cui, ta xp ni tip cc mnh ba t trn
xung di th cui cng s cn li mt vung 2 2 gc
di cng ca bng.
Nh vy, ta dng 1010024 mnh ba che c ti a 20102 4
vung ca bng. T , ta thy, s t gic p ln nht cn
tm l k = 1010024.
Chng ta th xt bi ton sau: C bao nhiu cch lt mt hnh
ch nht kch thc 2 n bi cc vin gch: hnh ch I (hnh ch
127

Tp ch online ca cng ng nhng ngi yu Ton

nht kch thc 1 2) v hnh ch L (hnh vung 2 2 b i mt


)? Nu thay hnh ch nht 2 n bng mt hnh khc th s cch
lt thu c s l bao nhiu?
Qua , ta thy rng vic chuyn i cc m hnh, t cc im,
cc hnh trong mt phng n cc s trong mt tp hp v
ngc li gip cho bi ton c mt dng v mi kh th v.
Chng ta th nhc n mt bi ton trong k thi IMO 1983: T
tt c cc im nm trn cnh ca tam gic u bi hai mu l
xanh v . Hi vi mi cch t mu nh th, c lun tn ti mt
tam gic vung c ba nh c t cng mu hay khng?

Bi ton ny c th gii quyt khng qu kh khn bng cch


xt cc im chia cc cnh ca tam gic ABC theo t s 2 : 1, c
kh nhiu tam gic vung c to thnh t 6 im ny v nh
vy m ta c th p dng nguyn l Dirichlet gii quyt bi
ton. Tuy nhin, nu chng ta thay i cch pht biu thnh
ta trong mt phng th bi ton s th v hn nhiu.
Trong mt phng ta Oxy,
mt
 tam gic
 u ABC
 c ta
ca cc nh ln lt l A 0, 3 , B 1, 0 , C 1, 0 th phng

trnh ng thng cha cc cnh AB, AC c th vit l 3x +

y = 3, ng thng cha cnh BC trng vi trc Ox. Hn na,


r rng di ca BC c th thay bng mt s bt k no khc
nn ta c bi ton sau:
Bi ton 26. Trong mt phng
ta Oxy, xt phn mt phng
gii hn bi th ca y = 3 (a |x|) , trong a > 0 v trc
honh. Gi S l tp hp cc im nm trn bin ca min .
128

Tp ch online ca cng ng nhng ngi yu Ton

Chng minh rng vi mi cch chia S thnh hai tp hp con ri


nhau th lun tn ti ba im l nh ca mt tam gic vung.
Cch gii bi ny cng hon ton tng t nh bi kia. Vn
mu cht y l nhn nhn c bao li trn thc cht l
mt tam gic u v cch chia tp con cng ging nh vic t
cc im bi hai mu.
Xut pht t bi ton phn hoch s nguyn: Hi c bao nhiu
cch biu din s nguyn dng n thnh tng c tnh th t cc
ca cc s nguyn dng khng vt qu k?, ta c th pht
biu thnh mt bi ton kh th v nh sau:
Bi ton 27. Cho mt cy k-phn y m mi nh khng
phi l u c ng k nh con. Mi cnh ni 1 nh vi cc nh
con ca n c nh s t 1 n k. Hi c bao nhiu ng i t
gc n l sao cho tng cc s nh trn cc cnh thuc ng
i ng bng n?
Tt nhin c nhiu cch chuyn m hnh m cch c sc,
th v nht vn l chuyn i theo graph, tuy nhin, l mt
cu chuyn di khc. y, ta ch xt mt s m hnh chuyn
i v bi ton m phng mt gc no ca vn ny.

3. Khai thc v m rng bi ton


Trong phn ny, ta s xt mt s bi ton n gin v t
pht trin ln thnh cc bi kh dn dn hoc cc dng lin
quan. Khai thc cc bi ton lun gip ta ch ng hn trong
qu trnh x l cc bi ton, khng s gp cc dng Ton l v
c th tm c cc lin kt gia bi ton mi vi nhng hiu
bit c sn ca mnh.
Bi ton 28 (Mt s vn v t mu v in s). d
1) Bn c 7 7 c cc c in cc s l 1, 1. Hi c xy ra
trng hp tng cc s cc hng, ct v ng cho u
phn bit hay khng?
Khng th xy ra do gi tr ca cc tng nhn c l t
7 n 7, c 15 s; trong khi cc gi tr cn phn bit ca
7 hng, 7 ct v 2 ng cho th c n 16.
129

Tp ch online ca cng ng nhng ngi yu Ton

2) Cu hi tng t nu khng tnh hai ng cho.


Ta chng minh c trng hp trn cng khng th xy
ra do cc tng trn ch nhn gi tr l t 7 n 7. R rng
c tt c 8 gi tr nh th l 7, 5, 3, 1, 1, 3, 5, 7.
3) Tng t 2) nhng tm s gi tr phn bit ln nht m cc
tng ca hng v ca ct c th nhn c.
Ta s xy dng mt bng ch ra rng tn ti mt bng
s c th nhn 8 gi tr lit k trn nh sau:
1
1
1
1
1
1
1

1
1
1
1
1
1
1

1
1
1
1
1
1
1

1
1
1
1
1
1
1

1
1
1
1
1
1
1

1
1
1
1
1
1
1

1
1
1
1
1
1
1

Ta thy rng 6 ct u nhn cc gi tr l t 7 n 3, ct


cui nhn gi tr l 7; trong khi , hng cui nhn gi
tr l 5 nn tha mn.
4) Cu hi tng t 3) nhng thay bng 7 7 bi 2015 2015.
T bi ton trn, ta d on kt qu l 2016. Vn l phn
b cc s thch hp sao cho m bo nhn 2016 s.
Ta c th p dng tng xy dng cho bng 7 7 vo bi
ton ny; c th l cho cc ct nhn cc gi tr thch hp
t 2015 n 2011, thm s 2015 ct cui. Ch rng s
cn li l 2013 to thnh bi 2014 s 1 v 1 s 1 v c
sp xp cho xut hin hng cui bng cch a cc s
1 mi ct xung cui l hon tt.
5) Vn l vn in s vo bng 7 7 nhng gi ta s quan
tm n tch. Hi c th xy ra trng hp tch cc s thuc
cc hng u l 1 v cc ct u l 1 hay khng?
Khng c do nu ly tch ca cc hng nhn li vi nhau
th c A = 1; cn ly tch ca cc s ca cc ct nhn
130

Tp ch online ca cng ng nhng ngi yu Ton

li vi nhau th c B = 1. Tuy nhin, A, B li u l tch


ca tt c cc s trn bng nn v l.
6) Cu hi tng t 5) nhng thay iu kin thnh tch mi
hng v mi ct u l 1. Hi c bao nhiu bng nh th?
Ta thy rng trong tnh hung ny, bng l tn ti (v t
nht, ta c bng gm ton s 1). Quan st cch in vo
bng vi kch thc nh hn, chng hn 3 3, 4 4, . . . ta
thy rng vic in s ch ph thuc vo bng con gc
trn bn tri; hng cui v ct cui u c th in mt
cch xc nh v duy nht (ch rng cui cng in
c). T y suy ra s bng cn tm l 236 (do mi ca
bng trn c hai cch in l 1 hoc 1).
7) ( thi th VMO Vin Ton) Cho bng m n vi m, n Z+ .
Hi c bao nhiu cch t mu cc vung ca bng sao cho
s lng c t mi ct v mi hng u chn?
R rng bi ny vi bi trn l mt, chng qua khc nhau
cch xy dng vn . Bng cch lp li lp lun tng
t, ta c p s l 2(m1)(n1) .
8) (Colombia MO) C bao nhiu cch in s vo bn c kch
thc 8 8 bi cc s 1 v 1 sao cho tng cc s cc hnh
vung con 2 2 u bng 0?
Tng t tng ca hai bi trn nhng y, yu t
quyt nh khng ph thuc vo bng vung nh trn
na m li l mt hng no ca bng, chng hn l
hng u tin. Nu hng c cc s 1, 1 xen k nhau
th r rng cc hng cn li cng s c in tng t
hoc ngc li v nh th th mi hng s c hai cch
in, ta s c 28 cch in.
Trong trng hp tn ti hai s ging nhau ng cnh
nhau th d thy cc s cn li xc nh duy nht theo
hng , y ta c 28 2 cch in (loi i hai cch in
xen k). Vy tng cng c 28 + 28 2 = 510 cch in.

131

Tp ch online ca cng ng nhng ngi yu Ton

Cn li l hai bi tp t luyn:
9) C bao nhiu cch in s vo bn c 4 4 bi s 1 v 1
sao cho tng cc s mi hng v mi ct u bng 0?
10) (AIME, 2007) C bao nhiu cch t mu 12 ca bng 6 4
sao cho mi hng c hai v mi ct c ba c t?
Bi ton 29 (V tp con v hon v). d
1) Cho tp hp S = {1, 2, 3, . . . , 2n}. Hi c bao nhiu tp con T
ca S sao cho T khng cha hai phn t a, b m |a b| = n?
Ta chia cc s vo bng nh bn di:
1
2
3
...
n+1 n+2 n+3 ...

n
2n

R rng mi ct s cho ta 3 cch to tp con l: ch chn


s trn, ch chn s di hoc khng chn s no c.
Ch thm rng chng c mt yu t quan trng l cc
ct c lp vi nhau nn p s bi ton s l 3n .
2) Cu hi tng t nu thay iu kin l a + b = 2n + 1.
Xy dng bng tng t nhng i vic ghp cp trong cc
ct li (cc s i v 2n + 1 i s c ghp vi nhau), p s
bi ton vn l 3n .
3) Cu hi tng t 1) nu thay 2n bi s n ty , c th l: Cho
tp hp S = {1, 2, 3, . . . n}. Hi c bao nhiu tp con T ca
S

n
sao cho T khng cha hai phn t a, b m |a b| = 2 ?
Ta xt hai trng hp:

Nu n = 2m l s chn th tng t 1, p s l 3m .

Cn nu n = 2m + 1 l s l th r rng cc s trong
b (1, m + 1, 2m + 1) s c lin h vi nhau; ta thy c 5
cch chn cho cc s trong b ny: chn tng s hoc
khng chn s no hoc chn s 1 v 2m + 1; cc s
cn li vn nh c. p s l 5 3m1 .
132

Tp ch online ca cng ng nhng ngi yu Ton

4) Cu hi tng t 2) nu thay 2n bi n ty .
Trong trng hp chn, p s vn nh c; nu n = 2m + 1
l, ta thy s m + 1 c c chn vo tp con hay khng
cng khng nh hng nn p s l 2 3m .
5) Gii li cu u tin bng cch s dng nguyn l b tr.
Ta thy rng tng chia bng nh trn l mt dng xy
dng m hnh kh tt. Tuy nhin, ta hon ton c th s
dng mt cch m t nhin l dng nguyn l b tr
thc hin cng vic ny.
Gi A l tp hp cc tp con cha t nht mt cp s (a, b)
m |a b| = n. Gi Ai l tp con c cha cp (i, i + n) vi
1 6 i 6 n. Khi , ta c
A = A1 A2 A3 An .
Theo nguyn l b tr th:
X
X
X
|A| =
|Ai |
|Ai Aj | +
|Ai Aj Ak |
1 set

2 sets

3 sets

Tuy nhin, do tnh bnh ng nn ta ch cn tnh i din


mt tp hp no . Ta c |A1 | = 22n2 (do c nh hai s
1 v n + 1 vo tp con, cc phn t cn li c 2 cch chn
l thuc tp con hay l khng). Tng t |A1 A2 | = 22n4
v c nh th. Khi , s tp hp con cn tnh s l:
n 0
22n |A| = 22n C1n 22n2 + C2n 22n4 C3n 22n6 + + Cn
n (1) 2 .

Ch rng tng cui chnh l khai trin nh thc Newton


n
ca (22 1) = 3n .
6) (IMO, 1989) Cu hi tng t nu thay tp con bi hai phn
t lin tip ca hon v. Chng minh rng s hon v ny
nhiu hn s hon v khng tha mn tnh cht tng ng.
Ta cng thc hin vic t tng t trn v c kt qu
l C1n 2 (2n 1)! C2n 22 (2n 2)! + C3n 23 (2n 3)!
133

Tp ch online ca cng ng nhng ngi yu Ton

Tuy nhin, nh gi trc tip cho tng ny l khng d.


Ta c th ct tng ngay on tr v a v nh gi:
C1n 2 (2n 1)! C2n 22 (2n 2)! >

1
(2n!).
2

Tuy nhin, iu ny c th c thc hin d dng bng


cc bin i i s.
7) Vi n > 2, tnh s hon v ca a1 , a2 , a3 , . . . , an ca cc s
{1, 2, 3, . . . , n} tha ai+1 ai 6 1 vi mi i = 1, 2, 3, . . . , n 1.
Gi Sn l s cc hon v dng ny. Gi s ai = n th do
iu kin ai+1 ai 6 1 nn ai1 = n 1 v c th, suy ra
a1 = n i, cc s cn li l 1, 2, 3, . . . , n i 1 to thnh
hon v tha mn iu kin cho nhng c n i 1 phn
t v l Sni1 . Do 1 6 i 6 n nn ta c
Sn =

n1
X
i=0

Sni = S1 + S2 + S3 + + Sn1 .

Ta cng c S1 = 0, S2 = 2 nn bng quy np, ta chng minh


c Sn = 2n1 vi mi n > 1.
8) ( chn i tuyn KHTN, 2010) Cu hi tng t 7) nhng
b sung thm iu kin ai 6= i vi mi i = 1, 2, 3, . . . , n.
y l mt bi kh v tm c p s ng cng v
cng rc ri. gii quyt, ta s m cc hon v tha mn
iu kin bi 7) nhng khng tha mn iu kin bi
8), t l iu kin T , tc l trong hon v c t nht mt
ch s i m ai = i.
Ta thy rng nu tn ti i 6= j m ai = i, aj = j th vi mi k
m i 6 k 6 j th ak = k. Gi x, y ln lt l s nh nht, ln
nht tha mn ai = i, 1 6 i 6 n vi 1 6 x < y 6 n. Ta nhn
xt rng vi cc s ak m k < x th ch c th nhn cc gi
tr t y + 1 n n; tng t, vi cc s ak m k > y th ch
c th nhn cc gi tr t 1 n x 1.
Khi , d thy rng nu hai i lng x 1, n y khc
nhau th khng th tn ti cch sp xp nn ta phi lun
c x 1 = n y hay x + y = n + 1.
134

Tp ch online ca cng ng nhng ngi yu Ton



Vi mi x m 1 6 x 6 n+1
, tng t cch tnh s hon v
2
tha mn iu kin bi 7) trn, ta thy s hon v tha
mn l 22(x2) nu x > 2 v bng 1 nu x = 1. Khi , s
hon v tha mn iu kin T l:
1+

n+1
[X
2 ]

2(x2)

n+1
[X
2 ]

=1+

i=2

x2

i=2

n+1
n+1
4[ 2 ] + 2
4[ 2 ] 1
=1+
=
.
3
3

Vy s hon v cn tm chnh l:
n1

n+1
4[ 2 ]1 + 2

.
3

9) (VMO, 2009) Cho s nguyn dng n > 1. K hiu T l tp


hp 2n s nguyn dng u tin. Hi c bao nhiu tp con
S ca T tha mn tnh cht: trong S khng tn ti hai s a, b
m |a b| {1, n}?
p s ca bi ton ny l:
n

n

3 + 2 1 + 2 + 3 2 1 2 2(1)n
.
4
Bi ton 30 (Mt s bi ton m hnh hc). d
1) Cho bng vung 6 6. C bao nhiu hnh ch nht?
Ch cn chn ra 2 hng, 2 ct trong bng l c hnh ch
2
nht, p s l (C26 ) .
2) Cu hi tng t nu thay hnh ch nht bi hnh vung.
m cc loi hnh vung kch thc 1 1, 2 2, . . . , 6 6
pht hin ra quy lut v s lng ca chng l 62 , 52 , 42 , . . . ,
12 ri cng li.
3) Cu hi tng t 1) nu hnh ch nht thay bi tam gic
vung, tam gic vung cn.
Ch s lng hnh vung gp 4 ln s tam gic vung
cn v s lng hnh ch nht cng gp 4 ln s tam gic
vung thng thng.
135

Tp ch online ca cng ng nhng ngi yu Ton

4) Cu hi tng t nu thay bi hnh vung c cnh khng


nht thit song song vi cnh ca hnh vung ban u.
Ch rng s c mt s hnh vung nghing, chng s ni
tip trong cc hnh vung thng thng. Ta ch cn tnh
xem vi mi hnh vung thng thng, c tt c bao nhiu
hnh vung nghing l c c cu tr li.
5) Cho li tam gic nh hnh v bn di. Hi trong c bao
nhiu hnh bnh hnh?

Ta chiu cc im ln mt ng thng song song vi 1


trong cc cnh ca tam gic cho, gi s l cnh BC.
m cc hnh bnh hnh c 2 cnh song song vi 2 cnh
cn li ca tam gic ban u, gi s l AB, AC bng cch
chn ra 4 im theo th t trn ng thng chiu.
R rng vi tam gic c cnh c chia thnh n phn th
s cch s l C4n+2 nn tng cng c tt c 3C4n+2 .
6) Cng li tam gic trn, hi c bao nhiu tam gic u?
m bng truy hi v b tr, t Sn l s tam gic trong
trng hp chia cnh thnh n phn th ta c cng thc l:

S(2n) = 3 S(2n 1) 3 S(2n 2) + S(2n 3) + 2
S(2n + 1) = 3 S(2n) 3 S(2n 1) + S(2n 2) + 1
136

Tp ch online ca cng ng nhng ngi yu Ton

Cng thc trn c th hiu d dng l vic thu nh s cnh


li 1 n v th tng ng vi mi nh, ta c 1 tam gic; khi
thc hin cng s tam gic cc nh th c phn chung,
tr ra, ri li b vo. Sau cng, ta cng thm tam gic ln
ngoi. Tuy nhin, ch rng trong trng hp s chn,
tam gic to bi trung im 3 cnh cha c xt vo nn
thay v cng 1, ta cng thm 2. p s bi ton s l:

n(n + 2)(2n + 1)

S(n) =
,
n 0 (mod 2)
8

S(n) = n(n + 2)(2n + 1) 1 , n 1 (mod 2)


8
7) Cu hi tng t 6) nhng cho php v thm cnh.
p s ca bi ton ny kh th v. S tam gic u bng
1
s hnh bnh hnh v l C4n+2 .
3
8) Khi chia mt a gic li hoc lm c n cnh thnh cc tam
gic th c bao nhiu tam gic tng cng?
p s bi ton l n 2. C th chng minh bng quy np.
9) Hi c bao nhiu cch chia mt a gic li thnh cc tam
gic mt cch ty ?
y l mt bi ton kh rc ri, chng minh bng quy np
1
Cn
v p s chnh l s Catalan ni ting n+1
2n .
10) Cu hi tng t 9) nhng thay tam gic ty bi tam gic
c t nht mt cnh l cnh ca a gic ban u?

137

Tp ch online ca cng ng nhng ngi yu Ton

138

Tp ch online ca cng ng nhng ngi yu Ton

CC VN
C IN V HIN I
Trn Nam Dng (HKHTN, HQG Tp HCM)

Chuyn mc ny dnh cho cc vn c in v hin i c


trnh by di dng cc bi ton xu chui. c th l chui
cc bi gii bi ton ng chu, chng minh ng thc Euler
2
k hiu 1 + 212 + 312 + = 6 , mt chui bi ton vn tr. . . Cch
trnh by xut pht t nhng vn n gin, d hiu, nhng
khi nim mi s c nh ngha lun trong bi c th c
tng i c lp. V mi mt chui bi s nu ra nhng vn
nht nh, c th l gii quyt mt bi ton kinh in hay
nu ra nhng gi thuyt mi, nhng vn mi.

1. Phng trnh Diophant 1


ton ngh cho Hi ngh ma h ca cuc thi ton gia cc
thnh ph nm 2013, xut bi S.Grigoriev, K.Kuyumzhiyan,
A.Petukhov, A.Semchenkov.
nh l 1 (Gauss). Mt s nguyn dng c th biu din c
di dng tng ca ba bnh phng khi v ch khi n c khng c
dng 4n (8m 1).
Bi ton 1. Chng minh rng cc phng trnh
2x2 + 2xy y2 = 1,
x2 xy + y2 = 2
khng c nghim nguyn.
Bi ton 2. Chng minh rng phng trnh:
x2 + 1000xy + 1000y2 = 2001
c v s nghim nguyn.
139

(1)
(2)

Tp ch online ca cng ng nhng ngi yu Ton

Bi ton 3. Chng minh rng cc phng trnh:


x2 2y2 = 1,
x2 3y2 = 1,
x2 6y2 = 1

(1)
(2)
(3)

c v s nghim nguyn.
Bi ton 4. C nh s nguyn t l p. Chng minh rng phng
trnh x2 py2 = 1 c nghim nguyn khi v ch khi p c s d
l 1 khi chia cho 4.
Bi ton 5. Chng minh rng vi mi m, s nghim ca cc
phng trnh sau l nh nhau:
x2 xy + y2 = m,
3x2 + 9xy + 7y2 = m.

(1)
(2)

Bi ton 6. Chng minh rng vi mi n Z, phng trnh:


x2 + y2 = n
c nghim nguyn khi v ch khi n c nghim hu t.
Bi ton 7. Hy nu v d mt phng trnh bc hai vi h s
nguyn, c nghim hu t nhng khng c nghim nguyn.
Bi ton 8. Chng minh rng vi mi s nguyn dng a, b, tn
ti v s cc s t nhin m sao cho phng trnh ax2 + by2 = m
khng c nghim nguyn.
Bi ton 9. Chng minh rng vi mi s nguyn m phng
trnh x2 + 2y2 3z2 = m khng c nghim nguyn.

2. Cc dng ton phng


Mt a thc thun nht bc hai ca n bin s c gi l mt
dng ton phng. Theo nh ngha, dng ton phng f i
din s m nu phng trnh f = m c nghim nghuyn khc
0 (tc l nghim m trong khng phi tt c cc bin u
bng 0, lu , khng phi dng ton phng no cng i din
0). Hai dng ton phng c gi l tng ng nu chng
cng i din mt tp hp s.
140

Tp ch online ca cng ng nhng ngi yu Ton

Bi ton 10. Hy m t tt c cc s nguyn, c i din bi


cc dng x2 + y2 , x2 y2 v x2 + xy + y2 .
Bi ton 11. Chng minh rng cc dng ton phng: f(x, y),
f(xy, y), f(x, yx), f(x, y), f(x, y) i mt tng ng nhau.
Bi ton 12. d
1) Chng minh rng cc dng ton phng x2 + y2 , x2 + xy + y2
khng tng ng.
2) Chng minh rng cc dng ton phng 4x2 6xy + 5y2
khng tng ng vi dng ton phng ax2 + by2 vi mi
s nguyn a v b.
nh ngha 1. Dng ton phng c gi l:
1) Xc nh dng nu n ch i din cho cc s dng.
2) Xc nh khng m nu n ch i din cho cc s > 0.
3) Xc nh m nu n ch i din cho cc s m.
4) Khng xc nh nu n i din c s dng ln s m.
Bi ton 13. Hy nu v d mt dng xc nh khng m m
khng phi xc nh dng.

3. S hc m rng: s p-adic
nh l 2 (Legendre). Mi s nguyn dng u c th biu din
di dng tng bnh phng ca 4 s nguyn.
Bi ton 14. Cho m v n l cc s nguyn khng chnh phng.
Nu phng trnh:
z2 mx2 ny2 = 0
(1)
c nghim hu t khc 0 th cc iu kin sau c tha mn:
1) t nht mt trong hai s m, n dng.
2) m l thng d bnh phng theo modulo n.
3) n l thng d bnh phng theo modulo m.
141

Tp ch online ca cng ng nhng ngi yu Ton

Bi ton 15. Hy a nh l tng qut v dng ton phng


hai bin v li gii ca phng trnh dng (1).
nh ngha 2. Biu thc dng:
ak pk + ak+1 pk+1 + + an pn +

(2)

(k l s nguyn bt k, ai Z) c gi l s p-adic. Nu k 6 0,
th ta gi (2) l s nguyn p-adic.
Bi ton 16. Chng minh rng phng trnh vi h s nguyn
f = 0 c nghim trong Zp nu v ch nu n c nghim trong h
thng d modulo pn vi mi n Z>0 .
Bi ton 17. Khi no s p-adic dng (2) bng 0?
Bi ton 18. Chng minh rng tch ca hai s p-adic khc 0
khng bng 0.
Bi ton 19. Chng minh rng Q Qp vi mi s nguyn t p
(chng minh rng vi mi cp s nguyn (m, n) khc 0, tn ti
s p-adic x sao cho nx = m).
Bi ton 20. Chng minh rng 1 l s chnh phng trong
tp hp cc s p-adic khi v ch khi p ng d 1 theo modulo 4.
Bi ton 21. Hy m t cc s p-adic l s chnh phng.
Bi ton 22. Chng minh rng mi s 3-adic khc 0 c dng
x2 , hay 2x2 , hay 3x2 , hay 6x2 vi s 3-adic x no .
Bi ton 23. Cho p l s nguyn t l, cn x1 , . . . , x5 l cc s
p-adic khc 0. Chng minh rng xxij l s chnh phng trong
tp cc s p-adic vi i, j no (1 6 i < j 6 5).
Bi ton 24. Chng minh rng vi mi s nguyn t l p tn
ti cc s p-adic khc 0 l x1 , x2 , x3 , . . . , xp1 sao cho:
x21 + x22 + + x2p1 + 1 = 0.
Bi ton 25. Chng minh rng phng trnh x2 + x + 1 = 0 c
ng hai nghim trong tp cc s nguyn 7-adic.
Bi ton 26. Chng minh rng phng trnh x2 + y2 = 1 c
nghim trong cc s p-adic vi mi s nguyn t l p.
142

Tp ch online ca cng ng nhng ngi yu Ton

nh l 3 (Nguyn l Minkowsky-Hasse). Phng trnh bc hai


f = 0 ca mt s bin c nghim hu t khi v ch khi n ng
thi c nghim trong:
Tp hp cc s thc.
Tp hp cc s p-adic (:= Qp ) vi mi s nguyn t p.
Bi ton 27. Chng minh nguyn l Minkowsky-Hasse cho
phng trnh 1 hoc 2 n s.
nh ngha 3. t (a, b)p = 1, nu z2 ax2 by2 = 0 c nghim padic, v t (a, b)p = 1 trong trng hp ngc li. Gi tr (a, b)p
c gi l k hiu Hilbert ca cp (a, b) i vi s nguyn t p.
Bi ton 28. Chng minh rng vi k hiu Hilbert, ta c:
1) (a, b)p = (b, a)p .
2) (a, c2 )p = 1,
3) (a, a)p = 1, (a, 1 a)p = 1.

4) (a, b)p = (a, ab)p = a, (1 a)b p .

Bi ton 29. Gi s (a, b)p = 1. Chng minh rng vi mi a 0 , ta


u c (a 0 , b)p = (aa 0 , b)p .
nh ngha 4. vit gn cng thc tng minh cho k hiu
Hilbert, ta cn n k hiu Legendre ( px ) xc nh vi mi s
nguyn x v s nguyn t p. N bng 1, 1 hay 0 ty thuc vo x
c phi l thng d bnh phng, khng thng d bnh phng
hay 0 theo mun p. Vi
t l p, k hiu Legendre c
 s nguyn
p1
x
tnh theo cng thc p = x 2 (mod p).

Bi ton 30. Cho p l s nguyn t l, a = p u, b = p v, trong


, , u, v l cc s nguyn sao cho u v v nguyn t cng
nhau vi p. Chng minh rng
   
(p1)
u
v
(a, b)p = (1) 2
.
p
p
Bi ton 31. Tm cng thc tng minh cho (a, b)2 vi mi cp
s nguyn a, b.
143

Tp ch online ca cng ng nhng ngi yu Ton

Bi ton 32. Chng minh rng (a, b)p (a, b0 )p = (a, bb0 )p vi
mi s nguyn a, b, b0 .
Bi ton 33. Chng minh rng phng trnh ax2 + by2 = c
(a, b, c l cc tham s, cn x, y l cc n s) c nghim trong
tp hp cc s p-adic nu v ch nu (c, ab)p = (a, b)p .
Bi ton 34. C nh a thc thun nht:
f = a1 x21 + a2 x22 + + an x2n (n > 2),
trong a1 , . . . , an 6= 0. t d = a1 a2 an v
Y
(ai , aj )p .
=

(3)

i<j

Chng minh rng phng trnh f = 0 c nghim khc 0 trong


tp cc s p-adic khi v ch khi xy ra mt trong cc iu sau:
1) n = 2 v d l s chnh phng trong Qp .
2) n = 3 v (1, d) = .
3) n = 4 v d 6= 2 , hoc l d = 2 v = (1, 1)p .
4) n > 5 (tc l nu f ph thuc vo 5 hay nhiu bin th
phng trnh f = 0 c nghim khc 0 trong Qp vi mi p.)
T nh l 34 hy suy ra mnh sau:
Bi ton 35. C nh a thc thun nht:
f = a1 x21 + a2 x22 + + an x2n (n > 2),
trong a1 , . . . , an 6= 0 v s nguyn a 6= 0. nh ngha d v bi
cng thc (3). Chng minh rng phng trnh f = a c nghim
trong tp cc s p-adic khi v ch khi mt trong cc iu kin
sau y c tha mn:
1) n = 1, v s

a
d

l s chnh phng trong Qp ;

2) n = 2 v (a, d)p = ;
3) n = 3 v ad khng chnh phng trong Qp hoc l ad chnh
phng v = (1, d)p ;
144

Tp ch online ca cng ng nhng ngi yu Ton

4) n > 4 (iu ny c ngha l nu f ph thuc vo 4 hay


nhiu hn bin s th phng trnh f = a c nghim khc
0 trong Qp vi mi p).
Bi ton 36. Chng minh nguyn l Minkowsky-Hasse.
Bi ton 37. S dng bi ton 35 v nguyn l MinkowskyHasse, hy chng minh rng s nguyn n biu din c di
dng tng bnh phng ca ba s hu t khi v ch khi n
khng c dng 4a (8b 1), tc l khi n khng phi l s chnh
phng trong Q2 .
Bi ton 38. C nh s nguyn n. Chng minh rng nu tn
ti cc s hu t x, y, z sao cho x2 + y2 + z2 = n, th cng tn ti
cc s nguyn x0 , y0 , z0 sao cho (x 0 )2 + (y 0 )2 + (z 0 )2 = n. T y hy
suy ra kt lun ca nh l Gauss.
Bi ton 39. T nh l Gauss hy suy ra nh l Legendre.

145

Tp ch online ca cng ng nhng ngi yu Ton

146

Tp ch online ca cng ng nhng ngi yu Ton

BI TON CHUYN XE BUS


L T ng Khoa (i hc FPT, Tp HCM)

1. M u
Xe but l mt trong nhng phng tin giao thng huyt mch
ca thnh ph, xp x ln n 33 nghn chuyn mi ngy. V vy,
lp tuyn xe but mi v ti u tuyn xe but c l mt trong
nhng u tin hng u ca thnh ph.
Mi tuyn xe but thng c biu din bi mt on thng c
di c nh v mt s trm xe but nm gia hai u mt.
Ngi dn mun cc trm nm sao ti u thi gian di
chuyn ca h. V vy, i tng cn c ti u l thi gian di
chuyn trung bnh ca tt c ngi dn.

2. M hnh
Chng ta xt m hnh sau:
Gi s c mt con ng di L km. Dn s c phn b u
nhau trn sut con ng ny. Chng ta cn tm s trm xe
but v v tr ti u ca chng gim thiu thi gian di chuyn
trung bnh m mt hnh khch phi b ra, i t mt im
bt k trn ng n mt im bt k khc.
i t P n Q, mt hnh khch phi i b n trm xe but
gn P nht, sau ln xe v dng li trm xe but gn Q nht,
ri i b n Q. Nu c hai trm xe but cch P mt khong nh
nhau, hnh khch s chn trm gim thiu s trm phi i
(tng t nu c hai trm cch Q mt khong nh nhau).
Tc i b l W km/h, tc ca xe but l B km/h, v mt
chic xe but phi dnh khong S gi nhn thm hoc b ra
147

Tp ch online ca cng ng nhng ngi yu Ton

cc hnh khch mi trm. Chng ta k hiu T (P, Q) l thi


gian m hnh khch phi b ra i t P n Q.
Chng hn ta xt bn sau vi di qung ng L = 20 km:

C 5 trm xe but v 3 v tr ngu nhin trn bn , ta tnh


thi gian di chuyn gia cc v tr ny:
1. i t P n R, hnh khch cn i 1 km n trm 2, sau
qua 2 trm vi di 14 km xung trm 4, ri i b 1
km n R. Tng thi gian l:
T (P, R) =

1
14
1
2
14
+
+ 2S +
=
+
+ 2S.
W
B
W
W
B

2. Tng t, i t Q n R, ta cn thi gian:


T (Q, R) = T (P, R) +

1
3
14
=
+
+ 2S.
W
W
B

3. i t P n Q, hnh khch s i b 1 km n trm 2, i


xe but 0 km n trm 2 (ngha l khng lm g c), ri i
b 2 km n Q. Tng thi gian l:
T (P, Q) =

0
2
3
1
+ + 0S +
=
.
W B
W
W

(Trng hp ny ch dng minh ha thut Ton i,


khng c ngha thc t.)
Chng ta thng nht mt vi iu kin v k hiu:
Lun c mt trm xe but 2 u mt ca on ng.
Gi s v tr ca cc trm l 0 = x1 < < xn1 < xn = L,
khi ta biu din tuyn xe but A qua b sp xp trm
l A = (x1 , x2 , . . . , xn1 , xn ).
148

Tp ch online ca cng ng nhng ngi yu Ton

Tuyn xe but A cng c th c biu din thng qua b


A = (d1 , d2 , . . . , dn1 , dn ), vi di = xi xi1 v i = 1, 2, . . . , n.
K hiu E (A) l thi gian trung bnh i t mt im bt
k ny n mt im bt k khc trn tuyn xe but A, khi
b sp xp trm ca tuyn ny c c nh.

3. Cu hi
Bi ton 1. d
1) C nh n v b qua thi gian n v th hnh khch
mi trm. Chng minh rng b sp xp ti u xy ra khi
cc trm xe but cch u nhau. Ngha l E(A) t gi tr
ti thiu khi d1 = d2 = . . . = dn1 = dn .
2) Xt trng hp L = 20, W = 5, B = 20, S = 0.05. Tm gi tr
ca n ti u ha E(A), bit A c n + 1 trm xe but cch
u nhau. (Do S khc 0 nn khng m bo y l cch
sp xp ti u nht vi mt gi tr n bt k.)
Bi ton 2. M hnh ca chng ta cn nhiu khuyt im:
1) Hnh khch hon ton c th i b trc tip nu 2 im
i v n gn nhau.
2) Hnh khch thng xuyn n mt s ni nh siu th, c
quan, nh ring, .v.v. hn mt s im trung gian khc.
3) Dn s phn b cha hn ng u trn ton tuyn.
Da trn cu 1.1) v 1.2), hy a ra mt m hnh c th gii
quyt ba vn trn. n gin, bn vn c th gi s tuyn
xe but l mt ng thng.

149

Tp ch online ca cng ng nhng ngi yu Ton

150

Tp ch online ca cng ng nhng ngi yu Ton

NHN XT V K THI VMO 2015


1. V thi
K thi chn hc sinh gii quc gia mn Ton nm hc 20142015 (VMO 2015) din ra trong 2 ngy 08 v 09/01/2015. V
tng quan, ta c th thy rng thi nm nay hay, ph hp vi
vic tuyn chn hc sinh gii.
Cc vn t ra trong kh cn bn, quen thuc nhng
cng c nhng kh khn nht nh tng bi. V d bi t
hp khai thc ch quen thuc v cc s chia ht cho 3 c
cc ch s thuc 1 tp hp (dng cn bc 3 ca n v hoc
truy hi), nhng a ch s 0 vo gy cht rc ri. Bi 6 s
hc th khai thc nh l Sylvester v biu din dng t hp
tuyn tnh ax + by. Bi bt ng thc rt nh nhng (so vi bt
ng thc khng nm ngoi), c th gii bng kin thc THCS
nhng cng gy kh cho khng t th sinh.
Nm nay c mt im c bit l ch c mt bi hnh hc, li l
bi kh kh nn c th s khin mt s bn gii hnh cha c
c hi bc l ht s trng ca mnh. Thay vo mt bi hnh l
mt bi t hp di, khng qu kh v bn cht nhng i hi
kh nng c hiu ca th sinh. Nhiu th sinh chia s rng:
"Em khng hiu bi t hp h hi g?". Qu tht, kh nng c
hiu, xy dng v chuyn i m hnh l im yu c hu ca
hc sinh Vit Nam.
i vo chi tit, ta c th im qua tng bi nh sau:
Ngy 1 kh c bn, gm cc vn t nhiu u c gii thiu
trong chng trnh Ton chuyn mt cch i tr.
Bi 1 (dy s) c cu a qu d v quen thuc, cu b li
kh khn k thut nht nh, cn s dng nh l kp,
n iu hoc b nh x co. Nhiu th sinh b ln su
151

Tp ch online ca cng ng nhng ngi yu Ton

vo cu b ca bi ton dn n thiu thi gian gii


quyt cc cu cn li.
Bi 2 (bt ng thc) th khng qu d cng khng qu
kh, n l mt bt ng thc i xng, ng bc, dng
tng i ph bin vi cc hc sinh. i bin kh cn
xong l c th nhn ngay ra bt ng thc Schur bc 4 v
AM-GM. Bi ny c rt nhiu cch gii nn s rt tic cho
hc sinh no b v sau, v y l v chnh ca bi ton.
y l mt bi ton kh hp l tng xng vi v tr ca
n trong thi.
Bi 3 (t hp) khai thc ch quen thuc ( xut hin
trong cc thi Romania 2003, Ph thng nng khiu
2009, Lm ng 2014). C hi rc ri ch s 0 nhng
li c "gii" bng iu kin n < 10k (ch khng phi c
m ch s). Phng php cn n v gii quyt gn nhng
cng cn trnh by cht ch. Phng php truy hi s gy
kh mt cht v c n 3 dy. bi ny, c l th sinh lm
trn vn khng nhiu, nhng gim kho chm s kh mt.
Bi 4 (hnh hc phng) l bi hnh duy nht, c hai cu
u mc trung bnh kh, nhng s hc sinh lm trn
vn bi ny s khng nhiu. C th thy ch v phng
tch, trc ng phng vn ng vai tr ch o trong sut
cc nm gn y.
ngy thi th hai, thi so vi ngy u tin "gy sc", kh
c v k thut ln t duy. Nguyn nhn l khng c bi hnh
v bi t hp pht biu qu di. Hai bi 5, 6 tuy quen thuc
nhng li l phn m cc th sinh t . V tng th ngy
2 hay hn ngy 1.
Bi 5 (dy s, a thc) tng i c bn. Dng phng
trnh c trng hoc quy np d dng tm c cng thc
fn = (2x 1)n + (x + 1)n . Kiu bi ton chia ht ny kh
ging vi nhng bi chia ht trong s nguyn. Cch lm
truyn thng l khai thc tnh tun hon ca s d. Tuy
nhin, thc t nhiu th sinh khng c phng hng g.
Bi 6 (s hc) s khng kh khn lm nu quen
vi nh l Sylvester. C hai bi ton mu trc l IMO
152

Tp ch online ca cng ng nhng ngi yu Ton

1983 v Vietnam TST 2000. Cng nh cc nm, bi s hc


t khi xut hin, nhng nu c th n s l mt bi kh.
Tuy nhin, nu cha bit nh hng s dng nh l ny
th y qu l mt th thch thc s.
Bi 7 (t hp) kh di, quan h gia cc khi nim kh
ri v d dn n hiu nhm. Hc sinh cn chuyn v mt
m hnh ton hc no (bng, graph hoc hm s)
thy r hn vn . Thc ra v bn cht th n ch l mt
bi ton m c th gii bng song nh v quy np.
Nh vy, nhn chung th nm nay hay. Ngy 1 ra tht c
bn v quen cho i tr. Ngy 2 gy kh v phn loi. Qua
kho st mt s i tuyn, th sinh nh gi ny va sc
v t nhiu cng gii quyt c mt s ni dung trong bi.
D on nm nay im t gii khuyn khch s vo khong
13-15, cn im lt vo vng 2 thi chn i tuyn IMO l 24.
Nm nay chc s t gii nht.

2. V kt qu
Cng tc chm thi c trin khai 1 tun sau k thi. Kt qu
cho thy d bi ngy 1 kh c bn nhng cc th sinh vn
gp rt nhiu kh khn. Vi ngy 2, bi s 7 hu nh b b
trng. Mt s th sinh c tip cn nhng khng c tin trin g
ng k trong li gii. iu ny mt mt ni ln im yu c
hu ca hc sinh Vit Nam v t hp nhng cng gi ln mt
suy ngh rng vic a bi ton mc tru tng cao nh
bi 7 mc thi HSG quc gia l cha cn thit.
Thc t chm thi cho thy bi ton ny hon ton khng c
ngha trong vic phn loi hc sinh, k cc cc hc sinh tp
trn. Ni cch khc, c th b bi ny m khng nh hng
n th bc. Vi tnh hung l thc t ch cn 34 im nh vy,
im trung v ri vo 12-13 im, chnh l im m BGK chn
ly xt gii.
Kt qu c tng cng 222 gii, trong c 1 gii nht (dnh cho
cc th sinh c im t 30.0 tr ln), 47 gii nh (dnh cho cc
th sinh c im t 21.5 n 29.5) 80 gii ba (dnh cho cc th
153

Tp ch online ca cng ng nhng ngi yu Ton

sinh c im t 17 n 21.0) v 94 gii khuyn khch (dnh cho


cc th sinh c im t 13 n 16.5).
Trong k thi nm nay, thnh tch xut sc nht thuc v trng
THPT chuyn HQG H Ni vi 1 gii nht, 3 gii nh, 5 gii
ba, 1 khuyn khch. Gii nht duy nht HQG HN, cng l duy
nht ca k thi thuc v Nguyn Tun Hi ng, hc sinh lp
12 chuyn Ton.
Cc i khc cng cng c thnh tch n tng bao gm: Ngh
An (8 nh, 1 ba, 1 KK), Nng (4 nh, 3 ba, 1 KK), HSP H Ni
(4 nh, 1 ba, 4 khuyn khch) H Tnh (3 nh, 6 ba, 1 KK). Tip
theo l cc i PTNK HQG Tp HCM, Nam nh, Hi Dng,
Bc Ninh. c bit Bnh nh, sau nhiu nm c thnh tch
khng my ni bt cng c 2 gii nh, 2 gii ba, 2 gii KK.
K thi nm nay ghi nhn s vn ln ca cc i vn t c
nhc tn mc thi HSG quc gia mn Ton: Tuyn Quang,
in Bin, Lai Chu, Lo Cai, Bnh Dng, Vnh Long, Lm
ng. c bit i tuyn k Nng (mt trong nhng i tuyn
tr nht) c 2 gii quc gia u tin v mn ton: 2 gii
khuyn khch. Mt s khch l ln lao cho cc bn tr yu ton
vng cao xa xm.
Tip ni theo k thi HSG quc gia, theo k hoch ca B gio
dc v o to, vo cui thng 3 ti y, 49 th sinh bao gm
cc hc sinh t gii nht v gii nh ca k th HSG quc gia
v em Nguyn Th Hon, thnh vin i tuyn Vit Nam d
IMO 2014 (huy chng vng) s tham d k thi chn i tuyn
Olympic Ton Vit Nam (Vietnam TST) tham d IMO 2015 t
chc ti Thi Lan vo thng 7-2015.
Phn b a l ca Top 49 nh sau: Cc trng THPT chuyn
cp B chim 11 sut, cc trng ng bng Bc B v duyn
hi chim 17 sut, cc trng Bc Trung b chim 12 sut, cc
trng t Nng tr vo (tr PTNK thuc HQG Tp HCM)
chim 9 sut. Vi tnh cht hon ton c lp ca k thi chn
i tuyn, kh nng l c 4 khi ny u s p gp i din
vo Top 6 nm nay.

154

You might also like